random NPTE thingssss

Ace your homework & exams now with Quizwiz!

sx assoc with hypoparathyroidism

- hypocalcemia - cardiac arrhythmia

contraindications for cancer rehab

- severe nausea, vomiting, diarrhea within 24-36 hours - acute infection and fever above 100 deg F - chest pain - dizziness, lightheadedness, disorientation, confusion, blurred vision, ataxia

A physical therapist examines a patient with carpal tunnel syndrome. As part of the examination, the therapist assesses end-feel. The therapist classifies the end-feel associated with wrist extension as firm. Which of the following provides the MOST logical explanation for this finding?

tension in the palmar radiocarpal ligament & the palmar joint capsule firm end feel with wrist flexion can occur with tension in the dorsal radiocarpal ligament and the dorsal joint capsule

the inferior portion of the capsule is tightened when:

the arm is raised overhead

pulse pressure

the difference between systolic and diastolic blood pressure (pressure responsible for perfusion of the tissue)

anterior apprehension test or crank test

the therapist applies pressure at the end range of shoulder abduction and external rotation. traumatic instability problems causing gross or anatomical instability of the shoulder; positive anterior apprehension test indicates anterior instability.

papilloma

tumors that grow from epithelial tissues

Upward Rotators of the Scapula

upper trapezius, lower trapezius, serratus anterior

pain associated with urinary calculi most often occurs due to blockage of:

ureter (pain from ureter contracting to dislodge the calculi(

when should sodium bicarb be administered to patient?

when they're in respiratory acidosis bc it brings the pH down

A physical therapist discusses the plan of care for a patient who has spinal stenosis with the referring physician. During the discussion, the physician shows the therapist an image of the patient's spine obtained through computed tomography. What color would vertebrae appear when using this imaging technique?

white

neurogenic shock

will occur in patients with SCI above T6 - hypotension - bradycardia - cyanosis, warm-dry extremities - decreased cardiac output, peripheral vasodilation, and venous pooling

After a stroke, a patient cannot see on the nasal side of the eyes (R side of the L eye, L side of the R eye) where is the lesion?

= bitemporal hemianopia lesion in optic chiasm

How are superficial reflexes graded?

present or absent

tarsal tunnel syndrome signs

posterior tibial n is entrapped in the tunnel formed between the flexor retinaculum and the medial mall - medial ankle pain - paresthesias - medial longitudinal arch pain - medial heel pain

A physical therapist treats a patient diagnosed with posterior tibial tendon dysfunction. When observing the posterior aspect of the patient's lower leg and ankle in standing, which of the following findings is the MOST probable for the therapist to observe? 1) forefoot adduction 2) hindfoot valgus 3) gastroc hypertrophy 4) swelling in lateral ankle

posterior tibial tendon dysfunction is the primary cause of medial ankle pain- occurs due to the inability of the post tib tendon to support the medial longitudinal arch patient tends to exhibit flat foot and may feel like the ankle rolls inward commonly characterized by hindfoot valgus

pre vs post central gyrus

pre= motor post= SS

DF stop too soft

premature knee flexion in late stance

In a clinical trial, patient height is measured in centimeters and weight is measured in kilograms. The physical therapist should recognize that height and weight are measured on which scale of measurement?

ratio 4 types: nominal, ordinal, interval, ratio 1) nominal: mutually exclusive categories (male, female, color) 2) ordinal: order, ranked (sensation, balance, MMT) 3) interval: no true 0 (temp) 4) ratio: has a true 0

A therapist is examining a patient's straight leg raise by taking their leg into hip flexion, knee extension and ankle dorsiflexion with inversion. Which nerve is MOST likely being biased in this position?

sural nerve

contraindications of cryotherapy

- cold intolerance - cold urticaria - cryoglobulinemia - infection - over an area of compromised circulation - over regenerating peripheral nerves - paroxysmal cold hemoglobinuria - peripheral vascular disease - Raynaud's

weeks 4-10 protocol for CL

- full pain-free ROM - improve kinesthetic awareness, esp in closed chain - dynamic control of L knee

ASIA A, C1-4 SCI. What are some functional capabilities, equipment, and AD that would be characteristic for this patient?

- full time attendant - direct care - wheelchair cushion - head/trunk support - portable vent dependent with wc positioning, environmental control units (ECU), brain computer interface (BCI)

glut med tendonitis characteristics

- hip abd weakness - (+) trendelenberg - pain/tenderness around greater trochanter - instability during the stance phase of gait

sx associated with hyperparathyroidism

- hyperreflexia - mm atrophy - gout - arthralgia - fx - osteopenia - bone pain

causes of delayed knee flexion during late stance in patient with BKA

- keel of sach too long - loe shoe heel - excessive PF - DF stop too stiff - socket too far posterior

C3-C4 SCI intact mm

- partial diaphragm - scalenes - lev scap - traps

large intestinal and colonic dysfunction symptoms

- poorly localized pain in the mid abdomen and sacrum - constipation, rectal pain, bloody diarrhea - pain may be relieved with defecation or passing of gas (pain will not be relieved in small intestinal dysfunction)

sacroiliac arthritis

- B/L SIJ referring to gluts - prolonged morning stiffness - coughing is painful -typical PROM - typical AROM except may have a slight limitation in flexion. - positive sacral stress test - palpation tenderness over SIJ - normal lab tests (with ankylosing spondylitis, ROM is decreased and lab tests would show an increased ESR/HLA)

Kypho lordotic posture

- Head held forward with cervical spine hyperextended - scapulae may be protracted - increased lumbar lordosis, and increased thoracic kyphosis - pelvis anteriorly tilted, hip flexed, knee hyperextended

parameters for e-stim (HVPC) with non-infected/clean/proliferative wounds - electrode - pulse frequency - pulse duration - amplitude - treatment time

- POSITIVE polarity pulse frequency: 60-125 pulse duration 40-100 preset amplitude to produce comfortable tingling treatment time 45-60 mins

pronator teres

- Pain and paresthesia along anterior aspect of elbow, medial forearm, palmar aspect of 1st thru 3rd phalanges - Pain aggravated by pressure distal to cubital crease - Pain with resisted elbow flexion, wrist flexion, and forearm rotation with palm down - (-) Tinel

what drains into the R lymphatic duct?

- R UE - R upper trunk above diaphragm - R half of neck and had

R thoracic scoliosis: what will you see

- R shoulder elevated - L hip elevated - vertebral bodies rotated to same side (R) - TP rotated to opposite side (L) - R rib hump - R scap prominence - L lateral flexion bias of thoracic spine in standing

e-stim parameters for mm re-ed - pulse frequency - pulse duration - amplitude - on:off - ramp time - treatment time

- pulse freq 35-50 pps pulse duration: - 150-200 small - 200-350 large mm amplitude sufficient for mm activity on:off depends on activit ramp time at least 2 secs treatment time depends on functional activity

PEDI

197 item inventory used to detect the functional limitations on the participation of the patient and the need for assistance by the caregivers

A patient is treated using pulsed wave ultrasound at 1.2 W/cm2 for 7 minutes. The specific parameters of the pulsed wave are 2 msec on time and 8 msec off time for one pulse period. The duty cycle should be recorded as what percentage? 1.10% 2.20% 3.25% 4.50%

2 sec on, total 10 sec 1/5: 20%

Aquatic therapy is contraindicated for patients with a vital capacity of:

< 1 L

what can cause significant terminal impact during late swing

= abrupt knee extension taut knee extension aid will cause abrupt knee extension

metabolic syndrome

>3 of these risk factors - waist size (40+ M, 35+ F) - elevated BP (>130/85) - triglycerides >150 - FBP >100 - HDL (<50 F, <40 M)

1) A patient has a boutonniere deformity that is being treated non-surgically. To maintain maximal hand function: a. DIP flexion with PIP supported in ext b. DIP ext with PIP supported in ext c. PIP flex with DIP supported in flexion d. PIP flexion with DIP supported in ext

A

1) A patient with shoulder pain has a (+) painful arc and (+) Hawkins Kennedy and (+) infraspinatus. Choose the most appropriate intervention a. Isometric shoulder retraction with shoulder in 5 deg of extension b. Active shoulder ER with arm in 90 deg ABD c. Scap retraction in 120 deg shoulder flexion d. Active shoulder flexion with resistance thru 150 deg

A

Paraffin: 1) temp 2) treatment time

1) 125-127 deg 2) 15-20 mins * used for small irregular distal areas (wrists/hands/feet)

Hot packs 1) temperature 2) # of layers 3) treatment

1) 165-170 2) 6-8 layers 3) 20-30 times * greatest risk for burns is in the first 5 mins

which 3 phases of gait is 0 deg of knee extension most important for:

1) initial contact 2) end of terminal stance 3) end of terminal swing

stages of RA: 1-4

1) no destructive changes on radiograph but osteoporosis may be present 2) radiograph: osteoporosis and slight subchondral bone and cartilage destruction. joint mobility dec but no deformities. may see nodules, tenosynovitis, and adjacent mm atrophy 3) cartilage and bone destruction, joint deformities (ulnar deviation) 4) fibrous + bony ankylosis

NY Heart Association Classifications for heart failure

1) no limitations 2) slight limitations in physical activities. ordinary activities may result in fatigue, palpitations, dyspnea or angina 3) marked limitations, comfy at rest, less than ordinary activity causes sx 4) may have sx at rest, unable to carry on any physical activity without discomfort

Meniere's disease

Abnormal condition within the labyrinth of the inner ear that can lead to a progressive loss of hearing. - dizziness or vertigo - hearing loss - tinnitus (ringing in the ears) - sensation of fullness

A physical therapist is educating a patient about the side effects of long term use of oral corticosteroids. Which of the following is LEAST likely to be included in the list? Select one: a. Osteoporosis b. Cushing's syndrome c. Decrease in blood sugar d. Severe hypotension

C

closure of mitral and tricuspid valves

S1

S1 lesion

S1 lesions would cause paresthesias along the lateral two toes, lateral foot, lateral leg to knee, plantar aspect of foot along with weakness of the calf and hamstrings, wasting of gluteals, peroneals, plantar flexors and limited straight leg raise.

which heart sound represents vibrations of the ventricular wall during the rapid influx of blood during atrial contraction from an exaggerated atrial contraction

S4 commonly heard with MI, htn, L ventricular hypertrophy, pulmonary stenosis, pulm htn, increased left ventricular and diastolic pressure

Which one of the following sympathetic cardiovascular changes occurs in a patient with long-standing type 1 diabetes mellitus?

fixed HR Long term complication of Type 1 diabetes mellitus is cardiac denervation syndrome which results in a fixed HR that is unresponsive to exercise, stress or sleep.

loss of lordosis with the pelvis in posterior pelvic tilt, hips and knees hyperextended, forward head posture.

flatback form of kyphosis

A physical therapist palpates the lunate bone on a patient post wrist injury. Which wrist motion will BEST allow the therapist to facilitate palpation of the lunate? 1.Extension 2.Flexion 3.Radial deviation 4.Ulnar deviation

flexion lunate is located in the center of the proximal row between the scaphoid and the triquetrum. palpate just distal to the radial tubercle

empty can test

flexion and internal rotation of the shoulder followed by resistance to observe for weakening of the supraspinatus muscle

Empty Can Test

flexion and internal rotation of the shoulder followed by resistance to observe for weakening of the supraspinatus muscle IR, ABD, 90 deg of flexion, and then therapist resists nerve implicated: suprascapular

A physical therapist examines a patient with multidirectional instability of the shoulder. Which position would be the MOST appropriate when assessing posterior instability of the shoulder?

flexion to 90 deg with full IR

posterior GH glide:

flexion, IR, horiz ADD

D2 flexion pattern LE

flexion, abd, IR, knee flexion, ankle DF and eversion (think posterior hip precautions for D2)

Phelp's Test

for the gracilis (ADD, flex, IR) prone, knees fully extended, passively abduct the hips as much as possible. Repeat with the knee flexed. If abduction is ↑ then the gracilis is tight

mobilization to improve IR of forearm

forearm IR= pronation PPP= posterior glide of radius at proximal radioulnar joint to improve pronation posterolateral glide and anterior roll of radius

talipes equinovarus

forefoot ADDuctus hindfoot varus ankle equinus

A physical therapist examines the foot of a patient with lower extremity pain. After placing the foot in subtalar neutral, the therapist determines that the medial border of the foot along the first metatarsal is higher than the lateral border of the foot along the fifth metatarsal. This position would MOST appropriately be documented as which of the following malalignments? 1) forefoot varus 2) forefoot valgus 3) rearfoot varus 4) rearfoot valgus

forefoot varus = inverted position of forefoot in relation to rearfoot may see low arches

A 35-year-old female made an appointment for chief complaints of muscle weakness. During the evaluation, the patient explained that she had difficulty keeping her eyelids up, changes in vision, new onset of difficulty swallowing, and speaking and an inability at times to use arms. Upon examination, the physician detected diplopia, limited facial expression, and significant weakness in bilateral upper extremities which was greater than the weakness in bilateral lower extremities. These symptoms are MOST likely to be associated with which of the following conditions?

myasthenia gravis Myasthenia Gravis (MG) is characterized by the inability to keep eyelids up, vision changes, difficulty swallowing and speaking and inability at times to use arms, diplopia, limited facial expression, and significant weakness in bilateral upper extremities greater than weakness in bilateral lower extremities.

side effects of chemo

nausea weight loss/anorexia vomiting diarrhea ULCERS hemorrhage bone marrow suppression, anemia leukemia thrombocytopenia fatigue skin rashes neuropathies phlebitis (vein inflammation) hair loss *** MYOPATHY IS NOT A SIDE EFFECT ***

cervical radiculopathy

neck pain radiating to u/l arm (+) spurling (+) ULTT 2A pain relieved with distraction ipsilateral c/s rotation <60 deg pain with ipsilateral side bending acute: cervical traction subacute: strengthening of deep cervical flexors

a patient presents to the clinic with monocular vision loss. where is the lesion? - optic chiasm - optic tract - optic nerve - occipital cortex

optic nerve optic chiasm lesion= loss of heteronymous hemianopsia optic tract lesion= homonymous hemianopia occipital cortex lesion= macular sparing

LCPD

osteonecrosis of the acetabulum/hip LLD dec extension and ABD during AROM limited in IR during flexion and ext AROM

2 major corticosteroid side effects

osteoporosis hyperglycemia

Bilateral Vestibular Hypofunction

ototoxicity (ABx), meningitis, autoimmune disorder, head trauma, tumor on CN 8 - oscillopsia (visual blurring with head movements) - disequilibrium - NO nausea - gait ataxia

A Physical Therapist is observing a patient's EMG examination. The patient is at rest and the Physical Therapist notices biphasic waves with a sharp initial positive deflection (below baseline) followed by a slow negative phase. How should the Physical Therapist classify this finding? 1) polyphasic potential 2) positive sharp wave 3) complex repetitive discharges 4) F wave

positive sharp wave: characterized by waves that are typically biphasic with a sharp initial positive deflection (below baseline) following by a slow negative phase * polyphasic potentials are seen during voluntary contraction, NOT rest. they are classified as motor unit potentials with 5 or more phases * f waves are seen with nerve conduction velocity testing and allows one to study the proximal nerve segments. the wave is elicited by the supramaximal stimulation of a peripheral nerve at a distal site * complex repetitive discharges: characterized by an extended train of potentials with the same or nearly the same waveform. will present with regular and repetitive waveforms

A physical therapist measures a patient's shoulder complex medial rotation in the standard position. The therapist records the patient's shoulder medial rotation as 0-70 degrees and classifies the end-feel as firm. Which portion of the joint capsule is PRIMARILY responsible for the firm end-feel? (anterior, posterior, superior, inferior)

posterior

What direction of glide should the physical therapist use to mobilize the first metacarpal on the trapezium in order to increase carpometacarpal abduction?

posterior

A physical therapist reads in the medical record that radiographs confirmed the presence of a reverse Hill-Sachs fracture. What injury would MOST likely be associated with this type of fracture? 1.Anterior glenohumeral dislocation 2.Posterior glenohumeral dislocation 3.Rotator cuff tear 4.Biceps tendon rupture

posterior GH dislocation reverse hill sachs: impaction of anterior medial humeral head. common with posterior GH dislocation hill sachs: impaction of posterior superior humeral head. associated with anterior GH disloc

Which nerve is commonly injured in the Arcade of Frohse?

posterior interosseous nerve

A physical therapist reviews the medical record of a patient who has experienced recurrent angina. A recent entry indicated that an exercise stress test ordered by the physician was positive. What is the MOST accurate interpretation of this finding? 1.Presence of balanced oxygen demand and supply 2.Presence of ischemia 3.Presence of normal vital signs 4.Presence of cardiac arrhythmias

presence of ischemia

e-stim parameters for edema reduction using mm pump: - pulse frequency - pulse duration - amplitude - on:off - ramp time - treatment time

pulse frequency: 35-50 pulse duration: - 150-200 small mm - 200-350 large mm Amplitude till visible contractions on:off EQUAL ramp time at least 1 sec 30 min treatment time, 2x/day

Which of the following is MOST appropriate to screen for in a patient with an acute exacerbation of Guillain-Barre Syndrome?

sensation (respiratory function will be preserved initially, fatigue is not as threatening)

T2 lesion

sensation on apex of axilla

appearance of superficial partial thickness

sensitive to light touch and temp changes

tight hip flexors will affect what part of the gait cycle? - loading response - initial contact - midstance - terminal stance

terminal stance (hip has to go into extension)

Sudden onset of dyspnea, crackles (rales) on auscultation, and S3 heart sound

terminate exercise due to cardiovascular pump dysfunction

Allen test

test that determines the patency of the radial and ulnar arteries by compressing one artery site and observing return of skin color as evidence of patency of the other artery close and open hand then squeeze hand tightly Place thumb and index finger on radial and ulnar arteries to compress and then patient opens hand

lateral rotation lag sign

tests integrity of supraspin and infraspin inability to maintain ER with elbow flexed and shoulder abd 90 deg

ageotropic nystagmus

up beating (away from ground) cupulothiasis: CUP IS UP uninvolved ear has stronger nystagmus

ASIA A, C6 SCI. What are some functional capabilities, equipment, and AD that would be characteristic for this patient?

upper body: independent with adaptive equipment lower body: assistance with adaptive equipment bowel and bladder care: may be able to be independent with adaptive equipment but likely to require assistance/dependent

A patient diagnosed with type 1 diabetes mellitus has a wound on the palmar surface of their first metatarsal head. Which of the following scales is MOST appropriate to classify this wound?

Wagner Classification System

traction to increase intervertebral disc space at L3/4

75-50 deg hip flexion

what is a metatarsal pad used for?

to transfer stress from MT heads to the shaft (reduces plantar pressure)

tongue deviation with hypoglossal nerve injury

tongue atrophy and tongue deviation TOWARDS side of lesion (uvula deviates to the opposite side with vagus nerve)

Egophony results in patient with consolidation vs pneumothorax

transmission of "A" when saying "E" - voice sounds INC with consolidation - voice sounds DEC with pneumothorax/atelectasis

A patient sustained a superficial wound that appears as a moderate abrasion on the anterior surface of their thigh approximately four inches above the superior pole of the patella. Which type of wound dressing would MOST likely be utilized? 1.Calcium alginate dressing 2.Hydrocolloid dressing 3.Hydrogel dressing 4.Transparent film dressing

transparent film dressing

shoe prescription for metarsalgia

transverse metatarsal bar rocker sole

upper crossed syndrome

forward head TIGHT: pec minor, SCM, UT, lev scap WEAK: deep neck flexors, lower trap, serratus

Colles fracture

fracture of the distal radius at the wrist FOOSH on extended wrist causing dorsal displacement of the distal radius

Colle's fracture

fracture of the distal radius at the wrist radius dislocates in dorsal or posterior direction dinner fork deformity

Smith's fracture

fracture of the distal radius with volar displacement due to fall on flexed wrist

open pack radiohumeral

full supination and elbow extension

Documentation of a burn assessment is as follows: The burn is present on the left dorsal forearm. The skin appears red in color with no evidence of conspicuous blanching. There is an absence of hair on the skin, in addition to the absence of sensation to light touch and pin-prick. Which of the following BEST describes the depth of this burn injury?

full thickness burns red in appearance with absence of blanching due to poor distal circulation. the skin is anesthetic and hairs pull out easily

pulse duration and on/off to improve L quad strength 4 weeks post surgery

- high pulse duration (250-300) - 1:5 (10 sec ON , 50 sec OFF)

individual post CABG- when should resistance training begin?

- must have done cardiac rehab for at least 3 weeks - must be at least 8 weeks post CABG

fulcrum test of hip

(+) pain or apprehension may indicate femoral shaft stress fx

medial cord of the brachial plexus

- ulnar nerve - median - medial pectoral

sx of respiratory acidosis

- v fib - visual disturbances - dec DTR - confusion - dizziness

signs of shunt dysfunction

- vomiting - lethargy - irritability - headache - inc seizure frequenc

CRPS 1 vs 2

1: trauma, no specific nerve injury 2: confirmed nerve injury

w/c can cross thresholds of up to ___ inches

0.5 in

A 25 year old female has been suffering from widespread pain with several tender points in the upper body which increases with stress. The Physical Therapist suspects the patient has Fibromyalgia. Which of the following intervention would be LEAST effective? 1) trigger pt massage 2) resistive exercises with t-bands 3) DBE and relaxation 4) mod intensity aerobic exercise 5 days/week

1 myofascial syndrome: radiating pain and trigger points but FIBROMYALGIA HAS NO TRIGGER PTS

Which of the following changes are NOT associated with Postural Scoliosis? 1. Shortened foot supinator muscles on the short side 2. Elongated hip abductor muscles on concave side 3. Short hip adductors muscles on the convex side 4. Shortened foot pronator muscles on the short side

1 on short side: will see overpronation (elongated supinator) will see elongated hip abd on concave side, short hip add on convex, and shortened pronator on short side

herpes simplex 1 vs 2

1 is above the waist 2 is below the waist

A physical therapist completes a respiratory assessment on a patient in an acute care hospital. The examination reveals decreased breath sounds and decreased fremitus. This finding is MOST indicative of which of the following medical conditions? 1.Pleural effusion 2.Pulmonary edema 3.Consolidation 4.Atelectasis

1) pleural effusion: decreased breath sounds and decreased fremitus (or pneumothorax) pulm edema= dec breath sounds, inc fremitus consolidation= dec breath sounds, inc fremitus atelectasis= dec breath sounds, inc fremitus

anatomical pathway for the drainage of LE's into venous circulation

1) popliteal lymph nodes 2) inguinal lymph nodes 3) lateral aortic 4) lymph nodes 5) lumbar lymphatic trunk 6) cisterna chyli at the diaphragm 7) thoracic duct 8) left subclavian

functions of lymphatic system

1) production of antigen specific antibodies and immunological memory for defense against diseases 2) absorption of excess interstitial fluid and proteins from tissues back into circulation 3) absorption of fat from the digestive system and debris from interstitial tissues ** does NOT absorb PROTEINS

KR feedback: most effective summary length for learning 1) simple movements 2) complex movements

1) simple: every 15 reps 2) complex: every 5 reps

why would you see the following deviations? 1) steppage gait 2) leaning away from involved side to avoid buckling 3) excessive lateral trunk lean 4) excessive knee flexion during stance 5) excessive ankle eversion 6) post thrust of trunk at heel strike and anterior tilt of pelvic

1) steppage gait: weak tib ant 2) leaning away from involved side to avoid buckling: weak quads 3) excessive lateral trunk lean: weak glut med (same side) 4) excessive knee flexion during stance: weak gastroc 5) excessive ankle eversion: weak tib ant, tib post (invertors) 6) post thrust of trunk at heel strike and APT: weak glut max

what are the expected resisted iso testing results for the following: 1) complete quad tendon rupture 2) minor lesion of quad mm or tendon 3) compression of femoral nerve 4) disuse atrophy

1) weak and painless 2) strong and painful 3) weak and painless (nervous system d/o) 4) weak and painless

when is crossed extension reflex integrated

1-2 months flexion, add, ext of RLE when noxious stimuli is applied to the LLE fixed in extension

normal Q angle

10-15 degrees 25 deg Q angle will be associated with femoral anteversion, genu valgum, and a laterally displaced tibial tuberosity

Normal prothrombin time

11-15 seconds

normal prothrombin time

12-15 sec may be prolonged with liver damage

pain arc for the AC joint

170-180 deg

A physical therapist grades a patient's ankle strength as Good Plus (4+/5) for the dorsiflexors and Fair Plus (3+/5) for the plantar flexors. Assuming that the patient does not compensate for the muscular impairments, which deviation is the therapist MOST likely to observe during the foot flat to midstance phase of gait? 1.Excessive dorsiflexion 2.Foot maintained in plantar flexion 3.Inadequate toe off 4.Decreased knee flexion

1. weak PF and gastroc would not be able to eccentrically control or oppose the DF moment you would see inadequate toe off with weak PF, but this would occur during later stance, not during foot flat to mid stance would see inc knee flex

normal excursion of diaphragm

1.2-2 inch (3-5 cm) will be dec (0.4-0.8) with COPD

A Physical Therapist is treating a patient status post a lateral meniscus repair and the patient is in their third week of rehabilitation. The Physical Therapist would like the patient to exercise on a stationary bike. Which of the following considerations is MOST important for accurate use of the stationary bike for this patient? 1. Stationary cycle is contraindicated 2. High seat height 3. Low seat height 4. Setting the seat forward

2) high seat height seat raised higher= quads recruited more bc of higher extension angles and greater mechanical advantage (vs more hamstrings recruitment with lower extension angles and lower seat height) avoid setting seat forward bc will inc knee flexion (no knee flex past 45 weeks in first 4 weeks)

A Physical Therapist is assessing a 35 year old patient who suffered a Right transtibial amputation after a motor vehicle accident. The patient is currently using a solid ankle cushion heel and undergoing rehabilitation. During evaluation, the Physical Therapist notices delayed knee flexion during the late stance phase of the gait cycle. Which of the following is the MOST LIKELY prosthetic reason for this gait deviation? 1) socket too far anterior 2) keel of SACH too long 3) DF stop too soft 4) high shoe heel

2) keel of sach too long keel- longitudinal wooden/metal portion on the dorsum of the foot that terminates at a point corresponding to the MTP joints. the posterior portion of the keel is resistant to absorbing shock and allows PF in early stance keel too long: delayed knee flex during late stance

A physical therapist reviews the results of a patient's pulmonary function test prior to examining a patient. The therapist notes that the patient's total lung capacity is significantly increased when compared to established norms. Which of the following medical conditions would MOST likely produce this type of result? 1.Chronic bronchitis 2.Emphysema 3.Spinal cord injury 4.Pulmonary fibrosis

2. emphysema: inc TLC, RV, FRC

standard 18-in chair requires on average how much knee flexion?

79 +/- 14 deg * tie shoe laces while sitting and bringing foot up: 106 +/- 9 deg ascending and descending 5 in step: 82-95 deg

When using hydrotherapy to strengthen the lower limb muscles, how could the Physical Therapist increase resistance to knee movements? 1) knee movement at slow speed towards surface of water 2) knee movements in the direction of the force of buoyancy 3) using paddles for the feet 4) water flowing in the same direction of movement

3 using paddles increases the SA of body moving thru the water and hence the resistance to movement increases moving away from surface would inc resistance rather than moving towards

which results in a higher maximal temp? 1 mHz or 3 mHz

3 MHz results in a higher maximal temp. even tho it delivers lesser depth

biceps femoris tendon repair

3 days: gentle iso strengthening to prevent adhesions of tendon to sheath and to promote alignment * WB restricted 6-8 weeks *8 weeks: passive end-range stretching and concentric exercises with gentle resistance

increasing mm mass: parameters for 68% RM

3 sets x 13 reps

A patient presents with pain in the neck, trapezius and shoulder region and reports no change in intensity of pain. The patient's X-ray shows mild increase in cervical lordosis with no other abnormalities. The patient does not have a fever or chills and he reports an increase in intensity with his pain while breathing in deeply during his yoga class as well as when coughing and laughing. Which of the following is the MOST LIKELY cause of his symptoms? 1) cholecystitis 2) discogenic pain 3) pleurisy 4) PE

3: pleurisy the visceral pleura is insensitive, so pain results from inflammation of the parietal pleura. Since the visceral pleura is innervated by the intercostal nerves, pain will be reported over the site of the pleuritis but can also be referred to the lower chest wall, abdomen neck, upper trap, and shoulder secondary to irritation of the central diaphragmatic pleura

A physical therapist examines a patient with a suspected lesion of the common fibular nerve. Which objective finding would be the MOST useful to rule out the possibility of a sciatic nerve lesion? 1.Inability to actively dorsiflex the foot 2.Preservation of the Achilles reflex 3.Presence of a steppage gait 4.Weakness of the quadriceps muscle

2. absence of achilles= tibial or sciatic injury

A Physical Therapist and Occupational Therapist working in a skilled Nursing facility are discussing wheelchair prescription for a patient who incurred a C6 (ASIA B) Spinal cord lesion. What is the best wheelchair prescription for this patient? 1. Manual wheelchair with propulsion aids 2. Manual wheelchair with friction surface hand rims 3. Manual wheelchair with standard hand rims for increased propulsion 4. Manual wheelchair with standard hand rims

2. manual wc with friction surface hand rims C5 SCI: manual wc with propulsion aid (bc no tenodesis or grip strength) C6 SCI: manual wc with friction surface handrims (can use tenodesis to propel, but grip strength is weak so friction surface is required) C8-T1 SCI: manual wc with standard hand rims (bc will have improved grip strength 2/2 preserved finger flexors and small mm of the hand)

normal radial deviation

20-30 deg

A physical therapist administers the Mini-Mental State Examination to a patient recently admitted to the hospital. What is the MINIMUM patient score necessary in order to avoid being classified as possessing a cognitive impairment? 1.18 2.24 3.30 4.34

24/30

initial starting lumbar traction

25% BW (disc protrusion, spasm, elongation)

A physical therapist examines a patient with a suspected injury to the thoracodorsal nerve. Which objective finding would be the MOST consistent with this injury? 1.Shoulder medial rotation weakness 2.Shoulder extension weakness 3.Paralysis of the rhomboids 4.Paralysis of the diaphragm

2: lats (shoulder ext, add, IR) rhomboids are dorsal scap

A patient, who suffered a RIGHT hemisphere stroke 3 weeks ago, resides in an inpatient care facility, and would present with which of the following deficits? 1. Highly distractible, ideomotor apraxia, difficulty with abstract reasoning, right-sided sensory loss 2. Perseveration, memory impairments related to language, disorganized, global aphasia, highly distractible 3. Difficulty with perceiving emotions, rigidity in thought, impulsivity, agnosia, visuospatial disorders, body image/schema disturbances 4. Left-sided neglect, non-fluent aphasia, perseveration, ideational planning, rigidity of thought, fluctuations in performance

3

A 25 year old female was admitted to the emergency department secondary to a deep-partial thickness burn. According to The Rule of Nines, her total surface area burn, (TSAB) was 40.5%. What areas are included to add up to the patient's total surface area burn? 1. Anterior surface of head, anterior & posterior surfaces of the trunk, and the entire anterior & posterior surfaces of the left upper extremity 2. Posterior surface of the head, the entire right leg, and the entire right arm, anterior & posterior 3. Anterior head, anterior trunk, anterior surface of both the right & left lower extremetries 4. Anterior surface of both the right & left upper extremities, anterior and posterior surfaces of the trunk

3) Anterior head, anterior trunk, anterior surface of both the right & left lower extremetries rule of nines: anterior head: 4.5 posterior head: 4.5 arms: total 9% (ant/post 4.5%) legs: total 18% (ant/post 9%) trunk total: 36% anterior upper trunk: 9% groin: 1%

A Physical Therapist observes PVCs in a patient's EKG strip during a regular exercise session. What is a good indication for termination of exercise and activating emergency response team? 1) 6 PVC in 1 min 2) multifocal PVC 3) 4 PVC in a row 4) PVC inc with activity

3+ PVC in a row= ventricular tachycardia Single PVCs observed in one minute that are less than 7 total don't compromise cardiac output and therefore are not considered an emergency. Physical activity can be done if proper hemodynamic responses are present. Multifocal PVCs suggest an irritable ventricle. Stopping exercise and referring the patient to their MD would be recommended if multifocal PVCs are observed, however it is not considered an emergency. If PVCs increase with activity, The Physical Therapist should stop the activity however it is not considered an emergency.

1) A patient reports falling on the shoulder and experiencing superior shoulder pain. The pain is produced by passive shoulder horizontal ADD. Which ligaments is MOST likely injured? a. Coracohumeral b. Coracoclavicular c. Inferior glenohumeral d. Superior glenohumeral

B

normal serum albumin

3.5-4.8 see less with liver damage

normal pelvic angle

30 deg the PSIS should be slightly higher than the ASIS dec pelvic angle leads to PPT

hip open packed and resting position

30 deg flexion resting in 30 deg flex and abd

ramps longer than ___ ft in length will need to change direction and may require a landing area

30 ft

minimum door width for wheelchair

32 in

ADA minimum doorway width

32 inch

ADA min doorway width

32-34 in

A patient with complete paraplegia discusses accessibility issues with an employer in preparation for return to work. The patient is concerned about the ability to navigate a wheelchair in certain areas of the building. What is the MINIMUM space required to turn 180 degrees in a standard wheelchair?

60 inc

A physical therapist treats a patient diagnosed with plantar fasciitis. During the treatment session, the therapist attempts to strengthen the muscles that support the medial longitudinal arch. Which of the following muscles would be MOST important to emphasize in the strengthening program? 1.Gastrocnemius, soleus, and plantaris 2.Fibularis (peroneus) longus and brevis 3.Tibialis anterior and extensor hallucis longus 4.Tibialis posterior and flexor digitorum longus PreviousNextMark

4

A Physical Therapist is planning goals for a patient admitted to the hospital following a cervical hyperextension injury. Which of the following goals would be the most challenging to achieve for this patient? 1) walking with cane 2) mini squats against wall 3) standing, feet together 4) building a block of legos

4 cervical hyperextension injury= central cord UE > LE residual hand weakness and issues with dexterity

A physical therapist treats a patient for a pressure injury on the ischial tuberosity and is concerned about the risk of infection caused by the patient's weakened immune system. Which of the following patients would have the LOWEST risk for infection? 1.A 55-year-old patient with malabsorption syndrome 2.A 50-year-old patient with acquired immune deficiency syndrome 3.A 60-year-old patient receiving monthly cortisone injections 4.A 65-year-old patient with iron-deficiency anemia

4 cortisone can lead to suppression of immune system as a side effect

increasing mm mass: parameters for 70% RM

4 sets x 11 reps will promote mm hypertrophy

initial starting cervical traction

7-10% BW

open pack ulnohumeral

70 deg elbow flexion, 10 deg supination

humeroulnar open pack

70 deg flex

A physical therapist treats a patient with several injuries impacting the upper extremity including mallet finger. The therapist notes that the affected finger is immobilized using a static splint. Which position of the finger would be the MOST essential when splinting? 1.5 degrees of flexion at the distal interphalangeal joint 2.5 degrees of flexion at the proximal interphalangeal joint 3.5 degrees of hyperextension at the distal interphalangeal joint 4.5 degrees of hyperextension at the proximal interphalangeal joint

5 deg of hyperextension at the DIP mallet finger= injury of the extensor digitorum tendon at DIP, occurs with forced finger flexion at DIP splint for 6-8 weeks in 5 deg hyperext at DIP

minimum door opening space for a door that swings towards the patient

5 ft x 5 ft

increasing mm mass: parameters for 81% of RM

5 sets x 7-8 reps

normal diastolic PAP

5-15 3: pt is hemodynamically unstable, percussion contraindicated

when is moro integrated

5-6 months (sudden drop backward from sitting will cause extension and abd of UE, hand opening, crying followed by flexion and ADD of UE across chest)

how much ROM increase can you expect after 1 round of serial casting?

5-7 deg

traction to increase intervertebral disc space at L5/S1

45-60 deg hip flexion

pain arc for the GH joint

45/60 to 120 deg

A 15 year-old gymnast walks into the Physical Therapy clinic complaining of constant low back pain. Patient presents with a hyperlordotic curve, retracted bilateral shoulder blades and displaced plumb line. Post examination, the Physical Therapist documents several observations regarding pelvic dysfunction, and pathological standing posture. The patient's pelvis is MOST likely positioned in which of the following? 1. Counternutation with posterior pelvic tilt 2. Nutation with a posterior pelvic tilt 3. Nutation with and anterior pelvic tilt 4. Counternutation with an anterior pelvic tilt

4: counternutation with APT counternutation: anterior rotation of the ilium on the sacrum or backward motion of the base of the sacrum out of the pelvis

A physical therapist assesses the end-feel of plantar flexion range of motion. The therapist classifies the end-feel as firm. Which of the following structures does NOT contribute to the firm end-feel? 1.Tension in the anterior joint capsule 2.Tension in the tibialis anterior 3.Tension in the anterior talofibular ligament 4.Tension in the calcaneofibular ligament

4: tension in CFL is associated with the normal end feel of DF

contrast bath ratio and temps

4:1 ratio of heat to cold - 4 mins of heat 100-111 deg - 1 min of cool 55-65

open pack radioulnar

50 deg elbow flex 35 deg supination

traction % BW for joint distraction

50 lbs or 50% BW

Where should a PT place electrode for biofeedback to facilitate hook grasp? a. Proximal anteromedial b. Proximal posterolateral c. Distal anteromedial d. Distal posterolateral

A hook grasp: finger flexors

A 45 year old male patient with below knee amputation of the left lower extremity arrived at an outpatient clinic having complaints of increased discomfort while wearing the prosthesis. Temporary redness at which of the following areas can be considered a normal sign for this patient? Select one: a. Tibial shaft b. Tibial crest c. Fibular head d. Tibial condyle

A other pressure tolerant: belly of gastroc, patellar lig, prox medial tibia, tibia and fibular shaft

hip pointer

A direct blow to the iliac crest (at, or near, the ASIS) may produce a contusion. Typically, there can be pain over the ASIS, pain with passive hip extension, and pain with resisted hip flexion, external rotation, and abduction. Activities such as laughing and coughing may increase the pain as well.

which fibers does low intensity tens stimulate?

A fibers (large) and substantia gelatinosa closes gate and blocks pain signals being sent by t-cells)

gold standard for AC injuries

A-P B/L radiograph with weights

what drains into thoracic duct?

L UE, L half of neck and head, L upper trunk, and everything below diaphragm

1) Which result is consistent with T1 SCI Asia B? a. Intact sensation on apex of axilla, active movement of elbow flexors against gravity, no anal sensation b. Intact sensation medial antecubital fossa, palpable mm activity of 5th digit, (+) anal sensation c. Intact sensation dorsal surface of the proximal phalanx of the 3rd digit, palpable mm activity of wrist extensors, absence of anal sensation d. Intact sensation dorsal surface of proximal phalanx of thumb, active movement of wrist extensors against gravity, presence of anal sensation

B

A physical therapist treats a patient diagnosed with osteonecrosis of the femoral condyle. Which patient profile is the MOST typical with this medical condition? 1.A 42-year-old female with osteonecrosis of the lateral femoral condyle 2.A 64-year-old female with osteonecrosis of the medial femoral condyle 3.A 46-year-old male with osteonecrosis of the medial femoral condyle 4.A 68-year-old male with osteonecrosis of the lateral femoral condyle

B

A 32-year-old patient has been referred for a L4 nerve root lesion. Which of the following special tests would MOST LIKELY be a positive test? a. Flamingo test b. Gaenslen's test c. Thigh thrust test d. Gillet's test

B (+) Gaenslen: SIJ, hip pathology, L4 nerve root lesion thrust and flamingo are used for SIJ gillet is used for ipsilateral posterior innominate rotation

A 59-year-old female patient arrived at an outpatient clinic with complaints of discomfort in the wrist/hand. Upon examination, the physical therapist observed deformity of extension of MCP and distal interphalangeal with flexion of PIP. Which of the following is the MOST likely cause of this presentation? Select one: a. Rupture of central tendon. b. Banding on palm and digit flexion contractures c. Contracture of intrinsic muscles along with dorsal subluxation of lateral extensor tendons. d. Rupture of the extensor tendon at the distal phalanx

A: The patient presents with boutonniere deformity that is an extension of MCP and distal interphalangeal with flexion of PIP which is caused by rupture of the central tendon. A contracture of intrinsic muscles along with dorsal subluxation of lateral extensor tendons will cause swan neck deformity. A rupture of the extensor finger at distal phalanx will lead to a trigger finger. A banding on palm and digit flexion contractures is seen with dupuytren contracture.

A physical therapist observes thenar atrophy when examining a patient's hand. In the absence of other relevant findings, this could BEST be explained by which of the following conditions? 1.C8 nerve root lesion 2.Paralysis of the interossei 3.Radial nerve lesion 4.Ulnar nerve lesion

C8/T1 nerve lesion, median nerve

how do you tension the inferior GH joint?

ABD, ext, ER

A 48-year-old female status post frequent syncopal episodes has been diagnosed with adrenal insufficiency. Which of the following is NOT expected to be observed in this patient? 1) fatigue and mm weakness 2) hypotension 3) hyponatremia 4) hyperglycemia

Adrenal insufficiency= addison's - dec cortisol and aldosterone --> hypoglycemia, hypotension, syncope (aldosterone= sodium retention) hyponatremia and hyperkalemia -- can cause fatigue and mm weakness

A 57-year-old male recently sustained a fall at home. EMS arrived at the scene, assessed vitals, and immediately transferred him to a nearby hospital. The patient has a past medical history of portal HTN, enlarged spleen, liver cirrhosis. After hours of testing, the patient's blood work revealed a platelet count of 18,850. The physical therapist is reviewing his medical record and working on the plan of care. What would be the MOST APPROPRIATE intervention for this patient? Select one: a. Initiating warm-up exercise with the use of a stationary bike. b. Performing AROM exercises in a gravity eliminated plane motion while focusing on ADLs. c. Having patient ambulate to his tolerance d. No treatment, the patient would need to undergo transfusion

B <10,000: transfusion indicated platelets <20,000: ADL, AAROM, AROM with gravity eliminated 20-30,000: light AROM, no PROM, walking 30-50,000: AROM, submax isometrics, walking, stationary bike, no prolonged stretches

S1 heart sound is closing of? S2 heart sound is closing of?

S1: closing of AV valves (mitral and tricuspid) S2: closing of semilunar valves (aortic and pulmonary) * systole occurs between S1 and S2 * S3 can be heard in early diastole and S4 can be heard in late diastole

ulnar nerve pathology at hand

Bishop/benediction hand wasting of hypothenar eminence

which heart sound is closure of aortic and pulmonary valves

S2

A 49 year old female patient with bilateral knee osteoarthritis (OA) was referred to physical therapy. While reviewing the patient's medical records, the therapist comes across her recent radiographs. Which of the following radiographic findings would be LEAST likely found with this patient? Select one: a. Sclerotic subchondral bone b. Cyst-like radiolucent lesions c. Accentuation of remaining trabeculae d. Osteophyte formation at joint margins

C accentuation of remaining trabeculae is a common feature of osteoporosis not OA

A 55-year old male was admitted to a local hospital for head trauma after a skiing accident. Upon examination, the patient seemed to be in a heightened state of activity as his behavior was erratic, and non-purposeful, incoherent, and confabulation were present. Also, his verbalization appeared to be inappropriate intermittently. This patient would MOST likely be classified to be at which stage per LOCF? Select one: a. Level III b. Level V c. Level VI d. Level IV

C level 4

knee flexion is supplied by which dermatome/myotome

S2 - sensation to buttock, back of thigh, lower leg

Which heart sound in a 70-year-old MOST likely indicates ventricular failure

S3

Trochanteric Bursitis

Lateral hip pain near the greater trochanter, aggravated by lying on the affected side - pain will passive hip ADD (stretching ABD) and resisted ER/ABD/EXT

medial antebrachial cutaneous n

C6-T1

insidious onset of pain and paresthesias in medial forearm and hand - C6 radic - C8 radic - medial nerve entrapment - radial nerve entrapment

C8 radic

Klumpke

C8-T1 weakness of finger extensors and intrinsic hand mm that perform MCP adduction

mm of respiration with their spinal cord levels

C1-2: SCM, upper trap, cervical extensors C3-4: partial diaphragm, scalene, lev scap C5-8: diaphragm, pec major, pec minor, serratus, rhomboids, lats T1-5: some intercostals, erector spinae T6-10: intercostals, abdominals

Myotomes C1-C4

C1-C2: neck flexion (rectus lateralis, rectus capitis anterior, longus capitis, longus colli, longus cervicis) C3: lateral cervical flexion (traps, scalenus medius) C4: shoulder elevation (diaphragm, traps, lev scap, anterior and mid scalenes)

Erbs palsy

C5-6 shoulder is held in extension, IR and ADD, and pronation reaching with IR and pronation will reinforce resting grasp intact, will see weakness of ABD, flexors, rotators, supinator

chronic autoimmune thyroiditis

Chronic autoimmune thyroiditis is associated with hypothyroidism. The patient with hypothyroidism presents with emotional changes, low heart rate, and is intolerant to cold

Wagner Classification System

Classification system for neuropathic ulcers (1-3 are open) 0: closed wound with hyperkeratosis and bone deformation healed ulcers or preulcerative lesions 1: superficial ulcer without subcutaneous tissue involvement 2: penetration thru subcutaneous tissue, which may expose bone, tendon, ligament, or joint capsule grade 3: osteitis, abscess, or osteomyelitis 4: gangrene

A 64-year-old male arrived at an outpatient physical therapy clinic with chief complaints of gradual onset of low back pain mainly on the right side. Upon questioning further, the patient reported having increased pain in standing. His symptoms decreased during both sitting and while retrieving objects from the floor. The physical therapist performed a straight leg raise test which revealed a negative result. The therapist concluded that the patient MOST likely has which of the following diagnosis? a. Spinal stenosis b. Spondylolisthesis c. Herniated Nucleus Pulposus d. Osteoarthritis

D With osteoarthritis, pain increases in standing and decreases both during sitting and while retrieving or picking up objects from the floor. SLR will be negative with OA. A negative SLR would rule out spinal stenosis and a herniated nucleus pulposus A patient with spondylolisthesis would experience increased pain during standing and while picking up objects from the floor (forward bending) but decreased pain with sitting and the SLR will be negative.

A patient came to an outpatient physical therapy clinic post injury to the left superior gluteal nerve. Which of the following gait deviations is MOST likely to be seen during the evaluation of this patient? Select one: a. Backward trunk lean during loading response b. Trunk lean to the right during left stance phase c. Posterior tilting of the pelvis during initial swing d. Trunk lean to the left during left stance phase

D superior gluteal: glut med, glut min, TFL L glut med weakness: R hip drop trunk lean towards weak side

1) Nerve damage associated with a mid-shaft fracture of the humerus is most likely to result in weakness of: a. Dorsal interossei b. Flexor digitorum superficialis c. Pronator quadratus d. ECRL

D (radial n)

landmarks for measuring shoulder IR

Fulcrum: olecranon process movable arm: ulna, other perpendicular to the floor

anterior tarsal syndrome

deep aching pain in the medial and dorsal aspect of foot burning around nail of great toe pins and needles that are exacerbated with PF

noxious TENS

highest tolerated stimulus pulse frequency can be high or low LONG pulse duration (250 micro sec to 1 sec) 30-60 sec for each trigger point

calcium alginate dressings

highly absorbent good for granular and draining wounds

collagen dressing

highly absorbent good for granular and draining wounds

lateral glide of femur on acetabulum improves what motion:

hip ADD and IR

Gillet's test

Patient is standing. Place your thumb of your hand under PSIS of limb to be tested and place your other thumb on center of sacrum at same level as thumb under PSIS. Ask patient to FLEX hip and knee of limb being tested as if bringing their knee to chest. Assess movement of PSIS via comparison of positions of your thumbs. Make sure your eyes are level with your thumbs. PSIS should move in an INFERIOR direction (+) TEST: no identified movement of PSIS as compared to sacrum

pt has L2 radiculopathy with motor weakness- what will you see

hip ER during swing (ER will be used to facilitate hip flexion in swing by using the ADD as flexors)

Apley's compression test

Patient prone with knee flexed to 90 degrees, the doctor anchors the thigh of the patient and grasps proximal to foot and applies downward pressure and rotates leg internally and externally. Heel points toward side being tested. Positive sign is pain in knee indicating meniscal tear.

(+) ortolani

hip dysplasia will feel a "clunk" as the dislocated femoral head reduces into acetabulum

hip: long axis distraction with ant glide promotes:

hip extension

A Physical Therapist is assessing the conjugate gaze of a patient. On stimulating the right pontine gaze center, the patient's left eye moves to the right, but the right eye doesn't move along with it. Which of the following cranial nerves may be affected?

R abducens nerve stimulation of the R pontine gaze center stimulates the.L oculomotor and R abducens, causing both eyes to move to the R

sciatic and tibial nerve bias

hip flex and ADD DF

D1 flexion LE

hip flex, ADD, ER knee extension ankle DF and inversion

how to bias common peroneal nerve in SLR

hip flexion and IR, knee ext, PF and inversion

R hemisphere lesion vs L hemisphere lesion

R hemi: L sensory loss - difficulty in spatial perceptual tasks and in grasping the whole idea of a task or activity - quick, impulsive, overestimate abilities, unaware of deficits L hemi: R sensory loss - cautious, anxious, disorganized - difficulties in communication and in processing info

supine to long sitting: R leg gets shorter. what does this mean

R innominate is anteriorly rotated

a positive sulcus sign is indicative of joint instability in which direction?

inferior jt instability - RTC strengthening will inc the stability of the GH jt and the ability of the humeral head to stay in the glenoid fossa

Raimiste's phenomenon

involved LE will abduct/adduct with applied resistance to the uninvolved LE in the same direction

A physical therapist would like to incorporate low rate (acupuncture-like) transcutaneous electrical nerve stimulation (TENS) in a therapy session. Which of the following parameters BEST correlates to low rate TENS? Select one: a. Pulse frequency: 5 pps; pulse duration: 50 microseconds b. Pulse frequency: 100 pps; pulse duration: 70 microseconds c. Pulse frequency: 150 pps; pulse duration: 300 microseconds d. Pulse frequency: 7 pps; pulse duration: 250 microseconds

D Low rate (acupuncture-like) TENS typically has a pulse frequency range of 2-10 pps, and a pulse duration range of 200-300 microseconds.

To prevent maximal compressive force being placed on the patella, a therapist should avoid placing a patient in what position? A. Prone with the knee flexed to 30 degrees B. Seated with the knee flexed to 90 degrees C. Supine with the hip and knee flexed to 110 degrees D. Prone with the knee flexed to 110 degrees and the hip in slight extension

D Positioning the patient in prone places the hip into an extended position. From this position, increasing knee flexion to 110 degrees results in passive insufficiency of the quadriceps, which creates higher compression at the patella. Since the quadriceps group is aligned anatomically with the shaft of the femur and not with the mechanical axis of the lower extremity, any quadriceps muscle contraction (regardless of knee flexion angle) results in compressive forces acting on the patellofemoral joint

which motion do you avoid 1 week after UCL reconstruction at the elbow

no shoulder ER - ER creates valgus forces on the elbow, which stresses the UCL * active wrist motion and gripping activities are encouraged

A male patient has been referred for rehabilitation, 4 weeks after a sudden onset of weakness. His postural control is impaired, and there are no signs of involuntary movements, hyperreflexia, flexor synergy, rigidity, or spasticity. He walks with a wide-based, unsteady gait. Based on the clinical findings, which of the following is the physical therapist LEAST likely to notice upon further examination? a. Overestimation of the distance required to accomplish a movement b. Underestimation of the distance required to accomplish a movement c. Rhythmic oscillations of the head d. Series of brief catches as a limb is passively moved

D cerebellar pathology: dysmetria, titubation (rhythmic oscillations of the head), normal or decreased reflexes, normal or dec tone, ataxic gait pattern with wide BOS, nystagmus, intention tremor, dysarthria brief catches: cogwheel rigidity (more common with a basal ganglia pathology)

A physical therapist is auscultating and listening to various sounds of the heart. The echocardiogram report revealed left ventricular hypertrophy with EF 25-30%. Which of the following heart sounds would MOST likely correlate to this scenario? Select one: a. S4 b. S2 c. S1 d. S3

S3: CHF S4: TN, left ventricular hypertrophy, increased left ventricular end-diastolic pressure, pulmonary hypertension, and pulmonary stenosis.

What conditions are the abnormal heart sounds S3 and S4 associated with, respectively?

S3= CHF S4= MI or HTN

A 24-year-old male soccer player presents to the clinic with chief complaints of constant pain in the right thigh (intensity 5/10 on VAS) since the last 5 months. His falls during practice and intensity of exercises do not seem to make a huge difference in the pain levels. The pain is often worse at night (9/10 on VAS) and the thigh is always warm and tender to touch. He has lost weight recently. He tries to do some stretches on his own to relieve the pain, but nothing really makes a difference except for taking an aspirin twice a day (0-1/10 on VAS). Which of the following is the MOST LIKELY diagnosis for this patient? a. Fracture of the femur b. Chondrosarcoma c. Partial rupture of the rectus femoris muscle d. Osteoid osteoma

D

A physical therapist performs a muscle length test for the long head of the triceps. Which of the following findings is the MOST consistent with shortening of this muscle? 1.Limitation of elbow flexion with the shoulder maintained at the end range of extension 2.Limitation of elbow extension with the shoulder maintained at the end range of extension 3.Limitation of elbow extension with the shoulder maintained at the end range of flexion 4.Limitation of elbow flexion with the shoulder maintained at the end range of flexion

D

A 62 year old male patient with a long-standing history of alcohol abuse reports upper abdominal pain that radiates to his back. He reports that his pain increases with walking and even just laying down; his pain decreases when he sits and leans forward. He explains how he has been feeling nauseous lately, and experienced some episodes of vomiting over the last couple days. He reports a fever and a feeling of weakness. A Grey Turner's sign and jaundice is noted on the patient. Which of the following is MOST likely expected to be present with the patient? Select one: a. Blumberg's sign b. Pain at McBurney's point c. A positive pinch-an-inch test d. A Cullen sign

D: cullen sign, which is a bluish discoloration at the periumbical area associated with acute pancreatitis grey turner sign: reddish-brown coloring of the flank (hemorrhagic pancreatitis)

Joe presents to the clinic with a large burn on his forearm. When the therapist tries to touch it, he does not complain of pain on pressure or light touch. Which of the following is the MOST LIKELY presentation of this wound? a. Mixed red, waxy white appearance with blanching and slow capillary refill b. Erythematous pink with no blisters c. Erythematous with blanching and quick capillary refill d. White or tan with no blanching

D: full thickness are usually white or tan with no blanching, loss of pain felt bc the pain fibers are all cut superficial partial thickness: erythematous, blanch, quick capillary refill deep partial thickness: broken blisters, sensitive to pressure, insensitive to light touch, blanch with slow capillary refill epidermal burns: erythematous with no blisters

A physical therapist works with a patient who has right hemiparesis post stroke. As the patient lies in the supine position on the mat, the therapist applies resistance to right elbow flexion and notes mass flexion of the right lower extremity as the resistance is applied. The therapist should document this response as which of the following associated reactions? 1.Raimiste's phenomenon 2.Souques' phenomenon 3.Coordination synkinesis 4.Homolateral synkinesis

D: homolateral synkinesis is often associated with hemiplegia, where there is mutual dependency between the involved upper and lower extremities

closed chain knee flexion is associated with increased

DF

how to stress tibial nerve

DF + eversion + toe ext

tibial nerve bias

DF, eversion, toe extension

GH capsular pattern

ER > ABD > IR

Patrick's test

FABER

common MOI for scaphoid fx

FOOSH with the wrist extended and radially deviated

GOLD stage 3 severe

GO BY FEV 1 FEV/FVC always <70 FEV 1 30-50%

GOLD stage 4 (very severe)

GO BY FEV 1 FEV/FVC always <70 FEV 1 <30%

GOLD stage 2 (moderate)

GO BY FEV 1 FEV/FVC always <70 FEV 1 between 50% and 80%

GOLD classification for COPD stage 1 (mild)

GO BY FEV 1 FEV/FVC always <70 FEV 1>80%

Which of following kinetic chain abnormalities is MOST commonly associated with foot supination?

Genu varum results in tibial external rotation, leading to a forefoot position of supination. ** APT, femoral anteversion, genu recurvatum result in pronated foot position

strengthening: higher ratio or lower ratio?

HIGH 1:3 or 1:4 to allow adequate resting times

biofeedback for strengthening (WEAK mm) - sensitivity - electrode placement

HIGH sensitivity WIDELY spaced (initially for more recruitment)

hip: long axis distraction with inferior glide promotes:

HIP FLEXION

A physical therapist administers a submaximal exercise test to a patient in a cardiac rehabilitation program. The protocol requires the patient to ride a cycle ergometer for a predetermined amount of time using progressive workloads. In order to predict the patient's maximum oxygen uptake, it is necessary to determine the relationship between which of the following parameters? 1.Heart rate and rate of perceived exertion 2.Heart rate and workload 3.Blood pressure and rate of perceived exertion 4.Blood pressure and workload PreviousNextMark

HR and workload

side effects of digoxin

HYPOKALEMIA - arrhythmia, palpitation, fatigue, weakness (due to inc strength of contraction) * digoxin tox: GI disturbances, loss of appetite, confusion, blurred vision, irregular pulse

parietal lobe lesion, non-dominant (R)

Hemiparesis and perceptual deficits/neglect

A 54 year-old male patient is warming up on the treadmill and is being monitored with an EKG. After 30 seconds, the patient has multiple inverted T waves. What medication should the primary care physician prescribe and what is a possible contributing factor for the inverted T wave? 1. Hypokalemia and beta blocker 2. Hypokalemia and Hydrochlorothiazide 3. Hyperkalemia and Hydrochlorothiazide 4. Hyperkalemia and beta blocker

Hypokalemia will cause inverted T waves while hyperkalemia will cause the T waves to peak. A side effect of hydrochlorothiazide is hypokalemia and therefore should not be prescribed in a patient that already has inverted T waves. The EKG is displaying an abnormal heart rhythm and therefore the patient could be prescribed a beta blocker to help regulate.

4 (-) charged ions for ionto

I SAD - iodine - salicylate - acetate - dexamethasone

medial medullary syndrome

IPSILATERAL tongue paralysis (CN 12) CONTRALATERAL paralysis of UE/LE due to corticospinal lesion

which way does talocrural jt rotate with pronation

IR

Trigger Finger

Inflammation and thickening of the tendons of the finger makes it difficult to flex or extend the finger, may become stuck and then snap into position

A physical therapist determines that a patient has 0-135 degrees of passive knee flexion and 0-120 degrees of active knee flexion. Which of the following tests would be the MOST appropriate to help determine the reason for the difference in the range of motion values? 1.Passive joint motion testing 2.Special tests isolating flexibility 3.Manual muscle testing 4.Diagnostic imaging

MMT- mm weakness normally presents with normal PROM and decreased AROm (special tests isolating flexibility are commonly used with a suspected musculotendinous lesion)

HIGH points on oswestry

MORE disability

gold standard for SLAP

MRA will have (+) clunk

which electrode do you use with infected wounds?

NEGATIVE bc infections are bad positive electrode used to promote healing of wounds without inflammation

should you use donut-cushions for prevention of ulcers when seated in wheelchair?

NO rim of cushion creates pressure, which occludes capillaries and decreases blood flow

which motion should you avoid 1 week after superior labrum epair

NO SUPINATION bc it is performed by active contraction of biceps and supinator no resisted contraction of biceps for 3-6 weeks - pronation, retraction, protraction okay

Signs of digoxin toxicity in patients with CHF :

Nausea, vomiting, headache, dizziness, confusion, abdominal pain, delirium, vision disturbance.

heberdens node seen with:

OA heberden are osteophytes seen at the DIP

lumbar quadrant test

OP in extension as patient is side flexed and rotated to 1 side= causes max narrowing of the intervertebral foramen and stress at the facet joint on the side of where rotation occurs

Bouchard's nodes

Osteoarthritis (PIP swelling 2° to osteophytes) seen at PIP

how to stress common peroneal nerve

PF + inversion

LE PNF for gait and LE weakness

PNF D1 flexion flexion, ADD, ER, DF with inversion

dec pelvic angle leads to:

PPT decreased hamstring mm length

A physical therapist assesses an infant by performing a pull-to-sit maneuver. During the maneuver, the therapist observes that the infant has learned to lead with the head as soon as the stimulus of being pulled to sitting occurs. Which muscle group would be the LEAST instrumental in performing the described activity? 1.Abdominals 2.Cervical flexors 3.Hip flexors 4.Scapular stabilizers PreviousNextMark

PT gently pulled UE at wrist to sit child up inadequate head control: will fall backward pull to sit: abdominals, cervical flexors, hip flexors

effect of digoxin on QRS complex, QT interval, and PR

QT shortened PR prolonged

what causes the most stress on the vertebral artery

ROTATION-ext-traction * flexion is least stressful

at what RR should you terminate opioids

RR <10

urethritis is commonly found in patients with what systemic condition?

Reiter

types of bandages for lymphedema

SHORT STRETCH low resting, high working pressure

Patient findings: L shoulder pain after sliding into homebase - scapular dyskinesis - TTP coronoid process - malalignment of levator scap - moderate protraction and elevation - shoulder flex 60-150 (ant deltoid) & abd (mid delt) 3/5 MMT - L shoulder pain with 10 deg of shoulder ABD with suspected decreased subacromial space

SICK syndrome (scapular dyskinesis)

A physical therapist is trying to figure out which is the best test for a diagnosis of a PCL tear in 20 to 40 year old patients. Which of the following will be the MOST APPROPRIATE study to answer the question? a. Case-series b. Systematic review of randomized control trials c. Randomized control trial d. Systematic review of cross-sectional studies

SR of cross-sectional studies is best suited for diagnostic studies RCT's= therapeutic interventions D

HIGH frequency, LOW pulse duration

conventional

4 types of TENS

conventional acupuncture brief intense noxious

Cervical Myelopathy

nontraumatic neck and shoulder pain, dec arm dexterity, paresthesia in UE, hyperreflexia, urinary retention with overflow UI

Which of following adrenal hormones helps in regulating blood pressure, as well as fluid balance by maintaining the electrolyte balance?

aldosterone: mineralocorticoid secreted by the adrenal cortex. aldosterone helps in maintaining the electrolyte imbalance by sodium conservation & assists to maintain the homeostatic balance

which dressing can be used over an infected wound?

alginate dressing

presbyopia

normal loss of lens elasticity farsightedness

normal ammonia values: what would you see with liver damage?

normal: <75 see more with liver damage bc the liver is unable to break down ammonia

how to bias the femoral nerve during the slump test

sidelying, cervical flexion, thoracic and lumbar flexion (slump), 20 deg hip flexion, knee flexion, ankle PF

A 13 year old female presents with anterior groin pain which started two weeks ago after turning over in bed. The patient is able to walk, but with difficulty. The Physical Therapist noticed limited ROM at the hip and weakness of the hip musculature. Which hip ROM would the Physical Therapist expect to be limited?

hip flexion, IR, ABD this is likely SCFE

sural nerve SLR

hip flexion, knee ext, DF + INV

how to stress sural nerve

hip flexion, knee extension ankle DF + inv

how do you instruct a patient with a LEFT hip fracture to use a large base quad cane?

hold cane on the R with the longer legs positioned AWAY from the patient

horizontal nystagmus

horizontal canals

Which of the following medical conditions or meds does NOT cause secondary osteoporosis? - chronic renal failure - GI diseases - hypothyroidism - glucorticoids

hypothyroid causes inc in bone density

MOI for syndesmotic ankle sprain

ankle planted in DF with ER of lower leg tests: - fibular translation test - lateral rotation test - distal tibfib compression test

late manifestations of CF

anorexia clubbing diarrhea

tethered cord

spasticity inc tone inc scoliosis weakened leg mm

Reverse TSH protocol stage 1 (0-6 weeks)

arm in abduction splijnt 24 hours for between 3-6 weeks NO GH EXTENSION OR IR PAST NEUTRAL 0-20 degrees ER, 90-120 deg elevation in scapular plane can do light NWB isometrics of shoulder stabilizers and deltoid

excessively firm heel wedge

block DF and will cause increased knee flexion in early stance

fibromyalgia vs myofascial syndrome

fibromyalgia: chronic, widespread pain, >3 months - 11/18 tender points - may develop after trauma with fluctuating sx aggravated by stress - NO referred pain, NO tight bands - constant fatigue, waking up unrefreshed myofascial pain: no fatigue related complaint - trigger points with referred pain patterns & tight bands of mm are common

excessive supination compensates for?

forefoot varus

A physical therapist would like to utilize vapocoolant spray to various trigger points and their related tight muscles. Which of the following is the MOST appropriate technique when using vapocoolant spray?

The can is positioned 15 inches away from the skin and is sprayed at a 30 degree angle to the skin, is correct. Typically for vapocoolant spray: there is an application of 2-5 parallel sweeps of the spray, 0.5-1 inch apart, at a speed of 4 inches per second (along the muscle fiber direction), the can held upright 12-18 inches away from the skin, and at an angle of ~30 degrees to the skin. There can be various trade names that can contain the same chemical components, but their delivery systems and Food and Drug Administration (FDA)-approved indications may differ.

superior patellar glide improves:

knee extension

Venous filling time test

The extremity is elevated and then lowered into a dependent position. The time it takes for the veins on top of the foot to refill is recorded. Normal filling time is 15 seconds. >15 sec: arterial disease <15 sec: venous disease

AIIS avulsion fx

knee extensors and hip flexors attach here: would not be associated with buttock pain or pain with resisted knee flex

inferior patellar glide improves:

knee flexion

how to stretch lev scap

The head is forward bent and rotated to the opposite side of tightness. The arm on the side of tightness is abducted and placed behind the head to help stabilize the head. To increase the stretch, the therapist may press down against the superior angle of the scapula.

stages of lymphedema

Stage 0: non-clinical Stage 1: reversible Stage 2: non-reversible Stage 3: lymphostatic elephantiasis, hardening of dermal tissues, papillomas

A 52-year-old male with a diagnosis of Parkinson's disease arrived at an outpatient clinic with complaints of low back pain. During the examination, the physical therapist observed him hesitating with task initiation, difficulty with sit to stands, reduced stride length, reduced speed of walking, and periods of lateral trunk lean. He is able to perform his activities independently and is employed as a bagger at a local grocery store. According to Hoehn-Yahr Classification of disability, the therapist would MOST like classify this patient as: Select one: a. Stage 3 b. Stage 4 c. Stage 1 d. Stage 5

Stage 3 is correct. The patient is experiencing unsteadiness with sit to stand with periods of lateral trunk lean which is seen in stage 3. In stage 4, all symptoms are present and patient also needs assistance with standing and walking. In stage 5, the patient is confined to bed or wheelchair. In Stage 1 symptoms are minimal or absent, unilaterally present.

what posture is this: anterior pelvic tilt with the hips flexed and knees hyperextended.

kypholordotic

Phase 2 cardiac rehab

Subacute period, recovery, healing phase - home health/ outpatient - 4-9 METS -continue working on function - increase duration and intensity of physical activity 3-4 x week, 30-60 mins, 5-10 mins warm up/cool down discharge: 9 METS functional capacity

will hip flexor weakness impact swing or stance more?

Swing

A physical therapist instructs a patient to squeeze a piece of paper between the index and middle fingers while the therapist attempts to pull it away. This type of testing would be the MOST appropriate to assess which of the following myotomes? 1.C6 2.C7 3.C8 4.T1

T1: weakness of finger abduction and ADD

TNM classification system for staging cancer

T= size and extent of tumor N= extent of lymph node involvement M= metastasis does not include rate of growth of the cancer cells

A physical therapist reviews the results of pulmonary function testing on a patient who has emphysema. Assuming the patient's testing was classified as unremarkable, which of the following lung volumes would MOST likely approximate 10 percent of the patient's total lung capacity? - tidal volume - RV - IRV - FRC

Total lung capacity: 4,000-6,000 tidal volume= 10%, 500 IRV= 50-55% RV= 25% (1000) FRC= 40%

T/F: Hydrostatic pressure is directly proportional to the depth and density of water

True

T/F: The use of deep heating agents, electrical stimulation, and traction is generally contraindicated during pregnancy.

True (ice is safe 1st trimester of pregnancy)

A male patient presents to physical therapy. During the evaluation, he was able to write a sentence, but he experienced difficulty reading the sentence. When asked the name of the individual who accompanied him, he was unable to name them. His medical chart reveals a degree of memory loss, topographical disorientation, and homonymous hemianopsia.The patient's clinical presentation is MOST likely correlated with a lesion in which of the following? Select one: a. Anterior cerebral artery b. Central territory of the posterior cerebral artery c. Peripheral territory of the posterior cerebral artery d. Middle cerebral artery

Typically with posterior cerebral artery syndrome, the patient may display the following (not a complete list): contralateral homonymous hemianopsia (or bilateral), visual agnosia, prosopagnosia (increased difficulty naming individuals), dyslexia (without agraphia), memory dysfunction, and topographic disorientation. peripheral territory vs central - central: thalamic pain, involuntary movements, Weber's syndrome, alteration in eye movements

What is forefoot varus? and what is the normal position when in subtalar neutral

When the subtalar joint is in neutral position, Forefoot is inverted to the rearfoot 30-45 deg

is VOR intact in BPPV?

YES

both eyes medial (towards nose) indicates lesion in:

abducens (CN 6) abducting eyes

why would you see ABD in stance (prosthetic)

abducted hip joint long prothesis

Hip inferior glide inc:

abduction

inferior GH glide improves:

abduction

why would you see ABD in stance? (anatomic)

abduction contracture knee instability

spinocerebellum

limb ataxia, dysmetria

A pediatric PT is evaluating a 2 month old child. The PT pulls the child from a supine to seated position using the child's forearms. The child extends the trunk in response to this stimuli. Which of the following statements is the MOST accurate? a. The child has a normal moro reflex b. The child has an normal traction reflex c. The child has an abnormal moro reflex d. The child has an abnormal traction reflex

abnormal traction The traction reflex is tested by pulling a child from supine to sit. A normal response is to fully flex and grasp. An extensor response would be considered abnormal.

foam dressings

absorptive and create an OCCLUSIVE envt for moist wound healing - they are TOO OCCLUSIVE for an infected wound * use with a granular wound that is draining, NOT infected

A Physical Therapist is treating a patient with Achilles tendonitis. Which of the following is the best exercise for this patient ? 1) manually resisted PF 2) resisted walking backward 3) side step down steps 4) seated rockerboard ankle PF

achilles tendinitis: resistance program that emphasizes eccentric loading of the ankle PF (heel lowering exercises) active PF, resisted walking backward= concentric

venous wounds

aching pain worse with dependent position= classic sign bc the dependent position causes increased pressure in the venous system - irregularly shaped, open wound, moderately severe edema - chronic venous insufficiency: edema usually presents in a gaiter distribution creating the appearance of an inverted bottle at the calf

does hypoventilation cause acidosis or alkalosis?

acidosis

which shoulder ligs are disrupted with AC joint injuries?

acromioclavicular & coracoclavicular

A Physical Therapist is re-evaluating a 60 year old female with a diagnosis of right shoulder adhesive capsulitis after five weeks of rehabilitation. The Physical Therapist notes that external rotation is limited and there are ROM restrictions. The patient reports that they have not been able to reach for overhead shelves. Which of the following treatments is LEAST recommended? 1) pendulum exercises with weight cuff to wrist 2) passive stretching above 90 deg 3) progressive resistance training 4) Anterior glide using self-mobilization technique

adhesive capsulitis: posterior & posterolateral glide more effective to increase ER ROM

A physical therapist treats a patient with a peroneal tendon subluxation. To observe the subluxation, which of the following motions is the MOST appropriate to reproduce the subluxation? 1.Active ankle plantar flexion and inversion 2.Active ankle dorsiflexion and eversion 3.Passive ankle plantar flexion and inversion 4.Passive ankle dorsiflexion and eversion

active DF and eversion

which dysfunction can be suppressed with visual fixation?

acute UVH CAN be suppressed with gaze stabilization BPPV CANNOT be suppressed with visual fixation

A patient with an acute burn is referred to physical therapy less than 24 hours after being admitted to the hospital. The patient's burns range from superficial partial-thickness to deep partial-thickness and encompass approximately 35 percent of the patient's total body surface area. Which of the following findings would be the MOST predictable based on the patient's injury? 1.Increased oxygen consumption 2.Hypernatremia 3.Increased intravascular fluid 4.Decreased core temperature

acute burn produces hypermetabolism: inc O2 consumption, inc minute ventilation, inc core temp, dec intravascular and interstitial and intracellular fluids HYPOnatremia

autonomic hyperreflexia

acute elevation in BP with BRADYCARDIA usually occurs at T6 or above -restlessness - nasal congestion - constricted pupils -headache AKA autonomic dysreflexia the patient will present with vasodilation above lesion and vasoconstriction below

A physical therapist reviews a patient's medical history prior to administering intermittent compression. Which of the following conditions would be considered a contraindication when using this type of mechanical device? 1.Venous stasis ulcer 2.Acute pulmonary edema 3.Intermittent claudication 4.Lymphedema

acute pulm edema

avulsion fx of isch tub

acute traumatic incident - localized pain but may radiate down thigh - pain with active or resisted knee flexion

hip superior glide inc:

adduction

uveitis is commonly seen with which MSK condition

advanced ankylosing spondy * this also causes los of chest wall excursion and can compromise breathing so they should do consistent aerobic exercise to optimize the efficiency of O2 transport and to maintain cardiopulm function

Disc herniation at C6-C7 level

affects the C7 nerve root - pain radiating along the back of the shoulder, posterior aspect of the forearm and into the posterior aspect of the long finger. - Weakness of wrist flexors, triceps and finger extensors will also be present.

What are rocker bars used for?

affix to the sole of the shoe proximal to the MT heads to reduce the distance the patient travels during stance phase and to shift the load from the MTP joints to the MTP shaft

pt has decreased eccentric control of elbow flexors. which pnf should you utilize? 1) rhythmic stab 2) dynamic reversals 3) agonist reversal (combo of iso) 4) rhythmic initiation

agonist reversal (combo of isotonic)

if you hear high pitched, musical, whistling sounds during expiration, what do you expect?

airway obstruction

conventional TENS

amplitude: sufficiency for a motor response HIGH pulse frequency (30-150 pps) SHORT pulse duration (50-100)

salicylate use

analgesic

anterior to posterior glide at talocrural joint inc:

ankle DF

tibial nerve lesion

ankle PF weakness and diminished sensation on the posterior aspect of leg and sole of foot

what happens during heel-off to toe-off?

ankle and foot move from 15 deg DF to 20 deg PF mm: gastroc, soleus, peroneals, FHL

common peroneal nerve lesion

ankle dorsiflexion weakness and diminished sensation over the anterior leg and dorsum of the foot.

glide to inc shoulder horizontal ABD

anterior glide (extension, ER, horizontal abd)

What is the MOST appropriate intervention to help increase supination on the right side?

anterior glide at the proximal radioulnar joint posterior/dorsal glide of the radius at the distal radioulnar joint

what glide do you do to inc wrist extension?

anterior glide of proximal row of carpals on the radius and ulna

glide to inc wrist extension

anterior glide of scaphoid

anterior acetabular labral tear

anterior groin pain 2/2 rotational injury - painful clicking - pain with combined end-range hip flexion, ADD, IR

which spinal ligament prevents hyperextension?

anterior longitudinal ligament (ALL)

Lower crossed syndrome

anterior pelvic tilt tight: hip flexors, lumbar extensors weak: gluts and abs

A patient sustained a grade I ankle sprain two days ago during a marching band competition. The patient's description of the mechanism of injury is consistent with inversion and plantar flexion. Which of the following ligaments would MOST likely be affected?

anterior talofibular ligament

where should the ground reaction force (GRF) line fall to maintain knee in extension?

anterior to axis of knee joint GRF always pushes towards the joint

glide to inc elbow flexion

anterior/volar glide to the radius at the humeroradial joint

median nerve pathology at hand

ape hand thenar wasting dec strength with thumb opposition

digital nerve compression test

applying pressure to thumb for Bowler's thumb

what are the typical end feels for the following: - arthrosis - hemarthrosis - meniscal displacement - patellofemoral syndrome

arthritis: empty end feel hemarthrosis (fluid or blood in joint): boggy meniscal: springy PFPS: pain and possible empty

gold standard for adhesive capsulitis

arthrogram/ contrast arthrography radiograph

Yocum test

ask patient to place hand on opposite shoulder (+) if pain with raising elbow = shoulder impingement

school functional assessment (SFA)

assesses and monitors the performance of children in functional tasks/activities in elementary school, social, and academic settings

sputum seen with asthma vs chronic bronchitis vs pneumococcal pneumonia

asthma: eosinophilic bronchodilators will improve spirometry chronic bronch: neutrophilic pneumococcal pneum: pinkish/blood-flecked/rusty sputum with bacteria

piriformis action:

at 90 deg hip flexion, piriformis becomes an internal rotator and ABD

if you hear crackles at inspiration or decreased lung sounds what do you expect?

atelectasis may also hear crackles with pulmonary edema

mallet finger deformity

avulsion of extensor tendon at distal phalanx DIP rests in flexed position

what happens with optic tract lesion

contralateral homonymous hemianopia

what happens with occipital lobe lesion

contralateral homonymous hemianopia with macular sparing

L sided HF signs and sx

back up of blood into the pulmonary system and decreased cardiac output - dry cough - wheezing - tachy - lightheaded - pallor, cyanosis - progressive dyspnea and productive spasmodic cough due to pulmonary edema

Dupuytren's contracture

banding at the palm + digit flexion contractures

treatment for horizontal canal BPPV

bbq roll, lempert, Baloh

Med that lowers HR during BOTH rest AND exertion

beta receptor agonists

nondisplaced midshaft radial and ulnar fx casted in mid-range elbow flexion with neutral forearm. which mm would be shortened?

biceps brachioradialis

A patient with complete C5 tetraplegia works on a forward raise for pressure relief. The patient utilizes loops that are attached to the back of the wheelchair to assist with the forward raise. Which muscles should be particularly strong in order for the patient to be successful with the forward raise? 1.Brachioradialis, brachialis 2.Rhomboids, levator scapulae 3.Biceps, deltoids 4.Triceps, flexor digitorum profundus

biceps and deltoid biceps: flex shoulder and elbow, supinate delt: flex, ext, ABD

which hamstring tendons are prominent with a leg curl

biceps femoris: lateral tendon semitendinosus: medial tendon

speed

biceps tendinitis or partial rupture

Cullen's sign

bluish or purple discoloration around umbilicus. Indicates intra-abdominal bleeding or signs of pancreatitis

vesicles vs blisters

both filled with serous fluid vesicles are smaller, blisters are large in size

basal cell carcinoma vs squamous cell carcinoma

both malignant BCC- more common, sun exposed, face, fair skinned - raised patch, rolled border, indented center/thickened area of skin - Tissue destruction risk but low metastatic risk SCC- less common, sun exposed areas: face, neck, back of hand - poorly defined margins: flat, red area, ulcer or nodule, grows quickly

A physical therapist examines a patient who has rheumatoid arthritis. During the examination, the therapist observes increased flexion at the proximal interphalangeal joints and hyperextension at the metacarpophalangeal and distal interphalangeal joints. The therapist would MOST likely document which of the following deformities in the medical record? - boutonniere - swan neck - mallet finger - ulnar drift

boutonniere

severed musculocutaneous nerve: which mm compensate?

brachioradialis (radial) and pronator teres (median)

platypnea

breathlessness or SOB with sitting relieved by laying supine not seen in CHF bc they have orthopnea (difficulty breathing in supine)

effect of climate on exercise induced asthma

bronchospasm is blunted when exercising in HUMID envt

iliac artery occlusion

buttock, thigh, hip diminished femoral pulses

a 21 year old patient has been receiving PT for 3 weeks s/p L knee ACL. what is a goal at this time: 1) preventing reflex inhibition of mm 2) achieving dynamic control of L knee 3) achieving full, pain-free ROM 4) improving kinesthetic awareness

by week 3, you want to work on preventing reflex inhibition of mm prevent atrophy in quads and hammies in order to maintain stabilization weeks 4-10: PT will focus on improving kinesthetic awareness during closed chain activities, full pain-free ROM, and dynamic control of knee

if cadence is 120 steps per minute, how many seconds will it take to complete 120 full strides?

cadence= steps per minute 120 full strides= 60 stride lengths per minute = will take 2 mins (or 120 secs) for 120 full strides

inversion & DF ankle sprain affects what ligament?

calcaneofibular

acetate use

calcium deposits

A Physical Therapist is using iontophoresis on the quadriceps femoris on a patient who has myositis ossificans status post a baseball injury. What is the best suited medication and electrode?

calcium deposits= acetate (-) ion= cathode

Rhabdomyolysis

can be side effect of cholesterol lowering drugs (-statin) Rhabdomyolysis is a potentially fatal condition is which myoglobin and other muscle tissue contents are released into the bloodstream as a result of muscle tissue disintegration. This could occur with acute trauma, severe burns, overexertion, from alcohol abuse or alcohol poisoning or with statins. It leads to muscle aches, cramps, weakness and soreness. Dark color of urine is due to liver failure.

Middle phalanx dislocation

can cause central slip to be torn and results in a Boutonniere deformity (DIP hyperextension and PIP flexion)

sx of AA subluxation (Down's)

can cause spinal cord impingement - decreased strength - decreased ROM - hyperactive reflexes (3+) - decrease in mm tone - spasticity - persistent head tilt

Semont maneuver: do you start with head to the affected or unaffected side? Canalith repositioning maneuver?

canalith: turn head to affected side Semont: start with turn head away from affected side

Canalithiasis vs. Cupulolithiasis

canalithiasis lasts <1 min - latency period 1-40 seconds - nystagmus fluctuates in intensity cupulo >1 min - immediate, persistent (bc crystals stuck to cupula)

A physical therapist is considering the use of an antibacterial agent for dressing a pressure wound with the goal of controlling an infection. Which of the following is LEAST likely to be used as an antibacterial? Select one: a. Silver sulfadiazine b. Silver-based dressing c. Iodine-based dressing d. Charcoal-based dressing

charcoal based dressing (foam): is used mostly for odor reduction

correct CPR sequence

check for responsiveness, call EMS/get AED if unresponsive: initiate CPR with cycle of 30 compression and 2 breaths if pt has pulse but is not breathing: initiate rescue breaths 10-12/min if patient refuses then collapses: can initiate CPR bc consent will be considered unless patient has signed DNR

cheyne stokes vs biot

cheyne: end of life; irregular and characterized by a period of apnea followed by gradually inc depth and frequency biot: highly variable respiratory depth and intermittent periods of apnea

latissimus dorsi stretch

child pose

A patient reports of a band like aching pain in his right upper abdomen and right upper back, which increases after eating a heavy fatty meal. He has a history of peptic ulcers and takes OTC antacids, which have helped in alleviating his pain. Upon palpation under the ribs, the patient stops breathing and winces due to pain. What do you suspect to be the cause of this new presentation?

cholecystitis will also see (+) Murphy: pain with palpation under ribs

scleroderma

chronic progressive disease of the skin and internal organs with hardening and shrinking of connective tissue. inflammation and fibrosis of many parts of the body, including the skin, blood vessels, synovium, skeletal muscle, and certain internal organs such as kidneys, lungs, heart, and GI tract. "A patient is being evaluated in a hospital following acute kidney failure. During evaluation, the PT notes that the patient has ovular patches of skin that have hardened, and the patient's nail beds appear blue."

Ankylosing spondylitis (Bekhterevs or Marie-Str€umpell disease)

chronic, progressive inflammatory disorder. The sacroiliac joints, spine, and large peripheral joints are primarily affected. Common characteristics of AS: - insidious onset of middle and low back pain - stiffness for more than 3 months in a person (usually male) - under 40 years of age. - Stiffness is worse in the morning - lasting more than 1 hour; localized to the pelvis, buttocks and hips. - sx increase with repeated extension

what to focus on after PCL reconstruction

closed chain quad strengthening to act as a dynamic restraint to posterior tibial translation avoid strengthening hammies right away bc will contribute to the posterior tibial translation

TMJ posterior glide

closing mouth

talipes equinovarus

club foot - forefoot add - hindfoot varus - PF ankle --> ankle DF limited

A 54-year-old female was admitted to a hospital and underwent an uncomplicated total hip arthroplasty using an anterior approach. On post-op day two, the PT is educating the patient on precautions in order to prevent hip dislocations. Which of the following is a combined motion that the patient should avoid?

combined: FABER (flex, ABD, ER) avoid: ext, add, ER

frieberg disease

commonly involves AVN of the 2nd MT epiphysis and leads to collapse of the osteochondrotic deformity - pain localized to MT head, exacerbated by activity - ROM limited - joint swelling - occasional plantar callosity under 2nd MT head

concurrent validity

compare a test or score with a measure that has already been validated (to a gold standard)

medial inferior pontine syndrome

contralateral hemiparesis of face and extremities ipsilateral nystagmus lesion of CN 6 (lateral rectus) causes ipsilateral medial strabismus

A patient is currently taking Neurontin (gabapentin) for the treatment of seizures. Based on the prescribed medication, the patient may have the MOST difficulty when performing interventions that attempt to improve which of the following components? 1.Coordination 2.Verbal comprehension 3.Strength 4.Flexibility

coordination gabapentin uses: seizures, nerve pain following shingles/trigeminal neuralgia, diabetic neuropathy side effects: ataxia, fatigue

A patient who has chronic pulmonary dysfunction is placed on a corticosteroid medication to reduce mucosal edema and inflammation. Which of the following cardiovascular side effects is the MOST common with use of corticosteroids? 1.Palpitations 2.Arrhythmias 3.Increased blood pressure 4.Tachycardia

corticosteroids inc BP and cause htn

shoe rx for pes cavus

cushion sole to absorb shock metatarsal bar lateral flare to inc stability

hallmark radiographic features of OA:

cystlike radiolucent lesions sclerotic subchondral bone osteophyte formation joint space narrowing

A patient diagnosed with multiple sclerosis uses extensor tone to assist them to successfully complete a sit to stand transfer. Which pharmacological agent would MOST limit the patient's ability to complete the transfer? 1.Calcium carbonate 2.Dantrolene sodium 3.Levodopa 4.Secobarbital

dantrolene sodium: post synaptic mm relaxant would dec spasticity

common surgical intervention for spondylolisthesis

decompression and spinal fusion

A physical therapist is treating a patient with COPD along with multiple comorbidities. The therapist has been monitoring the patient closely. Which of the following would NOT be considered criteria for terminating the exercise? Select one: a. A decrease in PaCO2 by 11 mm Hg b. Decrease in blood pressure with increasing workload c. Systolic BP 252 mm Hg, increase in diastolic BP by 20 mm Hg d. An increase in PaCO2 by 11 mm Hg

decrease in PaCO2 by 11 mmHg is not an absolute contraindication bc a patient with COPD already has increased CO2 Absolute stops: - inc in CO2 by 10 or over 65 - inc in DBP by 20 or SBP>250 - dec in BP with increasing workload

During vigorous exercise, a 30-year-old patient would expect what ACUTE changes to occur in the muscle capillary bed?

decreased PO2 When the oxygen consumption in muscle suddenly increases, an immediate way to increase supply is an enhanced extraction of oxygen from the haemoglobin in the flowing blood. An enhanced extraction can, however, only occur either when the oxygen binding curve is considerably shifted to the right or when the perivascular pO2 decreases. The former requires rapid and significant local increases in pCO2 or H+concentration. Therefore, most of the increased extraction should result from a decrease in perivascular pO2 due to enhanced oxygen consumption.

which burn wound classification produces a keloid scar?

deep partial thickness

Pacinian corpuscles

deep pressure and vibration located in subcutaneous tissue

lesion at midbrain

deficits in CN 2, 3, 4

abduction in the scapular plane strengthens which mm?

deltoid supraspinatus

ASIA A C5 SCI What are some functional capabilities, equipment, and AD that would be characteristic for this patient?

dependent transfers indep driving with Adaptive equip upper and LE independent dressing and grooming power wc, adapted keyboard controls

zinc use

dermal ulcers

Krause's end bulbs

dermis and conjunctiva touch and pressure

topographical orientation

determining the location of objects and settings and the route to the location difficulty remembering the relationship of one location to another (i.e. patient cannot find his/her way back to the room lesion involves damage to the right retrosplenial cortex with Brodmann's area

Meniere's disese

diagnosed symptomatically thru fullness in the ear and tinnitus as well as episodic vertigo can cause unilateral vestibular hypofunction - (+) Romberg, tandem Romberg, SLS, changes in gait, head turns, head-shaking-induced nystagmus

Lunate fx MOI

direct blow to wrist or recurrent trauma to a hyperextended wrist can cause AVN of the lunate: Kienbock's disease

A physical therapist documents in the medical record that a patient has moved from stage 5 to stage 6 of Brunnstrom's Stages of Recovery. This type of transition is characterized by which of the following changes in movement patterns? 1.Absence of associated reactions 2.Disappearance of spasticity 3.Voluntary movement begins outside of synergy patterns 4.Return of normal motor function

disappearance of spasticity

cushing disease vs syndrome

disease: tumor on pituitary syndrome: tumor on adrenal gland

which of the following is NOT pressure tolerant: - patellar tendon - fibular shaft - gastroc mm - distal anterior tibia

distal anterior tibia

cerebrocerebellum

distal limb function planning timing and coordination of voluntary movement

colle's fx

distal radius dislocated in DORSAL/posterior (dinner fork deformity)

smith's fracture

distal radius dislocated in a VOLAR direction garden spade deformity

A physical therapist examines a patient who reports abdominal pain. The patient's symptoms include left lower quadrant abdominal pain, loss of appetite, and nausea. The clinical presentation is MOST consistent with which of the following medical conditions? 1.Diverticulitis 2.Appendicitis 3.Peptic ulcer 4.Pancreatitis

diverticulitis - LLQ pain - nausea - bloating - flatulence - bloody stools - constipation or diarrhea

face validity

does the test "look valid" to those who take and administer it? does it measure it what it appears to measure (superficially)

content validity

does the test measure what it is supposed to- does it represent all facets of a given concept or construct?

ropinirole (requip) purpose

dopamine agonist for PD side effects: nausea, sedation, dizziness, constipation, hallucination

A physical therapist attempts to assess the temperature of a patient's skin in an area susceptible to a pressure ulcer. Which area of the therapist's body would be the MOST appropriate to utilize when assessing the patient's skin temperature?

dorsum of the hand most sensitive to temp changes in the body

geotropic nystagmus

down beating (towards ground) canalithiasis involved ear has stronger nystagmus

do lats upwardly or downwardly rotate the scap?

downward

A therapist is observing a patient with a transtibial prosthesis and notices "drop off" during the late stance phase of gait. Which of the following prosthetic/anatomical causes would be the LEAST LIKELY cause of this gait deviation? a. Socket too far posterior b. Insufficient plantarflexion c. Keel too short d. Knee flexion contracture

drop off= early knee flexion during stance phase A: socket too posterior causes insufficient knee flexion anatomical causes: insufficient PF or knee flexion contracture prosthetic causes: DF stop too short, high shoe heel, keel too short, socket too anterior

ichthyosis

dry scaly skin - hypothyroidism - hashimotos

which parameters controls the recruitment of peripheral axons during therapeutic e-stim

duration of stimulus

normal cardiovascular changes during 1st trimeter of pregnancy: BP

during first trimester, BP decreases (slight dec in systolic and greater decrease in diastolic) BP continues to dec approx thru mid pregnancy BP increases from mid pregnancy to 6 weeks after delivery: attributed to inc in venous pressure as a result from an increase in uterine size and increase in venous distensibility blood volume inc during pregnancy and returns to normal 6-8 weeks after delivery Q increases (HR and SV increase)

A physical therapist reviews a patient coverage form that lists the following parameters used during a recent ultrasound treatment: 1.5 W/cm2, pulsed 20%, 1 MHz, 6 minutes. If the objective of the ultrasound treatment was to increase tissue temperature, what parameter would be the MOST critical for the therapist to alter? 1.Time 2.Duty cycle 3.Frequency 4.Intensity

duty cycle: on/off time

prime movers in active SLR

dynamic hip flexion and isometric contraction of the quads rectus femoris prime mover

sustained limb posturing is a classic characteristic of:

dystonia

Ototoxicity

ear poisoning caused by drugs/chemicals

Hyaluronidase use

edema reduction

posterior glide of humeroradial:

elbow extension

involvement of C7 myotome

elbow extension, wrist flex

humeroulnar distraction increases:

elbow flex/ext

anterior glide of humeroradial:

elbow flexion

most effective intervention for urge UI to reduce detrusor or bladder activity

electrical stimulation to the detrusor muscle

what ranges stress CFL

end range DF and inversion

patient with anterior GH instability would have pain with:

end range ER

patient with acromioclavicular arthritis would have pain with:

end range abduction (170-180 deg)

A physical therapist treats a patient diagnosed with myasthenia gravis. Which of the following tests should the therapist MOST likely expect to be abnormal with this condition? 1.Coordination testing 2.Sensory testing 3.Deep tendon reflex testing 4.Endurance testing

endurance: myasthenia gravis is characterized by mm weakness and significant mm fatigability

A physical therapist works with a child who walks with an equinus gait pattern. Which of the following interventions would be the MOST appropriate to address the muscle shortening associated with this gait pattern? 1.Side stepping 2.Backward stepping 3.Activities in single leg stance 4.Toe walking

equinus: shortened PF 2: backward stepping- elongates PF and hammies

most common compensation for forefoot valgus

excessive midtarsal or subtalar (rearfoot) supination

uncompensated pes cavus leads to:

excessive supination

uncompensated tibial ER leads to:

excessive supination

A physical therapist is reading the lab reports of a patient and notes that the findings are correlated with metabolic acidosis. Which of the following would be the LEAST likely correlated with a potential cause of metabolic acidosis? Select one: a. Severe diarrhea b. Chronic Kidney disease with renal failure c. Excessive vomiting d. Ingestion of large quantities of aspirin

excessive vomiting would cause metabolic alkalosis

how does exercise affect insulin sensitivity?

exercise INC insulin sensitivity and ENHANCES the effect of insulin (glucose intake should be increased to counter the effects of exercise) - during a prolonged activity, a snack is recommended for every 30 mins of activity

A 33-year-old male visits an outpatient clinic 2.5 weeks post repair of flexor tendons of the hand. During this phase of rehabilitation, which exercise would MOST likely be performed? Select one: a. Adding carefully controlled active metacarpophalangeal flexion within a protected range with the wrist stabilized in extension. b. Exercise in a wrist tenodesis splint c. Exercise in a static palmar blocking splint d. Achieving full active flexion and extension of wrist and digits

exercise in a wrist tenodesis splint (maximum protection phase) most exercises will be performed in a dynamic dorsal blocking splint or wrist tenodesis splint

position to expose supraspin tendon

extension and IR

both moro and startle reflexes share what response

extension and abd of UE

vertebral artery compression 5 D's and 3 N's

extension and rotation 5 D's: dizziness, diplopia, drop attack, dysphagia, dysarthria 3 N's: nausea, nystagmus, numbness

capsular pattern for the 1st MTP joint

extension loss is greater than flexion loss

A physical therapist treats a patient who has lower extremity weakness due to a laceration injury to the tibial nerve. Which movement would LEAST likely be affected by this nerve injury? 1.Plantar flexion of the ankle 2.Extension of the great toe 3.Flexion of the great toe 4.Flexion of toes 2-5

extension of great toe: deep peroneal nerve

characterize forward head posture in terms of cervical flexors and extensors

extension of upper cervical spine flexion of lower cervical spine increase in resistance moment arm of head

D1 LE extension

extension, ABD, IR, knee flexion or extension, ankle PF and inversion

A 29-year-old female endurance athlete presents to your clinic with chronic groin pain. She was referred by a doctor for groin pain increasing with activity. She has a history of anorexia and osteopenia. Her hip ROM is painful, along with palpation of the greater trochanter. Which of the following condition is MOST consistent with these findings? A. Avascular necrosis of the femoral head B. Iliotibial band syndrome C. Femoral neck stress fracture D. Greater Trochanteric bursitis

femoral neck stress fx (osteopenia inc chance of stress fx) not bursitis bc it will cause pain at rest

A 53-year-old male patient post right cerebrovascular accident is currently being treated at a skilled nursing facility with comorbidities including controlled HTN, atrial fibrillation, controlled type 2 DM. The physical therapist noticed during the initial evaluation that the patient has difficulty making a distinction between an armhole from the entire shirt. Which of the following is the MOST appropriate way to document this presentation?

figure ground discrimination shirt= ground, armhole= figure inability to locate objects that are not prominent in a visual image lesion area: commonly R parietoccipital

A 72 year-old male who has been experiencing night sweats as well as sudden weight loss. He had previously been admitted to the hospital secondary to complaints of breathlessness and muscle weakness with a diagnosis of Tuberculosis. Currently he is taking first-line anti-TB medications. The Physical Therapist is evaluating the patient at home, and the patient expresses concern as he has noticed his saliva and urine to have changed color to orange-red. Which of the following will be the MOST APPROPRIATE action for the Physical Therapist? 1) tell the pt that it is a common side effect of TB meds 2) ask the patient to stop taking the meds 3) ask the patient to take the meds on alternate days 4) call the physician immediately to report hepatic cirrhosis

first line anti-TB drugs: isoniazid, rifampicin, ethambutol, pyrazinamide (PZA) rifampicin can cause red-orange discoloration of body fluids (urine, tears, saliva, sweat, sputum)

what ROM is decreased with avascular necrosis in the hip

flexion IR ABD

action of scalene with rib fixed

flexion, same sidebending, opposite rotation

A physical therapist instructs a patient to make a fist. The patient can make a fist, but is unable to flex the distal phalanx of the ring finger (4th digit). This clinical finding can BEST be explained by a ruptured tendon of which muscle? 1) FCR 2) FDS 3) FDP 4) Extensor digitorum communis

flexor digitorum profundus: flexes the DIP and assists in flexion of PIP the FDS flexes the PIP and assists in flexion of MCP and wrist

what is a floor effect? What is a ceiling effect?

floor: When a task is so difficult that examinees are unable to perform well on the least challenging items and all scores are very low. when the instrument does not register a further decrease in score for the lowest scoring individual ceiling: test items are not challenging enough so the score will not inc for a patient who has the potential to clinically improve bc they've already reached the highest score.

copper use

fungal infections

Smith's fracture

garden spade deformity radius dislocated in a volar direction (up toward palm)

For a normal ankle strategy response, which of the following is MOST accurate when referring to the order of muscle activation for a forward directed perturbation?

gastroc, hamstrings, paraspinals ankle strategy: distal to proximal mm activation backward sway: tib ant, quads, abdominals

tight heel cords will pull knee into

genu recurvatum

coxa valga is associated with:

genu valgus increased Q angle femoral anteversion hyperpronation

tomography

good for lesions such as - subtle fx - fx lines - presence and extent of fx healing

lisfranc injury

occurs at midfoot at and may include fx or dislocation caused by traumatic disruption of the tarsometarsal joints midfoot: cuboid, navicular, 3 cuneiforms & their articulation with the bases of the 5 MT bones

A 62-year-old female patient arrived at an outpatient clinic with complaints of generalized weakness. She has a PMH of type II diabetes, chronic kidney failure, arthritis, polyneuropathy. Upon further examination, the therapist identifies a closed wound on the plantar aspect of the forefoot with bone deformation with hyperkeratosis. The therapist suspects this as an unusual wound. According to the Wagner classification scale, this wound is MOST likely categorize as: - grade 0 - 1 -2 -4

grade 0 : closed wound with hyperkeratosis and bone deformation

obliteration of the lovibond angle

grade II clubbing

inferior pubic ramus fx

gradual onset of groin pain that inc with WB and walking

A physical therapist reviews the parameters of several pain modulation theories using transcutaneous electrical nerve stimulation (TENS). When comparing sensory stimulation to motor stimulation, sensory stimulation requires which of the following modifications to the parameters? 1.Greater phase duration 2.Greater frequency 3.Stronger amplitude 4.Shorter treatment time

greater frequency conventional tens: high frequency, short pulse duration

posterior tibial tendon dysfunction

hallmark sign: finding of too many toes= forefoot ABD and hindfoot valgus tib post plantar flexes, inverts, and supports medial arch

floor (ground) reaction AFO

has an anterior shell that provides a posteriorly directed force to resist knee flexion during stance

A 30-year-old obese female complains of having a foggy brain, dry itchy skin, low tolerance to cold weather, and decreased energy. She reports that she is gaining weight despite having a poor appetite and eating less. Blood reports show TSH of 14. Which gland is responsible for the secretion of this hormone?

he Pituitary gland is responsible for the secretion of TSH which stimulates the thyroid gland to release thyroid hormones T3 and T4 which are primarily responsible for controlling the metabolism of the body. With hypothyroidism there is a decrease in T3 and T4 and as a result TSH is elevated in as the pituitary gland is trying to compensate, therefore elevated TSH is an indicator of hypothyroidism. Reference: Goodman, Pathophysiology Implication for the Physical Therapist, Fourth Edition, Page 487

The therapist determines that the patient's hip width in the sitting position and the measurement from the back of the buttocks to the popliteal space are each 16 inches. Given these measurements, which of the following wheelchair specifications would BEST fit this patient? (seat width and height)

height: -2 = 14 in seat width: +2 = 18 in

normal hematocrit and platelet levels

hematocrit 37-52% (20%: no resistive ex) platelet 150,000-450,000 (<50,000 no resistive ex)

A physical therapist attempts to implement a formal exercise program for a patient who is three weeks post cardiac transplantation. Which of the following physiologic responses should the therapist MOST anticipate based on the transplantation? 1.Increased resting heart rate 2.Increased heart rate response with exercise 3.Increased peak heart rate during exercise 4.Increased age-predicted maximal heart rate

inc resting HR

expected ABG's for COPD patient

high PaCo2, low pH decreased PaO2

syndesmosis injury

high ankle sprain pain over the anterior or posterior tibiofibular ligaments and interosseous membrane with passive ER stress to the foot and ankle Kleiger test (positive ER stress test)

what can cause a high heel rise during early swing? slack extension aid or taut?

high heel rise= excessive knee flexion slack extension aid will cause excessive knee flexion taut extension ad would cause abrupt knee extension during terminal impact/late swing

why would you see a lateral bend in stance? (prosthetic)

high medial wall low lateral wall short prosthesis

metabolic alkalosis

high pH, high HCO3 excessive vomiting or upper gastrointestinal suctioning, diuretic therapy, or ingestion of large quantities of base substances such as antacids. - nausea, vomiting, numbness, convulsions

shoe modification for hammer toe

high toe box to provide space and dec irritation

a patient performs 1 sit up with arms crossed across chest and 1 with arm behind head. the relationship between EMG and force indicates that EMG amplitude(mv) of core abdominal muscles:

higher with arms behind the head (easier across chest)

primary risk factors for atherosclerosis

htn hyperlipidemia cigarette smoking

teres major action

humeral ADD GH IR

most occlusive dressing

hydrocolloids NOT good for infected wounds * indicated for wounds with low to moderate drainage that need protection from bacteria or other contaminants

protruding eyeballs: hypo or hyperthyroid

hyper exophthalmos (retraction of upper eyelids which can lead to bulging eyes)

What is swan neck deformity caused by?

hyperextension of PIP and flexion of DIP contracture of intrinsic mm along with dorsal subluxation of the lateral extensor tendons

medication for if u see inverted T waves

hypokalemia (in verted t waves) vs hyperkalemia (peak) - hydrochlorothiazide can cause hypokalemia as a side effect prescribe beta blocker to regular heart rhythm

neurogenic shock

hypotension bradycardia cyanosis warm-dry extremities decreased cardiac output peripheral vasodilation venous pooling

clunk test

identifies a glenoid labrum tear

what compensates for weak shoulder ABD?

if mid delts are weak, biceps compensates for ABD when shoulder is in ER

extensor tendon repair

immobilization in an extended position - zone 3/4: PIP and DIP in extension - zone 5/6: wrist in 30 deg of extension with MCP in 30-45 deg flexion

Walking on lateral edge of foot and diminished heel off (terminal stance

impaired eversion and PF peroneus longus: superficial peroneal nerve

A physical therapist examines the breath sounds of a patient diagnosed with pulmonary disease. The therapist identifies crackles during both inspiration and expiration. This finding is MOST representative of which of the following conditions? 1.Pleural effusion 2.Pulmonary fibrosis 3.Impaired secretion clearance 4.Localized stenosis

impaired secretion clearance= will hear crackles during inspiration and expiration pleural effusion= will hear dec lung sounds and pleural friction rub

cellulitis

infection of skin: inflammation, edematous, hot, red may be associated with fever

Posterolateral displacement of elbow

in order for the coronoid process to be inferior to the trochlea, the ulna has to be displaced posterolateral to the humerus

A Physical Therapist is treating a patient who is anxious about exercising as she is twenty four weeks pregnant. Which of the following physiological effects of aerobic exercise is NOT expected in a pregnant patient? 1. Cardiac reserve is increased during exercise 2. Hematocrit levels will increase during vigorous exercise 3. Respiration rate will not increase proportionally with moderate and severe exercise 4. Stroke volume and cardiac output will increase with steady state exercise

in pregnant patients, cardiac reserve decreases during exercises SV and Q both increase with steady state exercise maternal RR will adapt to mild exercise but will not increase proportionally with moderate and severe exercise maternal hematocrit is lowered during pregnancy but will rise with vigorous exercise

if a pt has flexion synergy, which way should you mobilize the scap?

in protraction and upward rotation (to preserve the GH rhythm that prevents soft tissue impingement in the subacromial space during overhead arm movements)

glaucoma

increased intraocular pressure- can damage optic nerve - characterized by loss of peripheral vision and sparing of central vision

thyroid storm

increased temp, pulse and HTN nervousness, irritability, heat intolerance and tachycardia.

how does inc hydrostatic pressure affect BP

increases BP

A physical therapist reads in the medical record that a patient has an indurated ulcer on their lower leg. Which method was MOST likely used to identify the induration? 1.Diagnostic imaging 2.Measurement 3.Observation 4.Palpation

induration= firmness or hardening palpation

Which of the following would be not be considered an emergency in a patient who has cancer? - spinal cord compression - fever in immunocompromised patients - sudden loss of limb function - inferior vena cava syndrome

inferior vena cava syndrome: caused by physical invasion or compression by a pathological process or thrombosis within the vein itself. It can also occur by obstruction during pregnancy ** superior vena cava: EMERGENCY bc it is caused by compression of the wall 2/2 malignant tumor in the mediastinum

shoe prescription for sesamoiditis

inflammation of sesamoid bones - transverse metatarsal bar (to redistribute pressure from MT heads to the shafts) - rocker sole to reduce motion

rate arrival of the following cells to site of injury 1.Endothelial cells 2.Fibroblasts 3.Leukocytes 4.Platelets

inflammatory: platelets then leukocytes proliferative: endothelial cells and fibroblasts

flaccid/non-reflexive bladder

injuries BELOW T12 no reflex action of detrusor - cannot establish reflex voiding bc the reflex arc is not intact (no tapping or stroking of suprapubic region) interventions: - crede maneuver (manual pressure on bladder) - valsalva to increase IAP - intermittent catheterization

anterior interosseous syndrome

injury to anterior interosseous nerve (branch of median nerve that can be pinched as it passes thru the 2 heads of pronatus quadratus) - flexor pollicis longus - flexor digitorum profundus - pronator quadratus

soft heel cushion

insufficient knee flexion in early stance

why would you see dynamic genu valgum during stance

ipsilateral glut med (L4-S1)

wheal

irregular edematous eruptions

A physical therapist treats a patient who had a reverse total shoulder arthroplasty. What is the MOST likely reason the patient had this surgical procedure as opposed to a total shoulder arthroplasty?

irreparable supraspinatus tear if a RTC cannot be repaired, reverse TSH is indicated (changes the ball and socket location so that glenoid is convex and humerus is concave)-- provides increased deltoid moment arm so patient can elevate arm in the presence of an insufficient rotator cuff

A 55 year old male arrived at an outpatient physical therapy clinic with complaints of unsteadiness during gait. During the evaluation, the patient mentioned having an early diagnosis of Parkinson's Disease. Upon further examination, the physical therapist administered the Clinical Test of Sensory Interaction on Balance (CTSIB); the results indicated that the patient experienced instability while standing on a foam surface with eyes open, standing on a foam surface with eyes closed, and standing on a foam surface with a visual conflict dome. The therapist concluded that this patient MOST likely: Select one: a. Has sensory selection problems b. Is dependent on the vestibular system c. Is dependent on the somatosensory system d. Is dependent on the visual system

is dependent on the somatosensory system is correct. There are 6 conditions to the test: Standing on a firm (stable) surface with the eyes open Standing on a firm (stable) surface with eyes closed Standing on a firm (stable) surface with a visual conflict dome Standing on a foam (moving) surface with the eyes open Standing on a foam (moving) surface with the eyes closed Standing on a foam (moving) surface with a visual conflict dome Patients that may be dependent on the somatosensory system may become unstable in conditions 4,5 and 6 (involves standing on foam surfaces with eyes open/ closed, standing on foam surface visual conflict). Patients dependent on vision become unstable in conditions 2,3,5 and 6. Patients with vestibular loss become unstable in 5 and 6, as they can not rely on vision or somatosensory function.

predictive validity

is the score helpful in predicting a future outcome? (ex: career or amplitude tests)

purpose of using an abdominal binder with weak core

it can compensate for laxity and improve respiratory function- in sitting, the abs will shift inferiorly and anteriorly when weak, which causes the diaphragm to pulled into a more horizontal position, putting it at a mechanical disadvantage

A patient presents to the clinic with a chief complaint of dizziness and swaying to the right side while walking. She had a viral infection about 2 weeks ago and was still recovering from it when these symptoms started. She complains of feeling unsteady and nauseous when walking down the aisle of the grocery store. Which of the following tests will MOST LIKELY be positive while assessing this patient? a. Head Thrust test b. Dix-Hallpike test to the right c. Cover-Uncover test d. Horizontal Roll test to the left

labrynthitis or UVH: ringing in ears, swaying on 1 side, history of viral info - with a U/L peripheral lesion, pt will not be able to maintain gaze when the head is rotated quickly towards the side of the lesion= will have a corrective saccade, making the head thrust test (+) * cover uncover test is used to assess phoria or alignment of the eye

sx of hyposecretion of pancreas

lack of insulin secretion - inc RR - irregular HR - increased fatty acid metabolism - acetone breath

vitiligo

lack of pigmentation on forearm or face seen with hyperthyroidism

common surgical intervention for spinal stenosis

laminectomy to relieve pressure on affected nerves

3 cords of brachial plexus

lateral, posterior, medial

lateral disc hern vs large central disc hern

lateral: LE sx elicited with SLR, achilles 1+ large central disc hern: back pain during SLR testing

Gerber

lift off sign for subscap (IR)

A physical therapist observes a patient during gait training. The patient has normal strength and equal leg length. The patient slightly vaults and exhibits early heel off during midstance. Which of the following impairments is the MOST likely cause of this deviation? 1.Weakness of the dorsiflexors 2.Weakness of the hip abductors 3.Limited plantar flexion 4.Limited dorsiflexion

limited DF- compensate with vaulting or bouncing in mid-late stance * weak DF would create foot slap and steppage gait

A 3-year-old patient is being evaluated by a physical therapist with a diagnosis of right sided congenital muscular torticollis (CMT). Which of the following range of motion (ROM) limitations of the head will MOST likely be displayed by the patient?

limited R rotation and L side flexion R SCM involvement: head positioned in R side flexion and L rotation while displaying ROM in L side flexion and R rotation

anterior stop orthotic

limits DF, which aids patients that have paralysis of triceps surae to achieve propulsion during late stance

coracohumeral lig

limits flexion and ext of GH joint

Role of Supraspinous Ligament

limits flexion of spine

role of interspinous ligament

limits flexion through restricting separation of the spinous processes of the vertebral column

pubofemoral ligament role

limits hip ER of femur

iliofemoral ligament role

limits hip IR and ER in hip extension

what does tibionavicular lig limit

limits lateral translation and lateral rotation of talus

strengthening: longer pulse duration or shorter?

longer pulse duration so more time is spent delivering current

claw finger deformity

loss of intrinsic muscle action and the overaction of the extrinsic (long) extensor muscles on the proximal phalanx of the fingers. The metacarpophalangeal joints are hyperextended, and the proximal and distal interphalangeal joints are flexed.

A patient presents to an outpatient clinic with an abnormal blood pressure. The patient has a history of hyperthyroidism. What is MOST LIKELY seen in this patient? (in terms of BP and tolerance)

low BP, heat intolerance signs and sx for hyperparathyroidism: - heat intolerance - LOW BP - tachy - weight loss - fatigue - hyperactive reflexes - increased sweating - tremor - nervousness - polydipsia - weakness - increased appetite and dyspnea

A patient, in the end stages of ALS, is receiving palliative care via hospice. The patient reports feeling confused and the Physical Therapist notes that the patient has hyperkalemia. Which of the following blood gas values and complication should the Physical Therapist suspect? 1. pH 7.43, PCO2 40 mmHg, the patient is in partially metabolic alkalosis 2. pH 7.47, PCO2 35 mmHg, the patient is in compensated metabolic acidosis 3. pH 7.25, PCO2 54 mmHg, the patient is in ventilatory failure 4. pH 7.27, PCO2 53 mmHg, the patient is alert and talkative and presents with normal ABG levels

low pH, high pCO2: respiratory acidosis = ventilatory failure

causes of metabolic acidosis

low pH, low bicarb - ketoacidosis - chronic kidney disease - lactic acidosis - diarrhea - ingestion of large quantities of salicylates/acetylsalicyclic acid

how to test mid vs low trap strength

low trap: prone, superman Y (arm overhead and IR) mid trap: prone, arms at 90 deg ABD with ER

tight lats: what happens to the lumbar spine?

lumbar spine can't extend - will see limited shoulder flexion/elevation

how does aquatic therapy affect lung expansion?

lung expansion is INHIBITED by the hydrostatic pressure against the chest wall & increased circulation to the chest cavity/center of the body

what % of fluid is normally collected by a functioning lymphatic and venous system?

lymphatic: 10-20% venous: 80-90%

A patient with multiple sclerosis with a history of optic neuritis is present. When shining a light in the patient's right eye, a reflex contraction is produced in both eyes. When shining a light in the patient's left eye, dilation occurs in both pupils. Which of the following terms is BEST correlated to this patient presentation? Select one: a. Nystagmus b. Marcus Gunn pupil c. Internuclear ophthalmoplegia d. Diplopia

marcus gunn pupil: typical in patients with MS/optic neuritis. a light shone in non-affected eye produces a reflex contraction in both eyes, but a light shone in the affected eye brings B/L dilation of the pupils internuclear ophthalmoplegia: incomplete eye adduction of the affected side and nystagmus of the opposite abducting eye

trochlear nerve damage-visual s&sx

may see ocular torticollis - diplopia - patient will compensate by tilted head anteriorly and laterally towards the side of the normal eye

glide to increase ankle eversion

medial glide

- L hemiparesis of face, UE, LE - medial strabismus of R eye - nystagmus which syndrome is this?

medial inferior pontine syndrome

T1 sensation

medial sign of antecubital fossa and contraction of finger abd

A 32 year old male patient arrived to an outpatient clinic with right heel pain during ambulation. Upon examination, the Physical Therapist observes the patient's calcaneus in eversion after heel strike. The Physical Therapist would like to utilize an orthotic that could help alleviate the patient's pain. Which of the following orthotics would MOST LIKELY benefit this patient) 1) medial wedge (rearfoot varus post) 2) lateral wedge (rearfoot valgus post) 3) cushion heel 4) forefoot lateral wedge

medial wedge (rearfoot varus) everted calcaneus= slight pes valgus a rearfoot valgus post (lateral wedge) prevents the calcaneus from going into inversion

wrist flexion reduces tension on which 2 nerves

median ulnar (will be stressed by elbow flexion)

tracheal/mediastinal shifts due to asymmetrical intrathoracic pressure in lungs:

mediastinum shifts away from affected side hemothorax: trachea shifts away bc pressure on side of hemothorax increases

A patient who has been on bed rest for three weeks has developed a plantar flexion contracture. Which phase of the gait cycle would be the MOST problematic for the patient based on the described impairment? 1.Heel strike to foot flat 2.Foot flat to midstance 3.Midstance to heel off 4.Heel off to toe off

mid stance to heel off: mid stance requires 10 deg of DF progressing to 15 at heel off

Which phase of the gait cycle has the maximum torque for the Hamstring muscles?

mid swing bc hammies act eccentrically to control excessive knee extension as the leg moves towards terminal swing

otitis media

middle ear infection

GOLD classification of COPD

mild: FEV 1 >80% moderate: 50-79% severe: 30-49% very severe: <30% of less than 50% of normal and chronic respiratory failure

typical inspiratory mm training guidelines

minimum 4-5 days a week with an intensity at 30% maximal inspiratory pressure measured at functional residual capacity. - two 15-min sessions each training day

subdermal burn

mm damage and neurological involvement with a charred appearance

A physical therapist completes a developmental assessment on an infant. Assuming normal development, which of the following positions would typically be the LAST to occur? 1.Modified plantigrade 2.Quadruped 3.Ring sitting 4.Bridging

modified plantigrade bridging: 5 mo ring sitting: 6 mo quad: 8 mo modified planti: 10-12 mos

what happens if the L optic nerve is damaged?

monocular vision loss

what kind of reflex is a DTR?

monosynaptic does not include spinal tracts or columns

chondrosarcoma

most commonly occurs in adults >40 - palpable mass - back, pelvis, or thigh pin - sciatica - bladder symptoms - unilateral edema

phase 1 (0-4 week) protocol of intac RTC, total shoulder

motion restricted to 120 deg ER restricted to 30 degrees with arm at side NO ACTIVE IR TO PROTECT THE SUBSCAP (6 weeks) no extension past neutral for up to 6 weeks

do you see motor loss, sensory loss, or both lost with injury to posterior interosseous n

motor loss without sensory

calcium or magnesium use

muscle spasm

weak inversion & great toe flexion. loss of sensation sole of foot which nerve?

nerve compression/injury= INC latency, DEC velocity of conduction weak inversion and toe flexion= posterior tibial nerve sensation on sole of foot= posterior tibial nerve post tib nerve travels in tarsal tunnel, posterior to medial malleolus may see increased latency at tarsal tunne

neuropraxia, axonotmesis, neurotmesis

neuropraxia: segmental demyelination, temp sensory sx axonotmesis: loss of axonal continuity but CT coverings remain intact; wallerian degeneration distal to lesion neurotmesis: complete severance of nerve fiber with disruption of CT covering; mm atrophy and sensory loss

A physical therapist is treating a patient with a burn injury on the dorsal aspect of his wrist and hand with a moderate level of edema. Which of the following positioning strategies will be MOST appropriate to avoid the formation of contractures? a. Wrist extension and digit extension b. Neutral wrist with the metacarpophalangeal (MCP) joints in 90 degrees of flexion, and extension of the interphalangeal (IP) joints c. Wrist extension and metacarpophalangeal (MCP) joints extension d. Wrist flexion and digit flexion

neutral wrist with the MCP in 90 deg flexion and extension of IP joint with a burn on the dorsal/post surface, we would predict a wrist extensor contracture wrist extension and digit extension would be contracture prevention for a burn on the volar surface

is high fowler recommended after transtibial amp?

no HOB 80-90 deg with knee flexed can inc risk of contracture -ideal: residual limb supported by pillows and knee resting in full extension

motions to avoid after THA with reattachment of the greater troch

no active hip ABD in s/l with knee extended - no resisted ABD and no isotonic hip ABD against gravity until osteotomy has healed - hip ext to neutral is ok

secondary risk factors for atherosclerosis

obesity stress activity level

Transient synovitis

occurs in younger children - antalgic gait - pain aggravated by IR and ABD

L-dopa side effects

off phase, dizziness, involuntary movements, dyskinesias, arrhythmias, hallucination, orthostatic hypotension

polymyalgia rheumatica

often affects shoulder and pelvis difficulty with ADL's (including STS, rolling in bed) different from myofascial pain syndrome bc that will present with trigger points

Anterior displacement of elbow

olecranon displaced anteriorly

where do you put your hands to promote expansion of the lingula?

on the L side of the chest below the axilla lingula= segment of L upper lobe

post ACL allograft reconstruction

open chain terminal knee extension not advised in early ACL repair no isometric quad contraction at 15 deg

shoulder loose packed and resting

open pack 55 deg abd resting 55 deg abd and horizontal add 30 deg

greatest laxity of knee is in what position?

open packed 25 def knee flexion

coxa vara

over pronation medial rotation short ipsilateral leg= anterior pelvic tilt genu valgum, tibial IR

A PT examines a patient who complains of foot pain while running. The examination shows that the patient has excessive foot pronation. Which of the following would be the MOST appropriate orthotic insert? A. A lateral forefoot wedge under the fifth metatarsal head B. A lateral rear-foot wedge under the calcaneus placing it in an everted position C. A medial arch wedge just beneath the head of the talus D. A medial wedge just proximal to the first metatarsal head

over-pronation= flat feet= flattening of medial longitudinal arch pronation= EDAB: eversion DF ABD of forefoot = medial wedge just proximal to 1st MT head

Osgood-Schlatter disease

overuse injury anterior knee pain 9-15 years, physically active ache in ant knee and clear tenderness of apophysitis mm contraction produces pain application of strengthening should not provoke sx

biceps tendinopathy

pain in anterior arm to elbow pain with jar opening (supination) repetitive stress

Tibial and common peroneal artery occlusion

pain in calf and feet

femoral and popliteal artery occlusion

pain in calf and feet

sesamoiditis

pain on WB and swelling of soft plantar tissue - pain is exacerbated when the sesamoids are palpated while passively DF the MTP joint

Morton's neuroma symptoms

pain on sole of foot that increases with weight bearing or great toe hyperextension full, pain-free AROM with intact strength pain when the metatarsal heads are squeezed together (+ morton's) The foot may be pronated with low arches with pain present in the web spaces of the toes

SIJD pain

pain that is aggravated by prolonged standing, asymmetrical weight bearing, or stair climbing - will most likely report pain with walking

iliopectineal bursitis

pain with resisted hip flex and passive hip ext

T4 syndrome

paresthesias, numbness or UE pain associated with or without headaches and back stiffness

A patient post traumatic brain injury is presently at the confused-appropriate level of cognitive functioning. The patient has progressed well in therapy, however, has been bothered by diplopia. Which of the following treatment strategies would be the MOST appropriate to address diplopia? 1.Provide non-verbal instructions within the patient's direct line of sight 2.Place a patch over one of the patient's eyes 3.Ask the patient to turn their head to one side when experiencing diplopia 4.Instruct the patient to carefully focus on a single object

patch with strengthening ex of extraocular mm

name levels for these reflexes: - patellar - achilles - medial hamstrings - lateral hamstrings

patellar L3-4 achilles: S1-S2 medial hamstring: L5-S1 lateral hamstrings: S1-S2

signs and sx of addison's

patient with Addison disease will have low levels of cortisol released from the adrenal gland (aka primary adrenal insufficiency). Addison's disease is caused by an autoimmune response which attacks the outer cortex of the adrenal gland. Some signs and symptoms of Addison's disease include: abdominal pain, abnormal menstrual periods, dehydration, depression, diarrhea, dizziness, loss of appetite, hypoglycemia, hypotension, muscle weakness, nausea, patches of dark skin around scars/skin fold and joints, sensitivity to cold, unexplained weight loss, weight loss, worsening fatigue

BP in patients with ESRD

patients with end stage renal disease have decreased diastolic BP increased BP is a cause for renal failure

A 55-year-old female patient is being seen by a PT 5 days status post unilateral mastectomy. The PT notes that the patient is unable to actively adduct the shoulder in the horizontal plane. Which of the following impairments is the MOST LIKELY origin of the patient's presentation?

pec major weakness pec major and minor= horizontal ADD pecs may have been damaged at time of mastectomy

common causes of pedal edema

pedal edema= venous fluid or lymphatic fluid in feet/lower legs - chronic wounds - inflammation - infection - cellulitis - DM - liver or renal disease - chronic venous insufficiency - phlebolymphedema (due to CVI and lymphatic insufficiency) - CHF - trauma

describe sway back posture

pelvis shifts posteriorly - PPT - hips and knees hyperextended - flat back - favored leg longer in standing - neck flexors elongated

unilateral vestibular hypofunction

peripheral, U/L- could be due to virus, trauma, vascular event - dizziness or vertigo - poor balance, esp with head turns - blurred vision, especially when turning head quickly - nausea - trouble walking, esp. outdoors, in dark rooms, or in crowded places

An acute care Physical Therapist performs a full pulmonary assessment and examination of a newly admitted patient who is suffering from pneumonia. The following examination findings are presented: frictional rub, tachycardia, jugular venous distention, bilateral edema graded at 2+, decreased breath sounds and decreased fremitus. Additionally, the patient reports shortness of breath during all ADLs, and a sharp "stabbing" pain that increases when he coughs. These findings are most consistent with? 1) pneumothorax 2) pleural effusion 3) pleural fibrosis 4) atelectasis

pleural effusion

absent breath sounds indicates

pneumothorax

which of the following can cause elevated BP? A. Anemia B. Thrombocytopenia C. Leukopenia D. Polycythemia

polycythemia Polycythemia is characterized by increases in both the number of red blood cells and the concentration of hemoglobin. People with polycythemia have increased whole blood viscosity and increased blood volume which can cause elevated blood pressure

Primary OA vs Secondary OA

primary: absence of specific etiology secondary: predisposing condition that accelerates the degenerative process

sx of gout

primary: acute monoarticular arthritis The individual will report being awakened from sleep secondary to intense pain in the affected joint; any pressure (even the touch of clothes or bed sheets) on the joint is intolerable. Redness and swelling may occur within a few hours and can be accompanied by low-grade fever and chills. The peripheral joints of the hands and feet are involved (90% attacks in the metatarsophalangeal joint of the great toe.) Other typical sites of initial involvement (in order of frequency) are the ankle, heel, knee, and wrist, although any joint in the body may be involved.

acupuncture TENS

produces a muscle twitch LOW pulse frequency (2-4 pps) SHORT pulse duration (100-300) 20-45 mins for analgesic effects

A 23-year-old female underwent ACL reconstruction on her left knee about 21 days ago and has been receiving treatment at an outpatient PT clinic for the same. Initially, the patient was on partial weight bearing (50lb) status and has now progressed to full-weight bearing status. At this time, the therapist should MOST likely emphasize on which of the following? Select one: a. Progressing to SLR with assistance as needed. b. Progress to performing mini squats. c. Progress to performing patellar mobilizations (Grade I). d. Progressing to walking weight bearing as tolerated with the use of crutches.

progress to performing mini squats to progress to closed chain activities for strengthening (during first 2 weeks: SLR/patellar mobs/prog from PWB to WBAT)

Hornblower's Sign

pt sitting. PT elevates arm to 90 in scapular plane and flexes elbow to 90. pt is asked to laterally rotate against resistance - rotator cuff TERES MINOR

why does a medial whip occur?

prosthetic foot is laterally outset excessive lateral rotation of the knee joint

Following a stroke, a PT is giving a 62-year-old patient education about positioning in bed. Which is the BEST position to place the lower extremity when lying in supine?

protracted pelvis hip slightly abducted and flexed knee on small towel roll ankle in neutral with nothing against soles of feet

alginate dressings

provide hemostasis and can be used over an infected wound

hydrogel dressings

provide hydration to dry wound beds - used for GRANULAR, not draining

A patient reports that they were lifting heavy weights in the gym a few days ago and have new lower abdominal pain. They have noticed that the pain is worse when lying on the left side. The pain subsides when awakening, yet will gradually increase throughout the day. What type of pain should the Physical Therapist determine is occurring? 1) common bile duct pain 2) pseudorenal pain 3) gall bladder pain 4) liver pain

pseudo renal pain: acute, usually associated with a traumatic hx such as lifting a heavy object. pt will report lower abdominal pain that is worse at night, esp. when laying on affected side. pain will get worse throughout day liver pain: RUQ pain. consistent and will not change throughout day common bile duct pain: mid-epigastic pain with vague discomfort. pain is constant- starts mild and increases steadily gall bladder pain: RUQ pain, may refer to R shoulder. dull aching and sense of fullness in abdomen or epigastric region

e-stim parameters for mm strengthenng

pulse dur 300 microsec pulse freq 35-80 on:off 1:5 originally

wheelchair modifications for patient with B/L transfemoral amputation

rear wheels 2 in more posterior for wider BOS & compensate for loss of weight of LE's

compensation for forefoot varus

rearfoot pronation

excessive foot pronation compensates for?

rearfoot varus

Blumberg sign

rebound tenderness press finger on RLQ for 15-20 secs then remove quickly. if the pain is increased by a quick withdrawal of the fingers then the test is (+)

UE flexion synergy

scap retraction & elevation shoulder flexion, abd, ER elbow flexion, wrist and finger flexion supination

deep partial thickness

red to white burn with blisters, and painful slow capillary refill sensitive to pressure & insensitive to light touch or pin-prick

A 42 year old patient with left congestive heart failure reports having new onset of visual disturbances and a headache. The Physical Therapists examines the patient and notices that the patient has decreased deep tendon reflexes and ventricular fibrillation. Which of the following is the MOST likely diagnosis for this patient? 1. Metabolic Acidosis 2. Respiratory Acidosis 3. Metabolic Alkalosis 4. Respiratory Alkalosis

respiratory acidosis can be caused by chest trauma, L CHF, airway obstruction, and COPD. signs and sx: visual disturbances, confusion, dizziness, DEC deep tendon reflexes, v fib

volar plate injury of PIP

result from PIP extension force - excessive passive PIP ext with empty end feel

pulmonary fibrosis

reticular or net like pattern due to destruction and fibrosis of the lung tissue characterized by: dry hacking cough, fatigue, muscle weakness, SOB, weight loss due to loss of appetite

A physical therapist observes an infant exhibiting a high guard position when sitting. Which of the following muscles would be the MOST essential for the infant to maintain this position? 1.Pectoralis major 2.Rhomboids 3.Serratus anterior 4.Lower trapezius

rhomboids high guard= arms being held near shoulder level with retraction of scap to increase midline trunk stability against the pull of gravity

mm that downwardly rotate scap

rhomboids levator scap pec minor

scapular retraction in prone will strengthen:

rhomboids, middle traps, posterior deltoid

A 30 year old patient diagnosed with possible nerve injury is being seen for an evaluation. On examination the Physical Therapist observes a loss of scapular upward rotation which results in scapular winging. Additionally, the patient has difficulty with resisted motion of glenohumeral abduction. What is the possible mechanism that could result in the inability to lift the shoulder beyond horizontal?

scapular winging often occurs 2/2 weak serratus anterior weak serratus creates lack of stability in scapula= deltoid acts as a downward rotator of a scap as it is unable to abduct the humerus against resistance - weakness in SA: downward rotation shoulder flex/abd: deltoid and supraspin these mm will have active insufficiency the delt will no longer have a firm scapular base secondary to weak serratus, which will pull down the entire scap as the GH joint is abducted

iodine use

sclerotic scars

osteoid osteoma

seen in males under 25 years - pain is worse at night - increased skin temp, sweating, and tenderness in the affected region - pain may be completely relieved by aspirin (salicylate) *** hallmark

chiari II malformation

seizure stridor apnea dysphagia ataxia

A patient with patellofemoral syndrome discusses their past medical history with a physical therapist. The patient reports having anterior cruciate ligament reconstruction surgery on their right knee two years ago, however, the therapist is not able to identify a scar over the anterior surface of the right knee. Assuming the surgeon utilized an autograft for the reconstruction, which of the following tendons would be the MOST likely graft site?

semitendinosus and gracilis

treatment for cupulolithiasis & canal

semont- cup epley- canal

calcific tendinopathy

severe subacromial lateral shoulder pain with more sudden onset in middle aged people

herpes zoster

shingles: painful blisters/rash that occur in a dermatomal pattern

ER with elastic resistance will strengthen which mm?

shoulder ER: infraspin, teres minor

A physical therapist treats a patient with a colostomy that is capable of producing solid stool on a fairly regular schedule. Which type of colostomy would be the MOST consistent with this description? 1.Ascending 2.Descending 3.Sigmoid 4.Transverse

sigmoid - final portion of the large intestine and serves as connection to rectum. most common type of colostomy- located a few inches lower than descending. has an additional working colon and will produce normal stool consistency on a regular schedule ascending colon: R side of abdomen- output is primarily liquid with many digestive enzymes transverse: upper portion of abdomen (ends with.a band in the colon called the splenic flexure)- may produce soft or loose stool at infrequent intervals descending colon: L side of abdomen, large portion of colon is active so output is often solid but tends to be irregular

psoriasis

silver and scaly plaques on scalps, elbows, knees - can result in erosive arthritis, particularly in the DIP

A physical therapist treats a patient who sustained deep partial-thickness burns to the anterior surface of both lower extremities. After identifying an irregularity in the patient's laboratory results, the referring physician discusses with the therapist the possibility of discontinuing the use of the topical medication silver sulfadiazine. Which of the following findings is MOST likely related to the use of silver sulfadiazine? 1.Leukopenia 2.Peripheral edema 3.Hypokalemia 4.Altered pH balance

silver sulfadiazine: topical ABx that works by interfering with bacterial nucleic acid production by disrupting folic acid synthesis in susceptible bacteria. side effect: leukopenia (dec in WBC) * mafenide acetate would alter pH

signs and symptoms in hypothyroidism

skin dryness/roughness, hair loss, constipation, depression, joint stiffness, sensitivity to cold, slow heart rate, swelling in extremities, weight gain.

A 55-year-old male suffered a left CVA 6 months ago and is being treated by a physical therapist to improve his gait and balance. After using a hot pack on the ankle, the therapist glides the distal tibia anteriorly on the talus with the patient in the standing position. Which of the following gait deviations would have led the therapist to perform this intervention? a. No heel off in terminal stance b. Excessive dorsiflexion in mid-stance c. Early heel rise in mid-stance d. Excessive dorsiflexion on heel strike

sliding the distal tibia anteriorly on the talus in standing = glide to inc ankle DF in the closed kinetic chain perform 2/2 early heel rise in mid stance : caused by inadequate DF or PF contracture C

most common cause of internal snapping

slipping of the iliopsoas tendon over the osseous ridge of the lesser trochanter or anterior acetabulum. occurs at 45 deg f flex when the hip is moving from flex to ext, esp with hip in ABD and ER

A patient with increased sympathetic output is examined in physical therapy. Which of the following treatment techniques would be the LEAST beneficial in decreasing the level of sympathetic activity? 1.Connective tissue massage 2.Rotating the lower trunk in hooklying 3.Slow reversal hold of the quadriceps and hamstrings 4.Gentle manual pressure to the abdomen

slow reversal hold of quads and hamstrings is a PNF technique to improve stability

a patient with a meniscus tear would most likely experience which type of end feel?

springy- rebound effect with tissue stretch indicating an internal derangement within the joint synovitis: would see boggy end feel with restrictive ROM (indicating increased tissue swelling and swelling within the capsule)

how do you stretch the R scalene

stabilize R clavicle and 1st rib extend lower cervical spine R rotation, L sidebending

stage I vs stage II lipedema

stage 1: spongy feeling of the subcutaneous tissue with soft skin. You may feel nodular changes with palpation but there will be no color changes stage 2: increased nodular and tough subcutaneous tissue, large fatty lobules (at medial thigh and medial and lateral ankles above the malleoli), skin color changes of the lower leg, and pitting edema that increases as the day progresses

what stage of lymphedema do you see brawny edema in

stage 2: spontaneously irreversible - swelling that feels hard with palpation

A physical therapist administers the Functional Reach Test to a patient who has a neurological disorder. What bony landmark would be the MOST appropriate for the therapist to utilize when measuring the distance the patient reached during each trial?

stand upright with static BOS and make a fist/raise arm to 90 deg lean forward and measure: subtract beginning position from ending measure position of the 3rd metacarpal on a yardstick after making a fist

correct measurement for seat height in adults, hemiplegics, children

standard adult: 20 inches hemiplegic: 17.5 in (easier to propel with legs) children: 18.75 in

fwd neck: where is the stiffness? - what do you strengthen? - what do you stretch?

stiffness of R occipitoatlantal joint - deep neck flexors are overstretched so you want to strengthen them - stretch: posterior cervical extensors, upper traps

excessive IR of shoulder & winging of scap

strengthen serratus, stretch pec

Ruffini endings

stretch and joint position located deeper in dermis

rounded shoulders and fwd head: which mm should you strengthen/stretch

stretch cervical extensors and pec major strengthen scap retraction and cervical retraction

Speed's Test

stretching or lengthening of the biceps tendon to assess the possibility of tenosynovitis shoulder flexion, ER, full elbow extension, supination

brief intense TENS

strong paresthesia or motor response HIGH pulse frequency (60-200) HIGH pulse duration (150-500) 15 mins in wound debridement

A physical therapist assesses a patient's upper extremity deep tendon reflexes as part of a screening examination. Which of the following locations is the MOST appropriate to elicit the brachioradialis reflex? 1.Radial tuberosity 2.Antecubital fossa 3.Biceps tendon 4.Styloid process of the radius PreviousNextMark

styloid process of radius

Erisypelas

superficial cellulitis secondary to GAS, significant dermal lymphatic involvement. often on face or legs *penicillin* raised, sharp demarcation of borders with an unmistakable bright red discoloration

moderate edema with very few scaring, inflamed dermis, capillary refill, blisters intact but very painful and sensitive to temperature changes. what type of burn is this?

superficial partial thickness

impetigo

superficial skin infection associated with inflammation, itching, and pus filled vesicles

A 25-year-old patient had a traumatic fall off his bicycle 1 week ago. Since the injury, the patient has not been able to elevate his scapula. Based on the information given, what is the MOST likely physical therapy diagnosis?

superior dislocation of the sternoclavicular joint during elevation of the scap, the clavicle slides inferior and it slides superiorly during depression

O'Brien Test

superior labral tear or proximal bicep involvement 90 deg shoulder flex, 10-15 deg horiz add, IR, resistance

ACA supply

superior surfaces of frontal and parietal, and the medial surfaces of the cerebral hemi, which control the motor and somesthetic cortex

Traction positions for intervertebral joints, facet joints, muscle elongation

supine with pillow under knee

PD position for: R middle

supine, LE raised 12 in

PD position for: anterior seg

supine, LE raised 18

how to bias obturator nerve during the slump test

supine, slump, hip flexion 90 deg, ABD, knee ext, ankle DF

Piano key test

support the wrist in pronation and apply force to head of ulna (+) ulna goes back to anatomical position after removal of force used as an indicator for distal radio-ulnar joint instability and tears of the triangular fibrocartilage complex of the wrist

Which of the following humeral fractures must be examined MOST immediately for neurovascular status? 1) supracondylar fx 2) lateral epicondyle fx 3) medial epicondyle fx 4) greater tuberosity fx

supracondylar: high # of neurological and vascular structures pass thru this region. - typical radial nerve involvement - may lead to Volkmann's ischemia - assess growth plate in children bc high incidence of malunion

which mm insert on the greater tubercle of the humerus?

supraspin infraspin teres minor (NOT SUBSCAP: inserts on the lesser tubercle)

FDS paralysis causes which deformity:

swan neck (PIP hyperext, slight DIP flex)

A physical therapist is performing a postural assessment on a patient who presented with hips and knees hyperextension, neutral pelvic tilt, elongated neck flexors and one-joint hip flexors, favored leg appearing longer in standing. Which of the following MOST accurately describes this type of posture?

swayback

swiping upward with thumb joint mob

swiping upwards= thumb extension DO A RADIAL GLIDE

A mother reports that her six month old infant girl does not seem to be able to prop herself on all fours and maintain that position. Which reflex should the Physical Therapist suspect is interfering with crawling and how should the Physical Therapist stimulate this particular reflex?

symmetrical tonic neck reflex - flexion of head produces flexion of UE and extension of LE STNR normally integrates in 8-12 months and can interfere with quadruped, creeping, and crawling if it persists

An 11-month-old child who has cerebral palsy attempts to maintain a quadruped position. Which of the following reflexes would MOST likely interfere with this activity if it was not integrated? 1.Galant 2.Symmetrical tonic neck 3.Plantar grasp 4.Positive support

symmetrical tonic neck: when neck is flexed, UE flex and LE extend

Reiter

systemic disease that causes pain in multiple joints - usually asymmetric, occurs after an infection, and presents over several weeks

RA symptoms

systemic: - weight loss - fever - fatigue hallmark: morning stiffness >1 hour bilateral and symmetric synovial joint involvement - limited mobility, inflammation commonly affected: cervical neck, hands, feet can also involve TMJ (inflammation of the TMJ results in pain, swelling, and limited movement with eventual ankylosis)

A patient with a past medical history of rheumatoid arthritis has complaints of right shoulder pain. The patient reports her shoulder has gradually become more stiff over the last year and reports an average pain level of 1/10. The patient reports difficulty washing her hair, but that it is easier to put her phone in her back pocket. Which of the following interventions is MOST APPROPRIATE? a. Aggressive stretching of the right bicep to improve elbow extension range of motion b. A grade I right glenohumeral distraction to decrease pain c. Rotator cuff strengthening exercises to improve shoulder stability d. A grade III posterior glenohumeral joint mobilization to improve glenohumeral external rotation

these are signs and sx consistent with adhesive cap. hx of RA inc risk of this with adhesive cap, always wanna do a posterolateral glide D

tx if you see a step-off deformity

this is a type 3 deformity - immobilized 3 weeks - return to activity 6 weeks * will have good structural strength at 3 weeks but will require at least 6 to return to activity

Scheuermann's Disease

typically affects T7-T10 - insidious mid to low thoracic back pain - worse with prolonged standing and sitting - active rotation in sitting is painful - excessive thoracic kyphosis and lumbar lordosis

arterial wounds

typically located at dorsum of foot, lateral leg, toes - minimal edema initially & typically only present in a dependent position - minimal exudate but severe pain - wounds can be deep and involve tendon and bone

radial or ulnar drift with RA?

ulnar drift - perform cervical stab, NO mobs or cervical traction

A 35 year old female with reports of tingling and weakness in the right upper extremity (UE) tested positive on the upper limb neurodynamic (tension) test (ULNT) with the following position: shoulder abduction to 90 degrees, elbow flexed, forearm supinated, and wrist extended with radial deviation. Which of the following is LEAST likely to be associated with the patient's clinical presentation at the wrist and hand? Select one: a. Decreased strength of thumb opposition motion b. Atrophy of the hypothenar eminence c. Weakness with adduction of the 1st digit d. Sensory impairments in the 5th digit and half of the 4th digit

ulnar nerve involvement dec thumb opposition strength is more median nerve

closed pack positions for: - ulnohumeral - radiohumeral -proximal (superior) radioulnar

ulnohumeral: full extension with supination radiohumeral: 90 deg flexion, 5 deg supination proximal radioulnar: 5 deg supination

A 55-year-old male patient arrived at a hospital after having multiple falls at home. Upon evaluation, a PET scan revealed decreased levels of dopamine with degenerative changes at the substantia nigra. The patient is at risk of MOST likely developing which of the following types of incontinence? Select one: a. Stress b. Mixed incontinence c. Overflow d. Urge

urge UI "overactive bladder" is the type of incontinence that is expected to be seen in patients with Parkinson's Disease. Urge incontinence appears more frequently in males and in Parkinson's disease. Urge incontinence is caused by impaired neurological signals from the brain to the bladder. Additional neurological conditions you would also expect to have urge incontinence are spinal cord injuries, multiple sclerosis, etc

medial wedge (rearfoot varus post)

used for calcaneus eversion and pes valgus

lateral wedge (rearfoot valgus post)

used for calcaneus inversion

compression therapy indications and contraindications

used for prevention and treatment of venous leg ulcers - papillomas and hyperkeratosis are common dermal abnormalities and do not prohibit the use of multi-layered compression bandaging * CONTRAINDICATED in the presence of PAD (wounds that are deep, regularly shaped, and dry)

digoxin desired effects

used in CHF to increase the force/strength of contractility of the ventricles of the heart - increases SV - DEC QRS complex and QT shortens (dec duration of contraction of ventricles) - causes HYPOkalemia (decreases Na+/K+ pump- reduces HR) - PROLONGED PR

when are symmetrical biphasics used

used primarily in neuromuscular stim for functional support or mm strengthening

craig scott orthosis

used with paraplegia & each orthosis includes either a shoe reinforced with transverse and longitudinal plates and BiCAAL ankle joints set in slight DF, pretibial band, pawl knee lock with bail release, and single thighband

thromboangiits obliterans (buerger disease)

vasculitis affecting peripheral blood vessels clinical manifestations are caused by occlusion of arteries, reduced blood flow, and subsequent reduced oxygenation. - sx occur from chronic ischemia: cold sensitivity, rubor (redness from dilated capillaries under skin), trophic changes (thin, shiny, hairless skin)

what do the vertebral arteries supply? common presentation?

vertebral (basilar) a. supply brainstem and cerebellum lesion will manifest and U/L or B/L weakness - loss of vibratory sense, 2pt discrim, and position sense (DCML) - diplopia, homonymous hemianopia - dysphagia - dysarthria - nausea - confusion

torsional/rotational nystagmus

vertical canals: anterior and posterior

CNS disorder

vertical nystagmus: pure, vertical pendular abnormal saccades, pursuits, severe ataxia, usually no hearing loss, diplopia, altered consciousness

A physical therapist instructs a patient to expire maximally after taking a maximal inspiration. The therapist can use these instructions to measure which of the following lung volumes? 1.Expiratory reserve volume 2.Inspiratory reserve volume 3.Total lung capacity 4.Vital capacity

vital capacity

hyperthyroidism with discoloration of skin (patches of light skin caused by loss of epidermal melanocytes). What is this called?

vitiligo

coordination synkinesis

voluntary contraction of certain muscle groups on involved side that, in turn, gives rise to involuntary contractions of synergistic muscles.

A physical therapist applies an automated external defibrillator (AED) to a patient in cardiac arrest. In addition to ventricular fibrillation, what condition is MOST likely to be identified and treated with the AED? 1.Atrial fibrillation 2.Premature atrial contractions 3.Ventricular tachycardia 4.First degree ventricular heart block

vtach (3+ PVC's at ventricular rate of >150 bpm) longer than 30 sec= life threatening

A patient has been in an inpatient rehabilitation unit for the last three weeks secondary to a motor vehicle accident. The physical therapist reviews the patient's medical chart and notices that the patient currently has active Tuberculosis. Which of the following isolation precautions is LEAST beneficial in this patient scenario? Select one: a. Washing hands when entering and leaving the room b. Patient uses a private room with positive airflow c. Use of an N-95 mask by the patient d. Keep the patient's room door closed

want a room with NEGATIVE airflow

ion for hyperhydrosis

water

low lateral wall

weak ABD

why would you see a lateral bend in stance? (anatomic)

weak ABD (same side) short amputation limb

A patient is placed in the supine position with the hips flexed to 90 degrees and the knees extended. As the patient slowly lowers the extended legs toward the horizontal, there is an increase in the lumbar lordosis. This finding is indicative of weakness of what muscle group?

weak abdominals failure to maintain the low back flat against the table is indicative of abdominal weakness

involvement of C6 myotome:

weak elbow flexors, and wrist extensors

backward lean in stance phase

weak hip extensors on that side!!

causes of posterior trunk bend in stance

weak ipsilateral glut max or contralateral weak hip flexor

A patient was brought into the emergency department with shortness of breath and a past medical history of atelectasis. The patient currently has severe right-sided chest pain, sharp pain with inhalation, rapid heart rate, cyanosis of the skin and lips, and decreased breath sounds. The patient is suspected to be suffering from a pneumothorax. Which of the following tracheal pathological changes would be observed in a large pneumothorax versus in a massive atelectasis?

with atelectasis= lung collapse on that side= the trachea will be deviated to the SAME side trachea goes to side of low pressure a pneumothorax pushes the trachea to the opposite side, atelectasis pulls trachea towards same side

what happens to the following values with kidney failure: - creatinine - urine output - urea levels - BUN

with kidney failure, you see - increased creatinine - increased urea - increased BUN - decreased urine output

posterior glide at wrist improves:

wrist flexion

preferred imaging for stenosis and disc hern

MRI

gold standard to diagnose ACL (imaging)

MRI: 90% diagnostic accuracy

dexamethasone use

MSK inflamm

Signs/sx of hypoglycemia

- pallor - diaphoresis - tachy - weakness - shaking - blurred vision - shallow respiration

A pulmonologist is performing a pulmonary function test on a 40-year-old male patient. The patient reports having a productive cough for a few months every year that seems to be getting worse each year. There are no signs of allergies or infection and related tests are negative. What are the changes in the PFT that are NOT expected in this patient? 1. Decrease in Total Lung Capacity 2. Decreased expiratory reserve volume 3. Increase in Residual Volume 4. Decreased FEV1/FVC

#1

A physical therapist reads in the medical record that the foot progression angle of a four-year-old child was recorded as -10 degrees (minus 10 degrees). Which range of motion measurement at the hip would MOST likely be associated with the obtained foot progression angle? 1.75 degrees of hip medial rotation and 25 degrees of hip lateral rotation 2.35 degrees of hip medial rotation and 70 degrees of hip lateral rotation 3.30 degrees of hip medial rotation and 20 degrees of hip lateral rotation 4.45 degrees of hip medial rotation and 45 degrees of hip lateral rotation

(-) number is in toeing (+) number is out toeing -10: anteversion: more IR (75), less ER (25)

A physical therapist is educating a patient on the functionality of a stress echo to compare left ventricular (LV) function and wall motion. With regards to the functional activity, which of the following is LEAST appropriate in terms of results from a stress echo? Select one: a. Increase VO2 resulting in increase in MVO2 b. Positive stress indicates that the LV is worsening as activity increases c. Negative stress indicates that the LV has easily adapted with an increase in energy demand d. Negative stress indicates that the LV has easily adapted with decrease in energy demand

(-) stress indicates that the LV has easily adapted to the INC in demand not decrease D

sx of metabolic acidosis

(can be caused by DKA, renal failure, shock, salicylate OD, and sepsis) - headache - mental dullness - Kussmaul breathing - stupor - cardiac arrhythmia

signs and sx of respiratory alkalosis

(can be caused by high altitude, pregnancy, fevers, hypoxia, or inc tidal volume in vented patients) - lightheadedness - numb digits - tetany - convulsions - cardiac arrhythmias

what can cause delayed knee flexion with transtibial prosthesis

- DF stop too stiff - excessive PF - socket is too far posterior - keel too long - low shoe heel

UE vs LE exercise

- HR higher with UE - SV lower with UE - SBP higher iwth UE - RPP higher with UE bc HR and SBP higher * SV is higher with LE

parameters for e-stim (HVPC) with infected wounds - electrode - pulse frequency - pulse duration - amplitude - treatment time

- NEG polarity pulse frequency 60-125 (double from e-stim) pulse dur 40-100 preset amplitude to produce comfortable tingling treatment time 45-60 mins

contraindications of thermotherapy

- active MSK trauma - arterial disease - bleeding or hemorrhage - over an area of compromised circulation - over an area of malignancy - peripheral vascular disease - thrombophlebitis

BPH sx

- age over 50, enlargement of prostate - lower abdominal, low back, or thigh pain - inc urinary urge/voids - hesitancy of urination - small amounts of urine when voiding - dribbling - nocturia

SCFE

- antalgic gait - pain in groin, knee, medial thigh - LE in ER - acute onset: pt will be unable to WB on affected extremity - restricted ABD and IR - obesity is factor in development

which ligaments comprise the deltoid:

- anterior tibiotalar - posterior tibiotalar - tibiocalcaneal - tibionavicular

what 3 tracts make up the spinothalamic pathway:

- anterolateral (ventral): crude touch - lateral spinothalamic: pain and temperature - spinoreticular: diffused pain sensation

recommendations for appropriate footwear and skin care

- cut toenails straight across and not into corners - ensure shoes are correct size and width - don't walk around barefoot in the house - shop for shoes in afternoon bc feet are largest at that time - may use lamb's wool for padding and circulation but avoid cotton bc the fibers may irritate the skin

Clinical findings associated with a lesion at the 4th lumbar nerve root:

- dec DF strength - hypoesthesia of outer lower leg and great toe - pain in lumbar area, inner buttock, outer thigh and leg - diminished tib post, patellar, and tib ant mm stretch reflexes

things that contribute to adverse rxns of meds in aging adults

- dec in total body water - dec hepatic blood flow - dec in lean body mass - inc in proportion of body fat

full immersion hydrotherapy

- decrease BP, decrease HR - increased cardiac output - increased cardiac volume - vital capacity decreases as the lung expansion is inhibited due to hydrostatic pressure against the chest wall

criteria to terminate exercise in patients with heart failure

- decrease of HR more than 10 bpm - CVP increase by 10 mmHg - RR >40 - fall in BP >10 mmHg - appearance of S3 heart sound - increase in pulmonary crackles in more than half of lung - marked fatigue, dyspnea, diaphoresis, pallor

what would you expect to see on a PFT with an asthma exacerbation

- decreased FEV1 - decreased PEF - decreased IRV - decreased VC - increased RV - increased FRC

R hemisphere lesion

- difficulty synthesizing info - difficulty grasping whole idea of task - difficulty expressing negative emotions - L sided unilateral neglect - agnosias - quick, impulsive behavior - poor judgment

TMJ: differentially diagnose 1) hypomobility 2) synovitis 3) capsulitis 4) disc displacement with reduction

- hypomobility: dec mouth opening with no pain - synovitis: dec mouth opening with pain, no clicking, no deviation - disc displacement with reduction: clicking - capsulitis: dec mouth opening, pain, no clicking, will have deviation

describe where the patient would weight bear on their foot during these phases: - increased stance phase - excessive toe off during swing - if pt has dec DF

- increased stance: pt would spend inc time WB on the side of their shoe - excessive toe off during swing: pt would spend inc time WB on the front portion of the shoe - dec DF: pt will have poor clearance and toe drag, so will see increased wear to the anterior portion of the toe box

As per OSHA, what MUST the MSDS (material safety data sheet) contain?

- info about the concentration and exact percentage of each ingredient - description of the sx and effects: acute, delayed, precautions for safe handing - recommendations for storage - minimizing release of chemical into envt - advice on general hygiene practices * cost of material and date of original purchase are not a part

SLAP repair protocol

- limit passive or assisted elevation to 60 deg (2 weeks) and up to 90 deg (3-4 weeks) 0-2 weeks: passive assisted humeral rotation, neutral ER, IR to 45 3-4 weeks: ER up to 30, IR to 60 AVOID TENSION IN BICEPS - no elbow flexion 0-6 weeks - no resisted bicep exercises until 8-12 weeks AVOID abduction + ER (places stress on biceps insertion on to glenoid)

what causes delayed knee flexion?

- low shoe heel - too much PF - long keel - stiff DF bumper - socket too posterior - socket insufficiently flexed

ACSM guidelines for cancer patients

- low to mod exercises for 20-60 mins - resistive exercises at least 1 set of 12-15 reps - focus on ADL training

L4 lumbar root syndrome

- medial lower leg sensory changes - diminished patellar reflex <2+ - weakness in tib ant and ext hallucis - "tripping over feet" - foot drop

lateral cord of brachial plexus

- musculocutaneous - median nerve - lateral pectoral nerve

individual post MI- when should resistance training begin

- must be in cardiac rehab for at least 3 weeks - must be at least 5 weeks post MI

which part of the meniscus has more blood supply- outer or inner?

- outer 1/3 of meniscus is more vascular

signs/sx of supraspinatus tendonitis

- painful arc with overhead activities - tight posterior capsule - excess upward migration of humeral head in the glenoid fossa - tight pec minor will cause the scap to tip anteriorly, which causes the acromion to tip anteriorly and reduces the subacromial space more

e-stim parameters for mm strengthening - pulse frequency - pulse duration - amplitude - on:off - ramp time - treatment time

- pulse freq 35-80 pps ***Only one that goes to 80, the rest are 35-50 pulse duration high: - 150-200 for small - 200-350 for large mm amplitude: >10% MVIC if injured, >50% MVIC if uninjured on:off - start 1:5 (6-10 sec ON, 50-120 sec OFF) ramp time of at least 2 secs 10-20 min treatment time for 10-20 reps perform every 2-3 hours when awake

types of MS

- relapsing remitting: discrete attacks with full or partial recovery - primary progressive: steady functional decline from onset - progressive-relapsing: steady deterioration but with occasional acute attacks. intervals between attacks are continuous - secondary progressive: initially relapsing remitting followed by change in status with progression to steady and irreversible decline with/without decline

ulnar nerve tension test

- scap depression - shoulder ABD - ER - elbow flexion - pronation - wrist and finger ext

expected results of x-rays in a patient with Scheuermann disease1

- schmorl nodules - irregularity of the vertebral end plate - anterior wedging of the vertebral bodies of 5 deg or more in 3 or more adjacent vertebral bodies tend to see inc thoracic kyphosis and lordosis - extension and rotation may be painful inc spinal flexion leads to ANT wedging not post

position for maximal radial nerve stretch

- shoulder depression - shoulder ABD - IR - elbow extension - pronation - wrist flexion sensory: posterior (dorsal) arm and forearm + radial (lateral) hand + front side of hand (first 2.5 fingers)

Slump test

- slump (thoracic and lumbar flexion) - apply overpressure across shoulders - flex neck - add overpressure to neck flexion - DF foot and extend knee if pain with extension: release OP to cervical flexion if no pain: OP to DF

what can cause early knee flexion during late stance?

- soft DF bumper - insufficient PF - keel too short - socket too far anterior - socket excessively flexed

ankylosing spondy diagnostic criteria

- stiffness >30 mins - improvement in back with exercise - waking up 2/2 back pain during the second half of the night only - alt. buttock pain hallmark: limited chest expansion (<2.5- normal is 5) - other: sacroiliitis, am pain, stiffness - chronic inflammatory disease that leads to spinal and chest wall rigidity - dec in TLC, VC, inspiratory mm function

typical OA radiograph findings

- superior loss of joint space - non uniform joint space loss - subchondral sclerosis - osteophytes

systolic vs diastolic heart failure

- systolic: EF reduced bc of compromised contractile function of ventricles - diastolic: EF not reduced bc of compromise in relaxing and filling both have dec SV and CO

You are treating a patient in an outpatient physical therapy office. The patient complains of left ankle pain, starting 6 weeks ago, located on the outside of her foot. The patient reports she injured her ankle while walking on a beam during gymnastics, when her foot twisted after a flip. The patient is able to maintain a single leg stance for 30 seconds on the ground without deviations. The patient is able to perform a bilateral squat with minimal deviations and reports her average pain level is 2/10. Which of the following interventions is the MOST APPROPRIATE? a. Single leg hopping to improve plyometric ability b. Return to run program for safe return to sport c. Single leg squat to 90 degrees to improve functional landing d. Bilateral heel raises to improve gastrocnemius strength

- these signs and sx suggest lateral ankle instability * the patient must have good single leg squatting before progressing to plyo's and return to run * B/L heel raises will not be challenging enough C

which nerves may be damaged with a distal humerus fx

- ulnar nerve (dorsal interossei) - median nerve (FDS, pronator quadratus)

List average developmental timelines for these positions - unsupported sitting - plantigrade - pull to stand thru half-kneeling - independent walking

- unsupported sitting: 5-7 months (full trunk extension, high guard) - plantigrade: 10-12 months - pull to stand: 10-13 months - independent walking: 12 months (10-15 months)

"normal" EMG test

- you may see brief activity after insertion of needle. this is a spontaneous burst lasting less than 300 msec 2/2 needle breaking through membrane of mm fiber - then you will see electrical silence (until the mm is contracting) seeing spontaneous potentials during electrical silence would be abnormal

Galant reflex

-lateral sidebending to same side as side of stimulus -abnormal response: delay in integration results in poor trunk control for sitting; related to scoliosis; delays in independent head movement

Stages of cancer: 0-4

0: carcinoma in situ (premalignant, preinvasive) 1: early stage, localized to primary organ 2: inc risk of regional spread due to tumor size or grade 3: local cancer has spread regionally but may not be scattered widely 4: spread and distributed to distant sites

A Physical Therapist notices that the patient deviates their trunk to the left during the swing phase of the right leg. Which of the following is the MOST LIKELY cause for this deviation? 1. Contracture of the right ankle plantar flexors 2. Shorter limb length of the right leg 3. Spasticity of the right knee flexors 4. Contracture of the left hip flexors

1

A patient presents to the ED post TBI. On reading the patient's notes, the Physical Therapist observes the following vitals: BP 130/90; HR 105 bpm, RR 20 breaths/min, GCS 5/15. Which of the following presentations would be MOST appropriate for this evaluation to be accurate? 1) Eyes open to pain, incomprehensible sounds, no movement of limbs in response to pain 2. Patient opens eyes in response to pain, incomprehensible sounds present, patient extends limb in response to pain 3. Eyes open spontaneously, patient is confused patient flexes limb in response to pain 4. Eyes do not open in response to pain, no verbal response in patient, withdraws limb in response to pain

1

A physical therapist works on weight shifting activities with a patient who is sitting over the edge of a mat table with the feet positioned on the floor. The therapist facilitates an anterior weight shift through the patient's pelvis. What pattern of activity would be required for the patient to maintain an upright posture? 1.Spinal extension due to concentric contraction of the spinal extensors 2.Spinal flexion due to concentric contraction of the spinal flexors 3.Spinal extension due to eccentric contraction of the spinal extensors 4.Spinal flexion due to eccentric contraction of the spinal flexors

1

A physical therapist works with a patient diagnosed with Down syndrome. The therapist determines that the patient has abnormalities in muscular tone consistent with the diagnosed condition. Which of the following techniques would be the MOST beneficial when treating the patient's tone abnormalities? 1.Quick stretch 2.Deep pressure 3.Prolonged icing 4.Neutral warmth

1

A physical therapist would like to minimize the likelihood of a burn when using iontophoresis. Which action would be the MOST consistent with the therapist's objective? 1.Increase the size of the cathode relative to the anode 2.Decrease the space between the electrodes 3.Increase the current intensity 4.Decrease the moisture of the electrodes

1

A 56-year-old male arrived at an outpatient physical therapy clinic with chief complaints of low back pain. Other comorbidities include a history of CHF, HTN. During the initial evaluation, the patient provided a list of medications with one of them being Candesartan. The therapist is considering the implications of the effects of this medication on physical therapy. Which of the following side-effects is MOST likely to be associated with this medication? 1) hypotension 2) hypokalemia 3) inc WBC 4) recurrence of afib

1 candesartan: ARB (angiotensin receptor blocker) side effects - hypotension - dec WBC - inc serum potassium ARB's help to prevent the recurrence of afib

A physical therapist reviews a patient's medical record and identifies that the patient was recently placed on a corticosteroid medication. Which of the following conditions would MOST warrant the use of this type of pharmacological agent? 1.Dermatitis 2.Folliculitis 3.Melanoma 4.Rosacea

1 corticosteroid: hormonal, anti-inflamm and metabolic effects dermatitis tx: cortico, antihistamine, immunomodulators folliculitis: inflamm of hair follicles- antiseptic cleaners or Abx rosacea: laser, vitamin A, ABx

A patient sustained a grade II strain to the iliopsoas muscle. Which of the following phases of the gait cycle should the physical therapist expect to be the MOST impacted by this injury? 1.Toe off (pre-swing) and acceleration (initial swing) 2.Heel strike (initial contact) and acceleration (initial swing) 3.Foot flat (loading response) and deceleration (terminal swing) 4.Midstance and deceleration (terminal swing)

1 iliopsoas contracts eccentrically at midstance and continues thru to toe off (pre-swing), then becomes concentric to advance limb at initial swing

A 45 year-old patient presented with pes planus and complains of pain on the plantar aspect during the late stance phase of gait. Which of the following interventions would be LEAST effective for this patient? 1. Medial glide at the subtalar joint 2. Stretching of gastrocnemius muscle 3. Marble pick ups 4. Improvement of 1stTMT joint ROM

1 pes planus: flat feet, dropped medial arch, limited windlass - overstretched and weakened plantar fascia - reduced ext of MTP joints limits usefulness of windlass effect

compare and contrast: 1) figure ground 2) form discrim 3) tactile agnosia

1) Figure-ground: ability to visually distinguish a figure from the background in which it is embedded. Compensatory for those who lack: place red tape over Velcro strap of shoe (to help patient find) and bright red tape to mark the edges on stairs 2) find an object among similarly shaped objects to assess if subtle differences in shape can be perceived 3) inability to recognize forms by handling them with eyes closed

A patient with a history of Parkinson's disease demonstrates a festinating gait. Which strategies given by the Physical Therapist is least effective in improving his gait? 1. A rolling walker to improve speed 2. Addition of a toe wedge to his shoes to slow propulsion 3. Leather shoes with hard flat heels to prevent falls 4. Vertical poles for upright posture Feedback

1) RW to increase speed - walker can inc festination and be a hazard toe wedge or flat heel preferred in a shoe to decrease fwd propulsion. hard composition shoe or leather shoe preferred over rubber soles: helps prevent falls vertical poles preferred: promote upright posture and can decrease festination due to forward propulsion

PNF patterns: 1) agonist reversals 2) dynamic reversals 3) rhythmic stab 4) rhythmic initiation

1) agonist reversal: resisted conc, ecc, iso. patient can benefit if they have dec eccentric control 2) dynamic reversal: concentric only 3) rhythmic stab: iso only 4) rhythmic initiation: concentric only

compare and contrast: crutches 1) two point gait 2) 3 pt gait 3) 4 pt gait 4) swing to gait

1) crutch and opposite LE moving together 2) when 1 extremity is NWB: advance 2 crutches then shift non-involved LE forward 3) 1 crutch, LE, crutch, LE 4) 2 crutches fwd, WB leg shifted to the crutches (not forward)

A patient presents with a painful ulcer on the lateral malleolus. He reports pain when walking a small distance and the pain decreases after resting for a few minutes. What is LEAST likely to be found in this patient? 1) pain relief on foot elevation 2) low granulation tissue in wound 3) pale wound with defined border 4) ABI of 0.60

1) pain relief on elevation (sign of venous ulcer not arterial)

A 41-year-old male patient arrived to an outpatient clinic with complaints of having low back pain with coughing and other functional activities. The patient describes having sharp pain originating from their back and extending to the foot, sharp pain with sitting, numbness/weakness in the back/leg, reflex changes, and hypersensitivity. The Physical Therapist concludes that the patient MOST likely has which of the following? 1) UMNL 2) LMNL 3) Sp cord involvement 4) GBS

2

A five-month-old infant is able to sit in a propped position. Which objective finding would be the MOST essential for the child to progress to ring sitting? 1.Increased strength of the trunk flexors 2.Increased strength of the trunk extensors 3.Integration of the symmetrical tonic neck reflex 4.Integration of the asymmetrical tonic neck reflex

2

A patient attending her third physical therapy session transitions from a hospital bed to standing in preparation for ambulation activities. Which blood pressure response would BEST support the physical therapist's decision to return the patient to a recumbent position? 1.Increase in systolic blood pressure of 18 mm Hg and an increase in diastolic blood pressure of 5 mm Hg 2.Decrease in systolic blood pressure of 5 mm Hg and a decrease in diastolic blood pressure of 12 mm Hg 3.Increase in systolic blood pressure of 20 mm Hg and a decrease in diastolic blood pressure of 7 mm Hg 4.Decrease in systolic blood pressure of 13 mm Hg and an increase in diastolic blood pressure of 2 mm Hg

2

A physical therapist observes that a patient with a history of recurrent lateral ankle sprains exhibits excessive supination during gait. Which of the following conditions would MOST likely be associated with this type of observation? 1.Tarsal tunnel syndrome 2.Peroneal tenosynovitis 3.Plantar fasciitis 4.Posterior tibial tenosynovitis

2

A physical therapist prepares to treat a patient using continuous ultrasound. What general rule BEST determines the length of treatment time when using ultrasound? 1.Two minutes for an area that is two times the size of the transducer face 2.Five minutes for an area that is two times the size of the transducer face 3.Five minutes is the maximum treatment time regardless of the treatment area 4.Ten minutes is the maximum treatment time regardless of the treatment area

2

A physical therapist works with a patient post CVA on a therapeutic exercise program. The therapist assists the patient in lateral weight shifting activities while positioned in prone on elbows on a mat table. Which of the following facilitation techniques would BEST allow the patient to improve dynamic stability with this activity? 1. Alternating isometrics 2.Approximation 3.Rhythmic initiation 4.Timing for emphasis

2 alternating iso: designed to facilitate isometric holding in agonists followed by holding of antagonists indicated for instability in WB, poor static postural control, and/or weakness approximation: ther ex technique designed to facilitate contraction and stability thru joint compression. the compression force is most often applied to joints thru gravity acting on BW, manual contacts, or weight belts RI: used for hypertonicity, inability to initiate movement, motor learning and communication deficits. begins with voluntary relaxation followed by passive movement thru increments in range. followed by active assisted progressing to resisted timing for emphasis: uses max resistance to elicit a sequence of contractions from major mm components of a pattern of motion. it allows overflow to occur from strong to weak mm.

As part of the medical history, a patient reports a sudden onset of pain. Which medical condition is MOST consistent with this clinical presentation? 1.Bicipital tendonitis 2.Hamstrings strain 3.Osgood-Schlatter disease 4.Peripheral vascular disease

2 bicipital tendonitis: pain comes on gradually hamstrings strain: sudden, sharp pain osgood: gradual PVD: intermittent claud and tingling/numbness

A physical therapist examines a patient post transtibial amputation. The patient resides in a retirement community and is socially active. The patient is presently using a temporary prosthesis consisting of a plastic socket, a pylon, and a SACH foot. The patient expresses concern that the permanent prosthesis will look awful and will be obvious to everyone. Which type of prosthesis would be the MOST appropriate for the patient? 1.Endoskeletal shank and single-axis articulated foot-ankle assembly 2.Endoskeletal shank and solid ankle cushion heel (SACH) foot 3.Exoskeletal shank and single-axis articulated foot-ankle assembly 4.Exoskeletal shank and solid ankle cushion heel (SACH) foot

2 single axis articulated foot ankle would be heavier and requires more maintenance

A physical therapist prepares a patient for a graded exercise test using an arm cycle ergometer instead of a treadmill due to a lower extremity injury. Which objective finding is the MOST accurate when using the arm cycle ergometer instead of the treadmill? 1.The results of the testing will be more heavily influenced by patient motivation. 2.The obtained maximum oxygen consumption will be significantly lower. 3.The work level associated with the point of volitional fatigue will be greater. 4.The incremental work levels will be progressively shortened during testing.

2 the obtained max O2 consumption is 20-30% lower than the value obtained with TM testing (smaller mm of arms vs legs)

For diagnostic purposes, what is the recommended number of chest radiographs to be performed on a patient when standard radiographs are ordered for a patient with suspected pulmonary pathologies?

2 radiographs 1) standing, P-A 2) left lateral view (unless pathology is suspected on the right side)

A physical therapist observes that a patient has difficulty controlling the affected lower extremity during the loading response. This phase of gait is characterized by which of the following muscle activity responses? 1.Increased quadriceps activity and increased hamstrings activity 2.Increased quadriceps activity and decreased hamstrings activity 3.Decreased quadriceps activity and increased hamstrings activity 4.Decreased quadriceps activity and decreased hamstrings activity

2) loading response= period between initial contact and the beginning of swing on the opposite leg requires inc quads to limit knee flexion, and hamstrings activity is decreased since the mm are no longer needed to prevent hyperext

A physical therapist treats a 30-year-old individual who was admitted to the hospital with insidious respiratory issues and small, red granulomas on their face, particularly surrounding the mouth. These clinical findings are typical of which of the following conditions? 1.Systemic sclerosis 2.Bronchiectasis 3.Sarcoidosis 4.Phenylketonuria

3

A physical therapist works with a patient who has been instructed to take non-steroidal anti-inflammatory drugs (NSAIDs) to help control the symptoms of arthritis. The therapist educates the patient that overuse of NSAIDs can result in gastrointestinal damage. This side effect is caused by the inhibition of the production of which hormone? 1.Angiotensin 2.Erythropoietin 3.Prostaglandins 4.Gastrin

3

A 65-year-old female patient with chronic low back pain is being treated by the Physical Therapist. After reading the patient's chart, the Physical Therapist cautions the PTA to avoid any spinal flexion exercises and abdominal crunches. What of the following is NOT a reason for the Physical Therapist's concern? 1) abdominal pain due to peptic ulcers 2) hyperparathyroidism 3) spondylolisthesis 4) regular corticosteroid use

3 avoid spinal extension with spondylolisthesis not flexion avoid spinal flexion in patients with: - peptic ulcers (increased IAP increases discomfort and chance of perforation) - corticosteroid use (osteoporosis) - hyperparathyroidism (can also lead to osteoporosis)

When deciding to perform postural drainage, which of the following is NOT a RELATIVE consideration for the Trendelenburg position? 1) pulmonary edema 2) hiatal hernia 3) CHF 4) axillofemoral bypass graft

4

A physical therapist inspects a patient's wheelchair and identifies that the wheel axle is aligned further posterior than it typically would be in a standard wheelchair. This type of alignment would MOST likely result in which of the following outcomes? 1.Decreased rolling resistance 2.Increased ability to balance on the rear wheels 3.Decreased turning radius 4.Increased energy required for propulsion

4 posterior alignment is often used for bilateral amputations to increase stability and compensate for change in COG - increases rolling resistance (decreases mechanical efficiency) - dec ability to perform a wheelie - increases turning radius - increases amount of energy for propulsion

in stork standing lumbar extension test, the patient has no pain with back extension during unilateral standing, but does have pain with back extension combined with rotation in standing. this pain presentation is likely correlated with: 1) TFL contracture 2) rec fem mm tightness 3) pars interarticularis fx 4) facet joint pathology

4 stork standing: stand on 1 leg and extend spine. If there is pain, this may be indicative of a pars interarticularis stress fx if the pain occurs with extension AND rotation, it is more correlated to facet joint pathology on the side that the rotation occurred

A group of physical therapists design a research study that examines the reliability of the Functional Independence Measure. The therapists utilize a test-retest design to measure reliability. What is the MOST significant source of error with this type of research design? 1.Sampling error 2.Tendency to rate too strictly or leniently 3.Change in test forms due to sampling of items 4.Change in subject situation over time

4 test-retest necessitates an interval of time between test administrations

A physician prescribes a spinal brace for a patient with chronic low back pain. Which of the following braces would be the MOST appropriate if the objective is to allow spinal motion while increasing intra-abdominal pressure? 1.Taylor 2.Milwaukee 3.Jewett 4.Corset

4 corset: non rigid spinal orthosis - will allow spinal motion but inc IAP taylor= TLSO milwaukee= CTLSO jewett: TLSO with 2 anterior plates

A patient presents with increased pigmentation and reports that they have been fatigued however they improve with rest. Additionally, the Physical Therapist notices that the patient has a blood glucose level of 60 mg/dL and blood pressure of 102/72 mmHg. The Physical therapist should suspect that the patient has which of the following? 1. Hypoparathyroidism 2. Hypothyroidism 3. Hyperthyroidism 4. Addison Disease

4) Addison: ark pigmentation of the skin, low blood pressure, progressive fatigue which improves with rest and hypoglycemia would not see hypoglycemia in hyperthyroid (would see fatigue, hyperpig and low BP) hypothyroid: would see elevated BP, delayed glucose uptake, decreased glucose absorption, no inc in pigmentation

A Physical Therapist is treating a 50-year-old male with end stage renal disease and is on dialysis. The patient has a history of hypertension and diabetes. Which of the following interventions would be LEAST appropriate? 1) exercises corresponding to 3-5 METS 2) exercise during dialysis 3) RPE during treatment to monitor intensity 4) exercise testing is recommended prior to starting exercise regimen

4) exercise prior to dialysis can cause instability low intensity during dialysis & moderate after dialysis is recommended

optimal pressure to clean wound with irrigation

4-15 psi (with wounds, do pressure close to 4 to prevent damage and to protect granulation tissue)

post medial meniscus repair, how many weeks should you limit knee flexion to 90 deg?

4-6 weeks to limit shear

FES parameters for muscle weakness - ramp up - rate - on/off

4-8 sec for strengthening pulse freq 35-80 pps on:off- 1:5 treatment time 10-20 mins

A physical therapist performs a gait analysis on a patient with a lower extremity injury. The therapist begins the session by observing the patient at free speed walking. What is the normal degree of toe-out at this speed?

7 degrees toe out

A patient reports pain during testing of active shoulder range of motion. The physical therapist suspects that the pain may be associated with anterior glenohumeral instability. Which portion of the shoulder range of motion should the therapist expect the pain to be MOST pronounced?

80 to 90 degree of lateral rotation

A female patient is referred to a physical therapy clinic with a diagnosis of chronic periarthritis of the shoulder. The patient mentions that she feels tired all the time, and has noticed recent weight loss, but with an increased appetite. Which of the following manifestations would MOST likely be displayed by the patient if hyperthyroidism was suspected? Select one: a. Increased deep tendon reflexes (DTR), low blood pressure (BP), heat intolerance b. Decreased deep tendon reflexes (DTR), high blood pressure (BP), cold intolerance c. Increased deep tendon reflexes (DTR), low blood pressure (BP), cold intolerance d. Decreased deep tendon reflexes (DTR), high blood pressure (BP), heat intolerance

A

A 45-year-old diabetic patient complains of gradual onset of right shoulder pain 3 weeks ago. The pain aggravates at night and with overhead activities. Pain is usually 0/10 at rest and can increase up to 6/10 with activities, such as combing hair or reaching behind for the back pocket to remove the wallet. The patient does not associate the pain with any traumatic event. Which of the following parameters of ultrasound would be the MOST APPROPRIATE for this patient? a. 3 MHz, 10 min, 0.5 W/cm^2, pulsed 20% US b. 3 MHz, 10 min, 2 W/cm^2, continuous US c. 1 MHz, 10 min, 0.5 W/cm^2, pulsed 20% US d. 1 MHz, 10 min, 2 W/cm^2, continuous US

A As the capsule is affected, which is located within 1-2 cm depth, we prefer a 3 MHz US. Generally, with a higher frequency, we keep the intensity low. As the tissue is in the acute stage, a pulsed US is preferred over a continuous one.

A patient with late stage Parkinson's disease (PD) presents to a physical therapy clinic with a report of frequent falls secondary to poor balance. Which of the following clinical presentations would be LEAST likely displayed for this patient? Select one: a. Sharp surge in the blood pressure (BP) with position change, such as a sit-to-stand b. Slow pupillary response to light c. Dysphagia d. Excessive daytime somnolence

A Typically, patients in middle and late stage PD may experience orthostatic hypotension (OH); a sharp drop in BP occurring during position changes (examples: supine to sit and sit to stand). This can place the individual at an increased risk of a fall / loss of balance (with injury). Dysphagia can be very common and possibly secondary to rigidity, decreased mobility, and restricted movement. Excessive daytime somnolence may be common in those with PD, along with insomnia at night. Those with PD may also display an abnormally slow pupillary response to light. Some cardinal features associated with PD: rigidity, bradykinesia, tremor, and postural instability. Note that there can be many more clinical features and manifestations associated with PD involving (not a complete list) motor performance, motor planning, motor learning, gait, posture, sensation, speech (voice/swallowing), cognitive function / behavior, autonomic nervous system, and cardiopulmonary function.

A 37-year-old librarian fractured the right proximal ulna. Her mid-forearm to mid-arm was in a cast with elbow flexed at 90 degrees for 12 weeks. Which of the following glides will be MOST APPROPRIATE for her to eventually be able to reach the books kept on an overhead shelf? a. Dorsal glide to the head of the radius at the humeroradial joint b. Volar glide to radius at the proximal radioulnar joint c. Dorsal glide to radius at the distal radioulnar joint d. Volar glide to the head of the radius at the humeroradial joint

A to reach overhead shelves, pt must improve extension at the humeroradial joint: POSTERIOR glide to inc elbow extension

A patient presents to the clinic with a painful wound. The characteristics of the wound are punched out margins over the dorsal aspect of the foot. The base of the wound looks pale and necrotic and lacks granulated tissue. When the extremity is lowered following elevation, the skin turns a dark red after 50 seconds. Which of the following interventions is LEAST appropriate at this time? a. Compression to have a pumping effect for improving blood flow b. Educating the patient about his condition and appropriate wound care at home c. A supervised walking program d. Elevation of the extremity until blanching followed by lowering of the feet

A this is an arterial ulcer tested by the rubor of dependency test (dark red color of skin >30 secs after lowering leg) compression therapy is CONTRAINDIC with arterial ulcers bc it will cut off the blood supply further D is Buerger-Allen exercises for improving peripheral claudication

sway back posture

A long outward curve of the thoracic spine with a backward shift of the trunk starting from the pelvis.

A physical therapist was evaluating a patient with persistent chest pain and occasional low back pain and cough. His cough had a brassy characteristic. Which of the following would the therapist MOST likely expect to be the cause of these findings? Select one: a. Interstitial fibrosis b. Aortic aneurysm c. Acute exacerbation of chronic bronchitis d. Heart failure

A patient with aortic aneurysm would present a cough that is brassy. Also, the patient would have persistent chest pain along with low back pain. A patient with heart failure would have a frothy, sputum with no chest pain. A patient with acute exacerbation of chronic bronchitis would have purulent sputum and can cause chest pain. A patient with interstitial fibrosis would develop a cough that is persistent and nonproductive and can cause chest pain.

A PT is performing nerve conduction velocity testing for the median nerve. The muscle most commonly used to assess this nerve is the:

Abductor pollicis brevis

A 24-year-old female is being evaluated for anterior knee pain. The patient walks without an assistive device and reports a dull constant pain when she is weight-bearing on the leg. She also has a history of multiple lateral ankle sprains from years of playing basketball. A clinical gait analysis reveals knee hyperextension during stance phase. The MOST LIKELY contributing factor to this gait presentation is: A. Soleus contracture retracting the tibia B. Contralateral abductor weakness causing a trunk lean C. Quadriceps contracture pulling the knee in hyperextension D. Degenerative joint changes causing a change in ground reaction forces

A: soleus contracture will retract the tibia, so the knee will hyperextend to compensate for lack of forward displacement during midstance

A physical therapist notices that a patient is experiencing early toe-off during terminal stance in gait. Which of the following identifies a likely cause, AND an appropriate intervention to address that cause? A. Hip flexion contracture; prolonged stretch B. Hip adductor weakness; progressive strengthening C. Gastrocnemius weakness; ultrasound D. Great toe flexion weakness; progressive strengthening

A: Hip flexion contracture will lead to lack of hip extension in terminal stance and will result in early toe off. An appropriate intervention to address this is to provide prolonged stretching to the hip flexor musculature to improve ROM.

role of piriformis

ABD and ER becomes IR after 90 deg hip flexion

you should avoid which motions at the shoulder when measuring active forearm pronation

ABD and IR

classify ABI scores by degree of arterial disease

ABI: LE BP/UE BP normal: 0.9-1.1 mild: 0.75-0.94 moderate: 0.50-0.74 severe: <0.50

which motion stresses AC joint?

AC joint is stressed with passive shoulder horizontal ADD

Which of the following Drug class would NOT be used to treat arrhythmias? 1. ACE inhibitors 2. Beta blockers 3. Calcium channel blockers 4. Sodium Channel blockers

ACE inhibitors Sodium channel blockers block fast sodium channels which slow the conduction in fast channel tissue and can be used to treat arrhythmias. Calcium channel blockers would be used to treat arrhythmias. Calcium channel blockers work by interrupting the movement of calcium into the heart and blood vessel tissue. They are used to treat high blood pressure, angina and arrhythmias. Beta blockers would be used to treat arrhythmias. Beta blockers decrease the heart rate and cardiac output, which lowers the patients blood pressure by blocking the effects of adrenalin. They can be used to treat arrhythmias and angina pectoris. ACE inhibitors would not be used to treat arrhythmias. ACE inhibitors are highly selective drugs that interrupt a chain of molecular messengers that constrict blood vessels. They can improve cardiac function in individuals with heart failure and are used for patients with diabetes or early kidney damage

A physical therapist reads in the medical record that a patient has an ejection fraction of 40%. Which class of medication is the patient MOST likely to be taking? 1.Angiotensin-converting enzyme inhibitor agents 2.Nitrate agents 3.Anticholinergic agents 4.Thrombolytic agents

ACE inhibitors: decrease BP and afterload by suppressing the enzyme that converts angiotensin 1 to angiotensin 2. this med is indicated for CHF and low ejection fractions

superior GH glide improves:

ADD

correct measurement for seat width

ADD.2 inches to the widest part of the hips

coxa vara posture

ANTERIOR pelvic tilt lumbar lordosis, fwd head, inc kyphosis genu valgum coxa vara

persist ATNR can interfere with:

ATNR: rotation of head causes same side extension (face side) and skull side flexion can interfere with hand-mouth activities and predisposes the child to scoliosis

A physical therapist treats a patient status post stroke. Which of the following actions would be the MOST likely to facilitate elbow extension in a patient who has hemiplegia? 1.Turn the head to the affected side 2.Turn the head to the unaffected side 3.Extend the lower extremities 4.Flex the lower extremities

ATNR: turn head to R, R UE will extend and L will flex

undetected childhood dev. hip dysplasia can result in:

AVN - Hip or groin pain - Gluteus minimum gait - Limited hip IR, flexion, ABD - TTP over hip joint - Femoral head is most common site. Sx may be mild initially and increase over time In babies: - asymmetrical gluteal folds - Asymmetrical knee height - Asymmetry of hip ABD ROM

A 7 year-old complains of a vague ache in their groin that radiates to the medial thigh and inner aspect of the knee. On examination, the Physical Therapist notices decreased abduction and internal rotation. What is the MOST appropriate intervention for this child? 1. Improving the flexibility of the adductors and internal rotators 2. Improving the containment of the femoral head in acetabulum using the brace 3. Improving the flexibility of abductor and external rotators 4. Improving the strength of abductor muscle using closed chain exercises

An important aspect in the intervention is the containment of the femoral head in the acetabulum. This is ensured by maintaining the hip in abduction and mild internal rotation for an extended period, using the Atlanta Scottish-Rite Hospital Orthosis. Although this brace does not use internal rotation-like casts as some other orthotics do, it does maintain the hip in abduction and permits approximately 90 degrees of hip flexion. Treatment methods include observation only; ROM exercises in all planes of hip motion (especially internal rotation and abduction);bracing; casting; gait training with brace and aquatic therapy

A 62 year old male with a 20 year history of smoking, DM II, HTN stage 1 and COPD presents with cough and expectoration. On auscultation the therapist finds presence of crepitus. Which of the following lobes can be drained MOST EFFICIENTLY in the sitting position? a. R Apical Segments of Upper lobe and R Superior Segments of Lower lobes b. L Apical Segment of Upper lobe and R Posterior Segment of Upper lobes c. R Apical segment of Upper lobe and R Anterior segment of Upper lobe d. L Apical Segment of Upper lobe and R Middle lobe

Apical and Posterior segments of the upper lobes can be drained in the sitting position anterior segments: supine superior segments: prone R middle lobe: sidelying B

A physical therapist is working with a patient who has a complete T6 spinal cord injury. In order to perform a successful transfer, the patient MUST use which of the following muscles?

At T6 spinal cord injury level, latissimus dorsi is innervated, the patient will need it to lift the buttock from the mat for a successful transfer. At this level of injury, quadratus lumborum (to lift pelvis) , internal oblique (to move trunk) and erector spinae (stabilize spine in extension) are not innervated.

A 32 year old female in her early pregnancy attended a seminar on the changes that can be expected during pregnancy; this course was conducted by her physical therapist. Which of the following would be LEAST expected during pregnancy? Select one: a. Increase in tidal volume (TV) and minute ventilation with no change, or slight decrease, in total lung capacity (TLC) b. Decrease in cardiac output (CO) with a decrease in heart rate (HR) by 5-10 beats per minute c. Increase in the subcostal angle and flaring of the ribs d. Increase in the size of the kidneys and an increased chance of urinary tract infection

B during pregnancy: CO and HR usually INCREASE (HR usually increases 10 to 20bpm and the CO increases 30-60%). The kidneys increase in length by 1cm (or 0.5"). Uterine enlargement causes ureters to enter the bladder at a perpendicular angle, which may cause a reflex of urine out of the bladder and back into the ureter; this may increase the chances of developing urinary tract infections secondary to urinary stasis. Changes in rib position occur prior to uterine enlargement and are hormonally stimulated; this causes the subcostal angle to increase progressively and the ribs to flare up and out. The respiratory rate remains unchanged, but depth of respiration increases. TV and minute ventilation increases, but the TLC is unchanged, or may slightly decrease.

A factory worker sustained an injury distal to the wrist on the dorsal surface of his right hand. Which of the following would MOST LIKELY be present on evaluation if he is suspected to have injured a branch of his radial nerve? a. Loss of 2-point discrimination on the dorsal surface of the last two fingers b. Loss of 2-point discrimination on the dorsal surface of the thumb c. Incomplete supination d. Inability to extend his fingers

B radial nerve enters the hand on the dorsal surface as the superficial radial nerve it is SENSORY only (C and D are out) the influence of radial nerve on hand mm is entirely proximal to wrist radial nerve injury= loss of 2 pt discrimination on the dorsal surface of the thumb

A physical therapist attempts to determine the number of known risk factors a patient has for coronary artery disease prior to initiating an aerobic exercise program. Which of the following objective findings would serve as the MOST relevant risk factor? 1.Blood pressure = 128/78 mm Hg 2.Waist-hip ratio = 1.15 3.Low-density lipoproteins = 96 mg/dL 4.Body mass index = 23.9 kg/m2

B waist hip >0.90 is an indication of central obesity normal LDL <100 CAD risk factors: age, smoking, sedentary lifestyle, obesity, htn, preDM

1) A patient has limited movements through the arc of forearm supination and pronation due to mod-severe pain. Manual technique to proximal radioulnar joint initially: a. Small amplitude mobs at the beginning of available ROM: 5 deg supination, 90 deg elbow flexion b. Large amplitude mobs at end of ROM: 10 deg supination, 70 deg elbow flexion c. Large amplitude mobs in the middle of ROM: 35 deg supination, 70 deg elbow flexion d. Small amplitude mobs at end of ROM: full supination and full elbow extension

C

A 42 year old male is admitted to the hospital with severe liver damage secondary to chronic alcoholism; he was also recently diagnosed with depression. Which of the following would MOST likely be correlated with the patient's lab report? Select one: a. Ammonia 70 mcg/dL; serum albumin 3.6 g/dL b. Ammonia 55 mcg/dL; serum albumin 4.5 g/dL c. Ammonia 110 mcg/dL; serum albumin 2.0 g/dL d. Ammonia 60 mcg/dL; serum albumin 4.0 g/dL

C

A patient presents to the clinic with loss of pain and temperature sensation bilaterally. The patient also has bilateral weakness of both extremities UE > LE. When asked about the position of his limbs in space, the patient could not respond correctly 8 out of 10 times. Cognition and speech are intact. Which of the following is the MOST LIKELY diagnosis for this patient? a. Posterior Cord Syndrome b. ACA infarct c. Central Cord Syndrome d. Anterior Cord Syndrome

C - loss of pain/temp= anterolateral is out - loss of proprioception= DCML is also out - motor weakness= corticospinal out --> can get rid of anterior and posterior cord - higher function is normal (cognition and speech), so can rule out ACA UE>LE central cord

A 70-year patient is brought to the clinic by her daughter with complaints of bumping into objects (such as the door, or a wall) while walking. She has had four falls in the last two months. Her sensations are intact and there are no complaints of dizziness or loss of hearing. The symptoms were gradual in onset. The resting position of the eyes was normal with no signs of involuntary eye movements, drooping of eyelids or double vision. Which of the following cranial nerves (CNs) is MOST likely impaired? a. CN III b. CN VIII c. CN II d. CN IV

C CN 2: optic- visual acuity and peripheral vision. when affected, it can cause impaired near vision, impaired far vision, or blindness. this CN 2 impairment can lead to bumping into objects CN 8 can be ruled out in this case bc hearing was intact with no signs of dizziness

An orthopedist refers to physical therapy a patient with advanced rheumatoid arthritis for maintenance therapy. While reviewing the patient's records, the therapist comes across his radiographs. Which of the following will MOST LIKELY NOT be found on the radiographs of this patient with rheumatoid arthritis? a. Synovial pseudocysts b. Erosion of the joint surface of the 1st MTP joint c. Tenosynovitis d. Periarticular swelling

C Common findings with RA: - inflammatory changes in tendon sheath, but they cannot be differentiated on an x-ray - subchondral radiolucent defects - periarticular swelling - loss of joint space - periarticular osteoporosis - synovial pseudocyst - telescoping of digits - erosion of articular cartilage - periarticular fusiform swelling

A physical therapist is assessing a patient with a chief complaint of weakness and pain in the right hand. The patient is a computer engineer by profession and spends hours at a time at his desk. During assessment, the therapist asks the patient to pull a piece of paper from the therapist's hand. The patient was unable to pinch between the index finger and the thumb without flexion occurring at the DIP joint. Which of the following nerve tension tests should the therapist expect to be POSITIVE in this patient? A. Shoulder abduction (110 degrees), elbow extension, forearm pronation, wrist flexion and ulnar deviation B. Shoulder abduction (10 degrees), elbow extension, forearm supination and wrist extension C. Shoulder abduction (90 degrees), elbow flexion, forearm supination, wrist extension and radial deviation D. Shoulder abduction (110 degrees), elbow flexion, forearm supination and wrist flexion

C Froment's sign: ulnar nerve entrapment ulnar nerve ULTT: - shoulder abd 90 - elbow flex - supination - wrist ext and radial deviation * radial nerve: 110 deg abd, elbow ext, pronation, wrist flex, ulnar dev

A PT consult is ordered for a 50-year-old gentleman in the late stages of ALS. In the patient's chart is an electromyography report and nerve conduction velocity test. All of the following findings are consistent with his diagnosis EXCEPT: A. Decreased amplitude of motor unit action potential B. Decreased duration of motor unit action potential C. Decreased sensory evoked potential D. Decreased polyphasic action potential

C Sensory evoked potential will not be seen in ALS as ALS is a motor neuron disease affecting the motor nerves and not sensory nerves. A, B and D are findings consistent with ALS.

A patient with congestive heart failure (CHF) has pitting edema bilaterally in the feet. The patient also reports that he must use at least three pillows to be able to sleep at night. Which of the following signs would be LEAST likely found on this patient's chest x-ray? Select one: a. Blunting of the costophrenic angle b. Enlarged cardiac silhouette c. Honeycombing d. Opacities in the lung fields with interstitial and parenchymal edema

C hallmark x-ray features for CHF: - enlarged cardiac silhouette - blunting of costophrenic angles 2/2 settling of fluid at the lower portion of the lung - opacities in the lung fields with interstitial and parenchymal edema 2/2 excessive fluid honeycombing more related to a restrictive lung disease. commonly seen with pulmonary fibrosis

An intern is interested in learning the difference between venous insufficiency and arterial insufficiency. Which disorder MOST likely occurs with abnormal venous blood flow? Select one: a. Thromboangiitis obliterans b. Raynaud's Disease c. Thrombophlebitis d. Buerger's Disease

C thrombophlebitis: venous blood clot thromboangiitis obliterans: aka Buergers, arterial blood clot

Which of the following is NOT a part of the Ottawa Rules for Ankle X-Rays (with Buffalo Modifications)? Select one: a. Tenderness over lateral malleolus to 6 cm proximally b. Tenderness over navicular c. Tenderness over base of first metatarsal d. Tenderness over medial malleolus to 6 cm proximally

C The Ottawa Rules for Ankle X-Rays (with Buffalo Modifications) includes the following: -Tenderness over lateral malleolus to 6 cm proximally •Tenderness over medial malleolus to 6 cm proximally •Tenderness over navicular •Tenderness over base of fifth metatarsal.

A 35-year-old female presents to the clinic with a chief complaint of pain in the mid-back. The pain is on and off throughout the day, but is certainly more when she wakes up in the morning and doesn't subside for an hour. It is only when she is sitting in her chair having breakfast that she notices that the pain is gone. She is an elementary school teacher by profession and must stand for prolonged hours. She attributes the pain to prolonged standing. She is trying to lose weight for her wedding and is practicing intermittent fasting. She has had episodes of mild stomach pain, lightheadedness, fainting, and vomiting. Which of the following is the MOST LIKELY cause of her symptoms? a. Gastric ulcer b. Mechanical low back pain c. Duodenal ulcer d. Ulcerative colitis

C: duodenal ulcer --> prominent when stomach is empty and in the early am. pain may be relieved by antacids and may last from mins to hours other sx: nausea, vom, bloody stools * gastric ulcers would aggravate sx in prsence of food * Ulcerative colitis: pain associated with large intestine or colon ma be referred to sacrum and relieved after defecating/passing gas

A physical therapist is learning about aquatic therapy and the effects of the properties of water. Which of the following will MOST LIKELY have the highest buoyancy in water? a. A patient with a BMI of 19 b. A patient with amputation of the left foot c. A patient with osteoporosis d. A patient immersed in water up to the ASIS level

C: patients with dec bone density have INC buoyancy * obese= dec buoyancy bc more fat tissue (has lower specific gravity) * buoyancy increases with FULLY INFLATED LUNGS - effects of buoyancy are higher when the person is completely immersed in water

A 65-year-old male had a right parietal lobe infarct 6 weeks ago. Since the CVA, he requires moderate assistance for ADLs, minimal/moderate assistance to ambulate in his home environment with a walker, and moderate assistance to maintain static standing balance. Based on the scenario, which of the following physical therapy interventions would be LEAST appropriate at this time in his recovery? A. Visual feedback for midline orientation B. Seated activities lifting both hands in midline C. Use of cane in unaffected side for walking D. ADL training using both hands to drink from a cup

C: use of cane in unaffected side will further promote imbalance by causing a lean to the affected side

Nerve injury below inguinal ligament a. Increased posterior lean of trunk in midstance b. Increased IR of thigh in mid swing c. Hyperextension of knee in stance d. Excessive PF during foot flat (loading)

C; femoral nerve lies under inguinal lig

adhesive capsulitis is commonly seen in association with all of the following diseases except: 1) DM 2) hypothyroidism 3) CAD 4) hyperthyroidism

CAD

4 functional classifications of RA according to the American College of Rheumatology

CLASS I -- pt can perform ADL's (self care, vocational, avocational) CLASS II -- pt can perform usual self care and vocational activities, but is limited in avocational activities CLASS III -- pt can perform usual self care but is limited in vocational and avocational activities CLASS IV -- limited in ability to perform self-care, vocational, and avocational activities

capsular pattern for 1st CMC capsular pattern for CMC 2-5

CMC 1: abduction > extension CMC 2-5: equally restricted in all directions

zig-zag deformity

CMC flexion, MCP hyperextension associated with RA, usually seen in thumb

You are treating a patient in inpatient rehabilitation with deficits in pupillary light reflex in the right eye and a failure to look at the nose with the right eye. When testing lateral deviation of the eye, the patient is able to follow the stimulus. No deficits noted in chewing. Which anatomical location is MOST likely affected? a. Pons b. Cerebellum c. Medulla d. Midbrain

CN 3 and 4: midbrain CN 5,6,7,8: pons Cranial nerve III is the oculomotor nerve, and damage to the oculomotor nerve causes ptosis (drooping eyelid), dilation of the pupil, and loss of accommodation of the light reflex. Cranial nerve IV is the trochlear nerve. The trochlear nerve's motor innervation is to the superior oblique muscle which is responsible for rotating the eye down and in.

lateral pterygoid is innervated by:

CN 5

temporalis is innervated by:

CN 5

lesion at pontomedullary region would cause deficits in:

CN 5,6,7,8

You are treating an infant with delayed milestones. During the session, the mother notices some dirt in the child's eye and tries to clean it with a tissue. The therapist notices that the child did not blink when the mother touched the eye, but is able to blink otherwise. Which of the following cranial nerves is MOST LIKELY impaired in this scenario?

CN V- trigeminal - corneal reflex: sensory branch of trigeminal, touching cornea lightly with wisp of cotton facial nerve will control the motor aspect of the corneal reflex (blinking) but this can be ruled out bc the patient can wink

vertical nystagmus

CNS dysfunction

compression of tibial nerve sx

Compression of the tibial nerve can happen as it passes through the popliteal fossa (can be secondary to an enlarged baker's cyst). The patient may present with pain behind the knee or in the calf with ankle dorsiflexion, along with decreased knee flexion. Hypesthesia (or anesthesia) may also be noted of the entire plantar aspect of the foot. The patient may demonstrate decreased strength of the gastrocnemius, tibialis posterior, flexor hallucis longus, flexor digitorum longus, along with the foot intrinsic muscles (minus the extensor digitorum brevis).

A 59-year-old female patient arrived to an outpatient clinic with complaints of having wrist and hand discomfort. Upon examination, the Physical Therapist observed flexion of MCP and distal interphalangeal joint with extension of the PIP joint. Which of the following conditions would MOST likely match this description? 1. Rupture of the central tendon 2. Contracture of intrinsic muscles along with dorsal subluxation of lateral extensor tendons 3. Banding on the palm and digit flexion contractures 4. Rupture of the extensor tendon at the distal phalanx

Contracture of intrinsic muscles along with dorsal subluxation of lateral extensor tendons swan neck: contracture of intrinsic mm with dorsal subluxation of lateral extensor tendon trigger finger= rupture of extensors at distal phalanx dupuytren: banding at palm + digit flexion contractures

A physical therapist was considering the use of Performance Oriented Mobility Assessment (POMA) on a patient with Parkinson's disease at stage IV of the Hoehn & Yahr. However, he decided against the use of that specific outcome measure due to the floor effect. Which of the following is the MOST appropriate description of floor effect? Select one: a. A limitation of a measure in which the instrument does not register a further increase in score for the highest scoring individual b. A limitation of a measure in which the instrument does not register a further increase in score for the lowest scoring individual c. A limitation of a measure in which the instrument does not register a further decrease in score for the highest scoring individual d. A limitation of a measure in which the instrument does not register a further decrease in score for the lowest scoring individual

D

A physical therapist would like to perform postural drainage to a patient's anterior basal segment of the left lower lobe. Which of the following can be considered the MOST appropriate technique to perform postural drainage on this patient? Select one: a. The patient is lying on the left side with a 1/4 turn backward (pillow placed behind the patient from shoulder to hip); the foot of the bed is elevated 16 inches and the patient's knees are flexed b. The patient lies prone with 2 pillows under the hips; the bed is flat c. The patient is lying on the left side with the foot end of the bed elevated 20 inches and a pillow under the knees d. The patient is lying on the right side with the foot end of the bed elevated 20 inches and a pillow under the knees

D

physical therapist decided to incorporate postural drainage to clear the secretions and improve ventilation in a patient. The patient was positioned on the abdomen, head down, and rotated 1/4 turn upward with his right shoulder off the bed, the right leg flexed over the pillow for support with the foot of the bed elevated 20 inches. Which of the following lung segments was the therapist MOST likely addressing? Select one: a. Upper lobes right anterior basal segments b. Upper lobes left lateral basal segments c. Lower lobes left anterior basal segments d. Lower lobes right lateral basal segments

D

A patient presents to the clinic with a chief complaint of numbness and tingling in the sole of the left foot. On examination, all reflexes were found to be intact. However, you noticed an absence of left heel off during ambulation, along with weakness of invertors of the left foot. Which of the following is the MOST LIKELY cause for this presentation? a. L5 radiculopathy b. S1 radiculopathy c. Superficial peroneal nerve injury d. Tibial nerve neuropathy

D - absence of heel off= weak PF= S1 and tibial nerve tibial nerve supplies the sole of the foot S1 supplies the lateral sole of foot and the lateral plantar surface of the foot S1 radiculopathy= loss of PF reflex and weak evertors superficial peroneal nerve= loss of sensation on the anterolateral aspect of the lower leg L5 radic= weak DF with loss of sensation on the medial side of the dorsal and plantar aspect of the foot

A 65 year old patient with diabetic neuropathy presents to the clinic with a complaint of frequent falls especially at night. Which of the following conditions of the Sensory Organization Test is this patient MOST LIKELY to score poorly? a. Conditions 2 and 4 b. Conditions 4 and 6 c. Conditions 1 and 3 d. Conditions 5 and 6

D Due to peripheral neuropathy, this patient is more dependent on their vision and vestibular systems. Patients depending on vision become unstable in conditions 2, 3, 5 & 6 where we either close the eyes, or have a conflict between vision and the vestibular system. With conditions 1 and 4, the patient will have the opportunity to rely on the visual system and maintain balance.

Which of the following is the correct terminology for the International Classification of Impairments, Disabilities, and Handicaps (ICIDH)?

Disease → Impairment → Activity-Disability → Environmental-Personal Factors

Hypokalemia symptoms

Dry mouth, thirst, weakness, drowsiness, muscle aches, tachycardia, mm cramps flattened or inverted T waves

capsular pattern of the shoulder

ER > ABD > IR POSTERIO INFERIOR GLIDE

A 62 year-old male arrived to the emergency department complaining of abdominal pain, nausea, vomiting and feeling fatigued. Upon further examination, the physician conducted a blood analysis and concluded that the value of INR would MOST LIKELY: 1) remain normal 2) increase 3) decrease 4) be of no concern to this patient

INC INR: thinner blood (dehydration= lacking vitamin K, malabsorption of vitamins, thinner blood) dec in INR: thicker blood, increases risk of developing a clot

what mm causes excessive IR of the shoulder?

IR: teres minor & subscap tightness of pec mm

A physical therapist observes a patient's wheelchair footrests touching the ground. What is the MOST LIKELY cause?

If the seat height is low, foot plate can touch the floor. - increased seat depth causes a kyphotic posture, sacral sitting and compromises circulation. - too low depth fails to support the thigh adequately and results in poor balance because of decrease in the base of support. -increased leg length encourages sacral sitting and forward sliding in the chair.

Meconium ileus

In cystic fibrosis, meconium plug obstructs intestine preventing stool passage at birth. -- early manifestation of CF in infant

A PT is assessing the gait of a patient inorder to figure out the reason for the patient's abnormal gait pattern. Which of the following combinations of orthotic and anatomical causes will MOST likely lead to similar gait deviations? Select one: a. Excessive height of medial upright of left knee-ankle-foot orthosis and left gluteus maximus weakness b. Inadequate dorsiflexion assist and weak hip flexors c. Excessive height of lateral upright of knee-ankle-foot orthosis and abduction contracture on the left d. Inadequate dorsiflexion stop and pes equinus deformity

Inadequate dorsiflexion assist and weak hip flexors is correct. Inadequate dorsiflexion (DF) assist and weak hip flexors both can lead to a circumduction gait. Inadequate DF stop or pes calcaneus deformity (not pes equinus) will lead to the same gait deviation of delayed transfer of weight over forefoot in late stance. Walking with a wide base of support is seen with abduction contracture or an excessive medial upright of knee-ankle-foot orthosis (KAFO). Lateral trunk bend towards the stance leg is seen with excessive medial upright of KAFO and weak gluteus medius.

posterior tibial tenosynovitis

Inflammation of posterior tibial tendon. Symptoms inferior to the medial malleolus. Supports the arch of the foot. Flat feet/pronation.

Retrocalcaneal Bursitis

Inflammation of the bursa between the Achilles tendon and the calcaneus. - pain behind ankle, posterior to talus - no pain with resistive testing

A physical therapist is performing a neurological evaluation of a patient admitted to a skilled nursing facility. When the therapist stimulates the back of the patient's throat on the left side, he notices an absence of response. Stimulation of the palate on the right side causes the elevation of soft palate with no deviation to either side. Which of the following cranial nerves is LEAST likely to be intact? Select one: a. Left vagus nerve b. Right glossopharyngeal nerve c. Left glossopharyngeal nerve d. Right vagus nerve

L glossopharyngeal nerve gag reflex: elevation of soft palate and B/L contraction of pharyngeal mm evoked by stimulating the back of the throat lesion to glossopharyngeal (sensory): NO response of soft palate. with vagus nerve, the soft palate will pull away from the affected side

where to mobilize to inc R rotation at C5/6

L posterior articular pillar of C6

pancreatic pain refers to:

L shoulder

You are assessing a patient with a CVA who presents with right sided body weakness, inability to understand commands, weakness of the R half of the face and homonymous hemianopsia. Which of the following will MOST LIKELY be absent in this patient? a. Loss of sensation on the right side of body b. Topographic disorientation c. Limb-kinetic apraxia d. Lack of fluency while talking

L sided stroke: R sided weakness inability to understand commands, weakness of R half of face, homonymous hemianopia B With an L MCA infarct, Broca's aphasia, limb-kinetic apraxia and loss of sensation of the right UE and face are commonly seen. Topographic disorientation is usually seen with lesions involving non-dominant primary visual areas. topographic disorientation: inability to orient in the surrounding as a result of focal brain damage

best orthotic to help provide a PF stop?

L weak DF- can use a PF stop orthotic to prevent toe drag during swing plastic hinged AFO (aka posterior steel stop) prevents the ankle from PF so that patients with weak DF will not catch their toes and stumble during swing

A PT is assessing a patient with a chief complaint of low back pain and a diagnosis of L2-L3 nerve compression. Which of the following gait deviations at the hip and pelvis are NOT to be expected in this patient? a. Trunk lurches backward and toward the unaffected stance leg from heel off to mid swing b. Posterior tilt of the pelvis during initial swing c. Semicircle movement of the hip during swing—combining hip flexion, hip abduction and forward rotation of the pelvis d. Excessive hip flexion at initial and mid swing

L2/3 compression: weak hip flexors - can passively generate hip flexion with backward movement of trunk (A) - APT= tight hip flexor, PPT= weak hip flexor (B) - semi-circle movement= circumduction (C)' will not see excessive hip flexion if the mm are weak

L4 nerve root- where does it exit

L3/4

at the L4/5 nerve level: which root is exiting? which nerve is involved with stenosis? which nerve is involved with herniation?

L4 is exiting L4 is involved with stenosis L5 is involved with herniation

great toe extension myotome

L5

A 38-year-old male patient arrived at an outpatient clinic with sensory symptoms in the right lower extremity. The patient also complained of weakness in the right big toe while walking. The physical therapist performed a sensory test by lightly touching the proximal anterior lateral lower leg as the patient indicated having inability in feeling the touch. Which of the following levels is MOST likely affected?

L5: proximal anterior lateral aspect L4: proximal anterior medial aspect S1: distal anterior lateral aspect

A physical therapist works with a six-year-old patient with hip pathology. Which condition would be the MOST likely based on the patient's age? - apophysitis - LCPD - SCFE - RA

LCPD: 5-7 years, more common in M>F SCFE usually pre-adolescent and obese

signs and sx of liver disease

LIGHT stool, darkened urine - hyperbilirubinemia, R shoulder pain, jaundice, bruising, palmar erythema, confusion, asterixis

biofeedback for OVERACTIVE TIGHT MUSCLES - sensitivity - electrode placement

LOW sensitivity WIDELY spaced

Lasix side effects

Lasix is a loop diuretic that increase the renal excretion of water and sodium, thus decreasing the volume of fluid in the vascular system. One of its side effects is orthostatic hypotension; the addition of systemic heat from the aquatic therapy will cause vasodilation, which places the patient at danger for low BP.

Wallenberg syndrome

Lateral medullary syndrome/ posterior inferior cerebellar artery syndrome: Dysfunction: - Vestibular nuclei: vomiting, vertigo, nystagmus, - Inferior cerebellar peduncle: ipsilateral cerebellar signs including ataxia, dysmetria, dysdiadokokinesia -Lateral spinothalamic tract: contralateral deficits in pain and temperature sensation from body (limbs and torso) - Spinal trigeminal nucleus & tract: ipsilateral loss of pain, and temperature sensation from face - Nucleus ambiguus (vagus and glossopharyngeal nerve): ipsilateral laryngeal, pharyngeal, and palatal hemiparalysis: dysphagia, hoarseness, diminished gag reflex - Descending sympathetic fibers: ipsilateral Horner's syndrome (ptosis, miosis, & anhydrosis)

MMT test for lats

Latissimus dorsi is a shoulder extensor, adductor and internal rotator. To test the muscle, the patient is prone with head turned to one side, arms at sides; test arm is internally rotated and the patient is instructed to raise arm off table.

A 28-year-old software engineer reporting difficulty at work is referred to an outpatient physical therapy clinic. The patient complains of pain and numbness and tingling sensations in her left forearm, elbow, and hand. The PT suspects a median nerve injury but Phalen's test is negative. To confirm the median nerve involvement, the PT should MOST likely expect pain and weakness during which of the following motions:

Left forearm pronation, finger flexion (FDS, FDP) and thumb opposition (impaired thenar= can cause ape hand deformity)

A patient presents to the clinic with sudden onset of weakness on one side of their body with passing of urine involuntarily. The strength of the patient's left lower extremity is 1/5, upper extremity 3+/5. Reflexes are brisk in both UE/LE. Babinski is positive on the left. The patient did not react to pinprick sensation on the left side of body. Additionally, the patient has impaired judgement and neglect of the affected side of the body. Speech is fluent, and patient is able to understand commands. No signs of aphasia or visual field deficits are noted. Which of the following is the MOST LIKELY cause of the patient's impairments? 1. Right superior division of MCA infarct 2. Left ACA infarct 3. Right Inferior division of MCA infarct 4. Right ACA infarct

Lower limbs are more affected than upper extremities- This is classic signs of an ACA infarct. Urinary incontinence is seen when superior frontal gyrus is involved (posteromedial aspect) supplied by ACA. Also, as the left side of the body is affected, the right cortex would be involved. Other signs e.g. absence of aphasia and presence of neglect is also indicative of damage to right cortex

A physical therapist is obtaining patient measurements for a proper fit in a standard wheelchair with footrests. Which of the following is the MOST appropriate measurement for the seat height? Select one: a. Measure along the lateral thigh from the patient's posterior buttock to the popliteal fold; subtract 2 inches b. Measure from the patient's heel to the popliteal fold; add 2 inches c. Measure from the patient's heel to the popliteal fold; subtract 2 inches d. Measure along the lateral thigh from the patient's posterior buttock to the popliteal fold; add 2 inches

Measure from the patient's heel to the popliteal fold; add 2 inches

A physical therapist is educating his intern student on the various types of sensory receptors that are located at the terminal portion of the afferent fiber. The therapist mentioned that this particular receptor located in the dermis highly discriminates touch as well as the movement of objects over the skin. Which of the following types of sensory receptors is MOST likely described by the therapist?

Meissner's corpuscle

paroxysmal supraventricular tachycardia/paroxysmal atrial tachycardia

NSR followed by sudden atrial tacy then back to NSR

During examination, a physical therapist wants to assess the carpal bones of the hand. To make the trapezium more prominent, the PT would ask the patient to perform which of the following motions?

Opposing the thumb to the little finger and ulnarly deviating the wrist Trapezium is located immediately proximal to the base of the first metacarpal bone, just distal to the scaphoid. It can be made more prominent by opposing the thumb to the little finger and ulnarly deviating the wrist.

Bakody's sign

Pt presents w/ arm raised above head bc it alleviates/reduces their sx indicative of C4/5 lesion if sx are worse: probs TOS bc pressure is increased in the interscalene triangle with ABD and ER

e-stim parameters for mm spasm reduction - pulse frequency - pulse duration - amplitude - on:off - ramp time - treatment time

Pulse freq: 35-50 pps pulse duration: - 150-200 small mm - 200-350 large mm amplitude: till visible contraction on:off EQUAL (2-5 sec ON, 2-5 sec OFF) ramp time at least 1 sec treatment time 10-30 mins Performed every 2-3 hours until spasm is relieved

scapular assistance test

Purpose: Detects weak scapular control causing impingement symptoms. Method: Pt. is standing and PT is behind pt. The pt. flexes shoulder in scapular plane while the PT pushes inward, laterally, and upward on the medial border of scapula. Positive Test: Decreased pain during shoulder elevation with scapular assistance. Both the serratus anterior and lower trap UPWARDLY rotate the scam in arm elevation

R posterior innominate rotation

R ASIS higher than L, R PSIS lower than L leg shorter in supine, longer in sitting stretch tight hip extensors

A physical therapist is treating a patient with a diagnosis of right cerebrovascular accident. On evaluation, the tone in his biceps was graded at a 2 on the Modified Ashworth Scale. Which of the following is the MOST effective intervention to address this finding? 1) quick icing of L biceps 2) prolonged cryo of L biceps 3) neutral warmth R biceps 4) quick icing R triceps

R CVA= L bicep quick icing will dec spasticity but is contraindicated in agonist mm group bc it will INC TONE hypertonicity can be refused by neutral warmth or prolonged cryotherapy to the hypertonic mm - can also be reduced indirectly by stimulation of antagonist mm contraction motor level ES or quick icing to antagonist

patient has R pleural pain- what side should they lie on?

R S/L In pleural pain, pain is alleviated by lying on the affected side (in this case- right side), which diminishes the movement of that side of the chest. This is called auto-splinting.

A physician is examining a patient with a gradual onset of difficulty raising the right eyebrow and holding air in the right side of the mouth, along with loss of hearing in the right ear. Which of the following is the MOST LIKELY location of this lesion? a. Right cerebral cortex b. Right pontomedullary region c. Left cerebral cortex d. Right midbrain region

R pontomedullary can rule out cortex bc the entire half of the face is affected instead of just the lower half

A patient presents to the clinic four weeks after a reverse arthroplasty of the right shoulder. Which of the following movements would you perform while treating the patient at this stage? a. Shoulder extension beyond 0 degree b. Shoulder IR up to 30 degree c. Shoulder ER beyond 20 degree d. Shoulder elevation in scapular plane up to 90 degree

REVERSE TSA: up to 6 weeks - NO GH extension or IR past neutral - NO combined extension, ADD, IR - up to 20 deg of ER - up to 90-120 deg elevation in scapular plane D

at what levels would you probs use the following: 1) RGO 2) KAFO 3) AFO 4) supramalleolar orthosis

RGO: L1 KAFO: L3-4 AFO: L4-5 supramalleolar: S1-2

A Physical Therapist is treating a patient who is recovering from TBI and is RLA stage 5. After 30 minutes into the therapy session the patient is not able to focus and starts losing interest in the activity. What is the appropriate response of the Physical Therapist? 1. Ask the patient to focus and complete the session 2. Take the patient to a different setting with group activities to keep him motivated 3. Take a break and resume the rest of the activity after the break. 4. Stop the activity for the day as he is fatigued.

RLA stage 5 is defined as confused inappropriate. In this stage patients generally cannot focus for more than 30 minutes and quickly lose interest. These patients require frequent breaks to prevent fatigue and loss of attention. Giving a break and resuming the activity is the best way to keep this patient's focus. Group activities can be overwhelming and distracting and not suitable in this stage. Stopping the activity for the day is not required.

A 22-year-old female student is referred to physical therapy for complaints of migraine headaches. The therapist notices that she has a rosy appearance over her cheeks, nose, and chin. She also admits to having burning episodes, accompanied by intermittent flushing of her cheeks, as well as gastrointestinal disturbances. Which of the following conditions BEST matches the clinical findings?

ROSACEA Rosacea is a chronic facial skin disorder, it causes redness of the cheeks, nose, chin, forehead that comes and goes. With advanced Rosacea swelling of the nose (rhinophyma) is seen. Bumps and pimples appear. Chronic facial flushing causes the telangiectasia. Migraines have found to be associated with Rosacea.

how does rotator cuff repair affect total shoulder arthroplasty?

RTC repair: need to wear sling for 4-6 weeks No RTC: sling can be removed same day of surgery

A physical therapist identifies a limitation in movement after performing a straight leg raise test on a patient in the supine position. The therapist then passively flexes the patient's knee on the test leg, but is unable to gain any additional hip flexion range of motion. Which of the following conditions should the therapist MOST likely suspect? 1.Gluteal bursitis 2.Hamstrings strain 3.Sciatic nerve pathology 4.Trochanteric bursitis

SLR with and without knee flexion: no change indicates buttock pathology - gluteal bursitis, abscess, tightness of glut max, tight posterior capsule sciatic nerve: flexing the knee would inc range

A 21-year-old female field hockey player has a history of several right leg hamstring strains. She has been performing frequent stretching exercises for the right hamstrings with minimal changes in ROM observed. What modality can PT utilize to BEST improve her ROM and facilitate stretching? A. Ultrasound B. Iontophoresis C. Short Wave Diathermy D. TENS

SWD is a deep heating modality and can be used to cover a large area like the hamstrings unlike ultrasound, whose heating is limited by the effective radiating area of the sound head. SWD helps in increasing soft tissue extensibility which can facilitate stretching and help improve the ROM.

APGAR Score

Scale till 10, evaluated at 1 min and 5 min post birth 8-10: normal <8: have to do again - APPEARANCE (2) pink (1) pink body, blue extremities (0) blue - HEART RATE (2) 100+ (1) <100 (0) no pulse - TONE (2) active (1) flexed arms and legs (0) no movement - GRIMACE/REFLEX IRRITABILITY (2) responds to stimuli (1) minimal response to stimuli (0) floppy - RESPIRATION (2) crying (1) slow and irregular (0) no respiration

excessive anterversion

TOE IN tibial IR genu valgum coxa vara over-pronation medial tibial & femoral torsion compensation: lateral tibial torsion, lateral rotation at knee, lumbar rotation on same side

pes anserine pain

TTP anteromedial knee below joint line - pain with active knee flexion (semitendinosus and sartorius attach= stress with ext and valgus), passive knee flexion, and valgus stress

Tarsal Tunnel Syndrome

Through this tunnel pass the tibialis posterior, flexor hallucis longus, and flexor digitorum muscles with their surrounding synovial sheaths and the tibial nerve artery and vein. - pain with passive ankle eversion - pronated foot - valgus deformity - weak toe flexors

tarsal tunnel syndrome

Through this tunnel pass the tibialis posterior, flexor hallucis longus, and flexor digitorum muscles with their surrounding synovial sheaths and the tibial nerve artery and vein. posterior to medial malleolus commonly associated with flat feet or pronation

A 17-year-old female track athlete is in the clinic and complaining of mild left arch pain, as well as pain just posterior and slightly superior to the medial malleolus when she runs. Upon a quick look at her feet in standing, the PT notes that her left foot is significantly more pronated than the right. The PT hypothesizes that the source of her pain is due to the inability to control pronation through stabilization of the subtalar joint. What muscle test would be MOST important to include in the initial evaluation?

Tibialis posterior muscle is a plantarflexor and invertor of the ankle. It stabilizes the medial longitudinal arch along with the peroneus longus and also balances the pull of the peroneal muscles, protect the spring ligament, and invert and stabilize the hind foot during toe off. So, it is essential to assess tibialis posterior muscle.

when is it okay to do SAQ in ACL rehab

Typically, short-arc quadriceps training between 45 (or 30) degrees to extension (open-chain exercise), is held for 6-12 weeks (the patient is only 3 weeks post surgery), secondary to increased tibial anterior translation bringing about stress to the surgical area.

when would you see pronator drift?

UMN d/o (CVA, SCI, MS)

hyperreflexic bladder

UMN/spastic lesions above the conus medullaris and sacral segments - may contract and reflexively empty 2/2 to a certain filling pressure (reflex arc should be intact) can use: - suprapubic tapping - tapping directly over the bladder valsalva is used more for areflexive

if a patient has exaggerated deep tendon reflexes, what is expected of their superficial reflexes? (UMN)

UMNL: hyperactive deep tendon reflexes superficial reflexes such as the abdominal and cremasteric reflexes are absent bc they are polysynaptic reflexes that require a connection from the brain and spinal cord. In corticospinal tract lesions, these pathways are disrupted and superficial reflexes will be absent.

A physical therapist reads in the medical record that a patient developed a hematoma in their lower leg. Which diagnostic imaging method would have MOST likely been used to identify the presence of the hematoma? 1.X-ray 2.Bone scan 3.Arteriography 4.Ultrasound

US arteriography= angiography= invasive (uses dye and x-ray to look at blood vessels)

A physical therapist instructs the patient to perform lateral (side) flexion of the spine. Lateral (side) flexion of the spine correlates MOSTLY with which axis of the body? Select one: a. Mediolateral (ML) axis b. Anteroposterior (AP) axis c. Frontal plane d. Vertical axis

anteroposterior axis defined as being in frontal plane, but AROUND a-p axis

what kinds of drugs are these: - bupropion - cyclobenzaprine - clozapine - flexeril

anti-depressants, can also relieve mm spasms and used as mm relaxant * flexeril blocks pain sensation, mm relaxant side effects: - seizure - dry mouth - dizziness - headache - chest pain - seizures * cyclobenzaprine has anti-cholinergic properties: blurred vision, dry mouth, drowsiness

Sever's Disease

apophysitis at the attachment of the Achilles tendon on the calcaneus - stretch gastroc and soleus - use heel wedge to decrease stress and traction of Achilles insertion

A 48-year-old female lawyer presents to an outpatient clinic with history of low back pain. The PT measures her bilateral arch index at 0.30. Which of the following orthotic interventions is MOST appropriate for this patient?

arch index 0.30= flat feet, needs medial longitudinal arch support an arch index of more than 0.30 indicates low arch or pes planus. Providing longitudinal arch support will be most appropriate intervention. It can be supplemented by foot intrinsic muscle strengthening exercises

As part of a cognitive assessment, a physical therapist asks a patient to count from one to twenty-five by increments of three. What component of cognitive function does this task MOST accurately assess? 1.Attention 2.Constructional ability 3.Abstract ability 4.Orientation

attention: the capacity of the brain to process info from the environment or from long-term memory. attention can be assessed by asking patient to count in increments of 3. should be easy but requires patient to exert a sustained, consistent effort constructional ability: ask a person to copy figures consisting of varying shapes or sizes or to draw a known item such a clock abstract ability: ask patient to interpret a common proverb or describe similarities/differences between 2 objects

A fracture of the surgical neck of the humerus will most likely damage what nerve?

axillary n

posterior cord of the brachial plexus

axillary nerve radial thoracodorsal upper and lower subscap

A patient complains of right foot pain during long distance walking. During gait assessment, the Physical Therapist notices the calcaneus is going into eversion and there is an internal rotation of the tibia. Which foot orthotic device is most suitable to limit flexible calcaneal eversion? 1) lateral wedge 2) posterior leaf spring 3) calcaneovalgus correction post 4) valgus post

calcaneovalgus correction post is used for a flexile calcaneovalgus deformity a lateral wedge/valgus post will be used for fixed

2 days s/p coronary artery bypass graft: pulsus paradoxus, distended jugular veins, hypotension. what do you suspect?

cardiac tamponade

how to carry a child with CP and extensor tone/posturing

carry in a symmetric position that does not allow axial hyperext and keeps the hips and knees flexed sitting position: will promote visual attending, use of UE and social interaction. flexed posture preferred so shoulder's are forward

cataracts

cause opacity to the lens of the eyes: patient sees light streaks and glares from light sources - can be dangerous at night when driving

1) A patient sustained a forced flexion injury to the middle finger 1 day ago. The patient is unable to actively extend the PIP joint, but it can be fully extended passively and has a normal end feel into extension. Which is most likely injured? a. Volar plate of PIP joint b. Tendon of flexor digitorum profundus c. Central tendon of extensor digitorum d. Terminal tendon of extensor digitorum

central tendon of extensor digitorum

A 40 year-old male is admitted to a hospital after suffering from a head injury status post a MVA. He opens his eyes to painful stimuli, has an abnormal flexion response and makes incomprehensible sounds. What is his score on the Glasgow Coma Scale and related severity of his brain injury.

eye opening to painful stimuli: 2 abnormal flexion response: 3 incomprehensible sounds: 2 =7 (severe TBI)

What is the correct sequence of the spastic muscles from scapula to the wrist in a patient with hemiplegia?

flexor synergy - scapular retraction (rhomboids) - shoulder and elbow flexion (biceps) - forearm pronators (pronator quadratus) - wrist flexion (flexor carpi radialis) . The muscles most commonly affected due to spasticity are scapular retractors, shoulder adductors, depressors, ,internal rotators, elbow flexors, forearm pronators, wrist and finger flexors.

A physical therapist treats a patient with a sacral pressure ulcer by applying a foam dressing impregnated with charcoal. This type of dressing would be MOST beneficial in treating which type of wound? 1.A dry wound that is infected 2.A dry wound that has a fetid odor 3.A heavily exuding wound that is infected 4.A heavily exuding wound that has a fetid odor

foam + charcoal= exudating wound with fetid odor (dressing impregnated with silver may be more useful in treating infection 2/2 antimicrobial properties of silver)

A physical therapist makes footwear recommendations for a patient with foot pathology. The recommendations include a high and wide toe box, small to no heel, medial arch support, and a contoured posterior counter. This type of shoe prescription would be the MOST beneficial for a patient diagnosed with which of the following conditions? 1.Sesamoiditis 2.Hallux valgus 3.Pes cavus 4.Metatarsalgia

hallux valgus (bunion)- results from lateral deviation of hallux and foot pronation. prescribe: high and wide toe box (to reduce friction and pressure to 1st MTP joint) - medial support (to decrease pronation) - reduced heel height (decreases forefoot pressure) - contoured posterior counter (better controls the subtalar joint)

A patient with a diagnosis of Rheumatoid Arthritis presents to the clinic with flared-up symptoms after the flu 3 weeks ago. She complains of swelling in the distal joints of her bilateral arms and legs with redness. She reports that it takes about an hour to get out of bed in the morning due to severe pain and stiffness. Which of the following is the MOST LIKELY finding associated with the following cluster of signs and symptoms? 1) plantar fasciitis 2) heberden's nodes 3) hammertoe deformity 4) dorsal subluxation of the MCP joints

hammer toe: forefoot pain is often the first complaint with RA. hammertoe or "cock up" deformities are caused by extensor tendon shortening along with subluxation of the heads of the MTP joints. you will see VOLAR subluxation of the MCP joints and ulnar drift

C6 breathing pattern

have diaphragm but no abs or intercostals - will see abdomen going outward with inhalation and the inward motion of the upper chest

injury to central tendon of extensor digitorum

inability to actively extend PIP

atopognosis

inability to locate a sensation

Simultagnosia

inability to perceive more than one object at a time inability to perceive a visual stimulus as whole

Form discrimination

inability to perceive or attend to subtle differences in form and shape (ex confusing pen with a toothbrush)

weakness of triceps surae can result in

inadequate knee ext during stance

hip lateral distraction

inc general capsular motion can add IR or ER to maximize improvements in IR or ER

dec ankle DF can cause what at the knee during swing

inc knee flexion

which glide inc knee flexion? inf or superior

inferior glide inc knee flexion superior glide inc knee ext

A patient who has a peripheral nerve injury is examined in physical therapy. The patient's primary symptoms result from an injury to the superficial peroneal nerve. What location should the physical therapist expect to be the MOST likely area of sensory alteration? 1.Sole of the foot 2.Plantar surface of the toes 3.Lateral aspect of the leg and dorsum of the foot 4.Triangular area between the first and second toes

lateral aspect of the leg and dorsum of the foot deep peroneal will be the triangular area between the first and second toes sole of the foot & plantar surface of the toes= branches of the sciatic nerve

PD position for: upper lobes, posterior

leaning fwd over pillow

A 52-year-old female was admitted to a local hospital after having a stroke. The physical therapist noticed the patient had difficulty initiating tasks, demonstrated significant processing delays, and got easily distracted during gait training. Based on these symptoms, the lesion is MOST likely located in which part of the central nervous system? 1) R hemisphere 2) both hemi's 3) L hemi 4) R caudal medulla

left hemisphere lesion: - difficulty initiating tasks - processing delays - can be easily distracted and are slow and cautious

A physical therapist informs a patient that a ramp needed to access the patient's home with a wheelchair will require two separate sections with a landing area. What variable would have MOST likely influenced this decision? 1.Slope of the ramp 2.Angle of inclination of the ramp 3.Length of the ramp 4.Width of the ramp

length of ramp minimum ramp width 36 in slope should be no greater than 1:12 (1 in rise per 12 inc run)

macular degeneration

loss of central vision (looking forward) due to degenerative changes in the eyes can still see in periphery

A physician orders an electrocardiogram (ECG) for a patient diagnosed with congestive heart failure. The medical record indicates the patient is currently taking digitalis. What effect would digitalis MOST likely have on the patient's ECG? 1.Sinus tachycardia 2.Lengthened PR interval 3.Lengthened QT interval 4.Elevated ST segment

lengthened PR interval digitalis: increases force of myocontractility in HF patients. will inc cardiac output and dec preload/cardiac workload/myocardial O2 demand digitalis prolongs PR by inc conduction time thru AV node & may produce SHORTENING of QT & may produce sagging in ST segment

vestibulocerebellum

lesion would impair eye movements and postural mm control

which structures provide active compression of the urethra?

levator ani - pubococc - iliococc - puborec * pubovesical lig and anococc lig provide passive support

Cafe au lait spots

light brown macules on skin (benign)

You are treating a long distance runner in an outpatient physical therapy. Prior to performing a VO2 max test, you perform cardiac auscultation and hear an extra sound prior to systole. The patient's resting heart rate is 62 bpm, BP is 110/70, and O2 sats = 99%. Which of the following statements is MOST ACCURATE? a. The patient has an S4 heart sound secondary to non-pathological ventricular hypertrophy b. The patient has an S3 heart sound and is in congestive heart failure c. The patient has an S2 heart sound secondary to pulmonary hypertension d. The patient has an S4 heart sound secondary to hypertension

long distance runner= LV may be enlarged bc of the need for enlarged contractility S4 sound is consistent with ventricular hypertrophy

what is a long spinal fusion and what diagnosis is it used for commonly?

long- occurs across many levels fusion creates a solid bridge of bone between 2 or more adjacent vertebrae often used for scoliosis

correct measurement for seat depth

measure length from posterior buttock to posterior aspect of popliteal fossa and SUBTRACT 2 INCHES

do bronchodilators improve spirometry in emphysema patients?

no- · higher than normal RV (destroyed alveolar walls and enlarged air spaces), absent or mucoid sputum, spirometry unimproved with bronchodilators

will complete C5 be independent with rolling?

no- intact delt, biceps, rhomboid but no pec major or teres major should be able to perform surface level transfers with assistance

small diameter fiber cells (c fibers) are stimulated by:

noxious stimuli

TMJ anterior glide

opening mouth

Glascow Coma Scale

out of 15 points EYE OPENING - (4) spontaneous - (3) verbal stimuli - (2) pain - (1) no response VERBAL RESPONSE - (4) oriented - (3) confused but can answer questions - (2) incomprehensible speech - (1) no speech MOTOR RESPONSE - (6) obeys command for movement - (5) purposeful response to pain - (4) withdrawal response to pain - (3) flexion response to pain (decorticate) - (2) extension response to pain (decerebrate) - (1) no response >8: severe head injury 9-12: moderate head injury 13-15: mild head injury

excessive femoral retroversion

out toe, inc ER, less IR coxa valga increased supination tibial ER knee varus compensations: tibial IR, knee valgus, pronation

pleural effusion characteristics

progressive DOE nonspecific chest discomfort, can be pleuritic (sharp, stabbing pain exacerbated by coughing or breathing that changes with position) may see: weight loss, fever with TB, cancer, signs of heart failure tachy, distant heart sounds, fixed JVD, edema, paradoxical pulse

posterior radioulnar glide (distal) increases:

pronation

PD position for: lower superior segments

prone with bed flat

PD position for: lower lobes

prone, LE raised 18 in

MMT for gastroc

prone: poor 3/5 fair: 1 heel raise 4/5: 2-25 5/5 normal minimum 25

A physical therapist monitors the vital signs of a patient running on a treadmill at a series of steadily increasing speeds. A change in which variable would be MOST responsible for an observed increase in pulse pressure during the exercise session? 1.Heart rate 2.Systolic blood pressure 3.Diastolic blood pressure 4.Cardiac output

pulse pressure is the difference between systolic and diastolic pressure inc in direct proportion to the intensity of the exercise 2) SBP

supinator innervation

radial C6

A physical therapist inspects the static wrist and hand position of a patient who has advanced rheumatoid arthritis. Which positioning would MOST likely be observed based on the medical diagnosis? 1.Radial deviation of the radiocarpal joint and radial deviation of the fingers 2.Radial deviation of the radiocarpal joint and ulnar deviation of the fingers 3.Ulnar deviation of the radiocarpal joint and radial deviation of the fingers 4.Ulnar deviation of the radiocarpal joint and ulnar deviation of the fingers

radial deviation of the radiocarpal joint and ulnar deviation of the fingers

glide to improve thumb extension (swing upwards)

radial glide of 1st CMC (same direction)

A patient is being evaluated in a hospital following acute kidney failure. During evaluation, the PT notes that the patient has ovular patches of skin that have hardened, and the patient's nail beds appear blue.

radial nerve The nerve may be damaged as it winds around behind the humerus in the radial groove and injury may occur at the time of the fracture, or the nerve may get caught in the callus of fracture healing.

which nerve can be injured following a midshaft humerus fx

radial nerve is in direct contact with the shaft of the humerus and may be injured (ECRL)

Souque's phenomenon

raising the involved UE above 100 degrees with elbow extension with produce extension and abduction of the fingers

A 56-year-old male arrived at an outpatient physical therapy clinic with chief complaints of low back pain with past medical history of hyperlipidemia, CHF, HTN. During evaluation, the patient provided a list of medications with one of them being Ramipril. Which of the following side effects MOST likely correlates with this drug? Select one: a. Increases glomerular filtration rate b. Regulates heart rate c. Causes hypokalemia d. Causes hyperkalemia

ramipril is an ace inhibitor side effects: - hypERkalemia - acute kidney failure - dec in glomerular filtration rate - irregular HR - dizziness - dry hacking cough

ITB syndrome

repetitive stress injury that results from friction to the ITB as it slides over the prominent lateral femoral condyle at approx 30 deg of knee flexion commonly found among cyclists 2/2 pedalling stroke, which causes the ITB to be pulled anteriorly during downstroke and posteriorly during up stroke extrinsic factors: - excessive bike seat height - IR cleats (tibia IR, valgus at knee= stresses ITB)

A patient completes a D1 extension pattern for the upper extremity. The prime movers of the scapula during this pattern are the: A. trapezius and middle deltoid B. pectoralis minor and pectoralis major C. serratus anterior, pectoralis major, and anterior deltoid D. rhomboids, pectoralis minor, and levator scapula

rhomboids, pec minor, lev scap D1 extension: scapular depression, extension, adduction, and downward rotation The pectoralis minor assists with both depression and downward rotation. The rhomboids assist with adduction and downward rotation, while the levator scapulae assists with downward rotation.

An injury to the C5 nerve root will MOST LIKELY lead to clinical presentation of 0/5 on manual muscle testing for which of the following muscles? 1) biceps 2) serratus 3) deltoid 4) rhomboid

rhomboids: C5 nerve root biceps: c5,6 deltoid: c5,6 serratus: c5,6,7

tinea pedis and tinea corporis precautions

ring worm/athlete foot standard pxns

how is hematocrit affected by burns

rises immediately after a severe burn and gradually reduces with fluid replacement

A physical therapist treats a patient wearing a shoe that incorporates a rocker bottom. This type of modification would be the MOST beneficial for a patient diagnosed with which of the following conditions? 1) achilles tendonitis 2) hallux rigidis 3) plantar fasciitis 4) posterior tibial tendonitis

rocker bottom shoe has thicker than normal sole with a rounded heel. this will serve to relieve metatarsal pain, shorten the gait cycle, and assist with DF rocker bottom will reduce extension of hallux during normal gait

persistence of tonic labyrinthine reflex can interfere with:

rolling TLR: flexion bias in prone, extension bias in supine

deformity; excessive ext at MCP and DIP joint flexion of PIP joint

rupture of central tendon

cause of trigger finger

rupture of the extensor tendon at the distal phalanx

PD position for: R lateral segment

s/l with legs raised 18

atrial flutter

sawtooth P wave pattern; greater than 1 P wave prior to each QRS; atrial depolarization rate of 250-350 times per minute; QRS may be typical and identical, with a duration between 0.06-0.10 second.

A physical therapist would like to assess the properties of Adson's test in diagnosing patients with thoracic outlet syndrome (TOS). Out of the 125 patients who had TOS, 25 tested positive and the remaining tested negative. And out of the 40 patients who did not have TOS, 9 tested positive and rest tested negative. Which of the following is the MOST appropriate conclusion based on these findings? Select one: a. Sensitivity- 0.6 b. Sensitivity-0.8 c. Specificity-0.2 d. Specificity- 0.7

sensitive= true positive. recognize the condition when present 25/125= 0.2 specificity= true negative 31/40 D

equation for sensitivity

sensitivity= (true positive) / (true positive + false neg)

Meissner's corpuscles

sensory receptor in the dermis DISCRIMINATIVE TOUCH and movement of objects over skin

Apprehension sign

shoulder abducted 90 degrees, elbow at 90 degrees, shoulder ext rotated, + is apprehension tensing up anterior GH instability

ULTT for ulnar nerve

shoulder depression ER and ABD (10-90 deg) elbow FLEXION supination, wrist extension, radial deviation finger and thumb extension

ULTT Radial Nerve

shoulder depression & abduction elbow extension, forearm pronation, wrist flexion, wrist ulnar deviation

A 66 yr old patient was admitted to ICU following a cerebrovascular accident. The physician orders PT services to address position recommendations when lying on the hemiplegic side. For the protection of shoulder, the MOST appropriate position to place the patient's shoulder in is: A. Shoulder forward, slight abduction & external rotation B. Shoulder neutral, slight abduction only C. Shoulder forward, slight abduction and internal rotation D. Shoulder neutral, slight abduction & external rotation

shoulder forward, slight abd, ER Positioning strategy in side lying on the more affected side - Scapula protracted; shoulder forward; arm placed in slight abduction and external rotation; elbow extended, forearm supinated, wrist neutral, fingers extended, and thumb abducted.

pt with wedging of vertebral bodies. which mm should be stretched

shoulder horiz ADD and shoulder IR hip flexor and hip IR

orthosis for S1 myelomeningocele with poor (2/5) gastroc?

solid AFO - S1: weakness in mm of posterior leg and tibia without affecting mm strength in hips and knees

L3 disc hern

sluggish knee jerk reflex positive prone bend pain on full flexion inner knee and ant lower leg paresthesia

LCPD

small boys, 3-13 years may have LLD primary: antalgic limping with ABD weakness

A 45-year-old male patient with PMH of type II diabetes mellitus, coronary artery bypass grafting, below knee amputation of left lower extremity is being trained by a physical therapist with a prosthesis. Currently, the patient is demonstrating an early left knee flexion during late stance. The therapist concludes that this deviation MOST likely is contributed from: 1) socket too posterior 2) DF bumper too soft 3) socket insufficiently flexed 4) low shoe heel

soft DF bumper causes early knee flexion during late stance

equation for specificity

specificity= (true negative) / (true negative + false pos)

Telangiectasia

spider angioma

Presence of which of the following reflexes can be considered a normal finding in a 9 month old infant? Select one: a. Flexion, adduction and extension of the right lower extremity when a noxious stimulus is applied to the ball of the foot of the left lower extremity that is fixed in extension b. A sudden loud, harsh noise brings about extension or abduction of the upper extremities and crying c. Maintained flexion of the fingers as pressure is applied (and maintained) to the palm of the hand d. A sudden drop backward from sitting brings about extension and abduction of upper extremities, hand opening, and crying, followed by flexion and adduction of the upper extremities across the chest

startle reflex: a sudden loud, harsh noise brings about extension and ABD of UE and crying. this will persist past 9 months

A 28 year-old male reports having right shoulder pain, especially while reaching overhead, and after performing repetitive activity above the shoulder. He also mentions that he has difficulty sleeping at night on the right shoulder secondary to pain. During the examination, the Physical Therapist notes the patient has significant weakness with resisted abduction, and during forward flexion. The Physical Therapists observes the patient has a painful arc between 70 degrees and 120 degrees of abduction. The Physical Therapist performed several specials tests: Yocum test (-), clunk test (-), drop arm test (-). Which of the following diagnosis/pathology is BEST fits? 1) labral tear 2) supraspin tendonitis 3) AC impingement 4) RTC

supraspin: weakness with resisted ABD and during forward flex + painful arc

a 26 year old male has a history of unrepaired neurotmesis of superficial peroneal nerve. Which foot deformity and cutaneous sensation loss should be expected in this patient?

the superficial peroneal nerve innervates the evertors if the evertors are weak, the foot is pulled into inversion and PF = equinovarus deformity cutaneous sensation: dorsum of foot, anterolateral aspect of leg (except webspace between 1st 2 toes is deep peroneal)

A Physical Therapist is walking a patient to the rehab unit when he complains of frequent urination, weakness, and muscle cramps. On looking through his charts, the Physical Therapist notices that his ECG shows flattened T waves. The patient reports that they recently changed blood pressure medication . Which of the following medications can cause the symptoms and ECG changes as observed in this patient? 1) ace inhib 2) thiazides 3) BB 4) Ca2+ channel blockers

thiazides= water pills (diuretics) used to treat htn can cause excessive loss of potassium leading to hypokalemia side effects of thiazides: nausea, weakness, polyuria, hyperglycemia, hypokalemia, postural hypotension

A lymphedema Physical Therapist is performing manual lymphatic drainage with effleurage directed toward the thoracic duct. Which of the following areas CANNOT be drained into the thoracic duct? 1. Right upper extremity and right upper trunk 2. Right lower extremity and right lower trunk 3. Left lower extremity and left lower trunk 4. Left upper extremity and left upper trunk

thoracic duct drains L and under diaphragm 1

what mm does the deep peroneal supply? where is the sensory distribution?

tib ant extensor digitorum brevis EDL EHL peroneus tertius altered sensation around webspace between 1st and 2nd toes injury to deep peroneal n can cause foot drop, which may lead to high steppage gait and inability to evert the foot

weak tib ant: what phase of gait should you apply FES

tib ant: DF initial swing to mid swing in order to prevent toe drag and produce appropriate ground clearance ground clearance occurs during swing phase from initial swing to mid swing

A physical therapist attempts to palpate the tibialis posterior tendon. Which of the following activities should the therapist do to facilitate palpation of this structure? 1.Ask the patient to invert and plantar flex the foot 2.Ask the patient to evert and dorsiflex the foot 3.Ask the patient to invert and dorsiflex the foot 4.Passively evert and plantar flex the foot

tib post: PF and inverts

compensation for functional forefoot valgus

tibia IR

which way does tibia move in OPEN CHAIN knee flexion

tibia rolls and glides posteriorly

glide to improve knee extension

tibiofemoral anterior glide

normal biomechanics of open chain knee flexion

tibiofemoral joint: posterior glide patella: inferior glide tibia: IR

lateral trunk shift during midstance is caused by

tight contralateral QL

lower crossed syndrome

tight hip flexor and erector spinae weak abs and gluts APT: inc lordosis, slight hip flexion

lordosis in stance

tight hip flexors = high anterior wall weak extensors= low posterior wall

high anterior wall

tight hip flexors, pulling pelvis into an anterior pelvic tilt

in prosthetics, high wall =

tight mm

type 2 heart block type 1 vs 2

type 1: wenckebach (longer, longer, longer, drop) - monitor exercise type 2 mobitz: drops without progressively longer QR intervals - stop or slow patient down

in prosthetics, low wall=

weak mm

low anterior thigh wall

weak quads

why would you see forward flexion in stance?

weak quads prosthetic: low anterior wall

L4 lesion

weak tib ant weak toe ext (extensor hallucis) paresthesias in the medial aspect of calf and ankle, weak knee jerk and a limited straight leg raise.

asthenia

weakness - inability to hold against gravity with manual resistance training

L5 lesions

weakness of extensor hallucis, peroneals, gluteus medius, ankle dorsiflexors, hamstrings along with paresthesias on the lateral aspect of leg, medial three toes with decreased ankle jerk and painful crossed straight leg raise.

involvement of C8 myotome

weakness of thumb extension

A physical therapist consults with a teacher regarding a child who has impairments in sensory processing. Which piece of equipment would be the MOST useful to address the child's dyspraxia? 1.Swing 2.Weighted vest 3.Sit and spin 4.Rocking chair

weighted vest can provide proprioceptive input and provides child with an improved sense of position and understanding of where joints and mm are in space other options are more vestibular input

A patient arrived at a burn unit with large, severe (major) burns over the anterior chest, abdomen, bilateral arms and the genital area. Which of the following responses would MOST likely be seen in this case INITIALLY? Select one: a. Decrease cardiac output (CO) b. Decrease metabolic activity c. Bradycardia d. Increase cardiac output (CO)

with major burns, you initially see a decrease in cardiac output CO may be dec as much as 50% for the first 2-4 2/2 to a severe burn - sinus tachy may present within hours of burn as a way to maintain CO - will see a hypermetabolic state that will peak 7-17 days after the injury and then slowly return to normal as the wound closes

Wrist flexor tendon repair

wrist is immobilized for up to 5 days after surgery Zane 1,2,3 repair: immobilized in 10 to 45 deg wrist flexin and 40-70 deg MCP flexion IP joints are in extension

xanthoma

yellow tumor (benign, primarily in the skin) fatty, fibrous, yellow plaques, especially around tendonds seen in uncontrolled DM and d/o that affect lipid metabolism

are manips contraindicated for spondy?

yes

are spinal defecation reflexes intact with spastic bladder?

yes, level of cord injury above S2-4

if you have advanced OA in the medial compartment of the knee, which way will you lean

you will lean IPSILATERALLY to bring COM laterally and transfer the load to the lateral compartment R medial OA: R trunk lean

dystonia

· co-contraction of the agonist and antagonist mm and is associated with BG dysfunction - Segmental: 2 or more adjoining body regions dominated by sustained mm contractions, causing twisting and repetitive movements and abnormal postures

ulnar nerve entrapment signs/sx

·Ulnar nerve entrapment as it courses thru hook of hamate and pisiform can lead to: - Paresthesia along the ulnar side of the hand into the palmar (volar) aspect of little finger and medial half of ring finger. - Weakness of hypothenar: difficulty with gripping (opening and turning doorknobs) or repetitive gripping such as stapling, knitting, tying knots - weakness in medial aspect of hand after prolonged stapling


Related study sets

#2 Checkpoint Exam: Network Access

View Set

Combo with "sociology midterm chapter 1+3+4+5+6" and 23 others

View Set

Unit 1 Edhesive AP Computer Science

View Set

BIO139FALL2021/CHAP19HEART/SALADIN

View Set

AP Optional Final Semester 1 - Review

View Set

Ancient Egypt - Important Places

View Set